<<

52

TARGET PT - 2019

CURRENT AFFAIRS (ART & CULTURE)

Time Allowed: 1 Hrs. Maximum Marks: 100

Roll No.:

INSTRUCTIONS

1. IMMEDIATELY AFTER THE COMMENCEMENT OF THE EXAMINATION, YOU SHOULD CHECK THAT THIS TEST BOOKLET DOES NOT HAVE ANY UNPRINTED OR TORN OR MISSING PAGES OR ITEMS, ETC. IF SO, GET IT REPLACED BY A COMPLETE TEST BOOKLET. 2. Please note that it is the candidate’s responsibility to encode and fi ll in the Roll Number carefully and without any omission or discrepancy at the appropriate places in the OMR Answer Sheet. Any omission/ discrepancy will render the Answer Sheet liable for rejection. 3. You have to enter your Roll Number on the Test Booklet in the Box provided alongside. DO NOT write anything else on the Test Booklet. 4. While writing name and Roll No. on the top of the OMR Sheet in appropriate boxes use “ONLY BLACK BALL POINT PEN”. 5. This Test Booklet contains 50 items (questions). Each item comprises four responses (answers). You will select the response which you want to mark on the Answer Sheet. In case you feel that there is more than one correct response, mark the response which you consider the best. In any case, choose ONLY ONE response for each item. 6. You have to mark all your responses ONLY on the separate OMR Answer Sheet provided. 7. All items carry equal marks. (2 marks each) 8. After you have completed fi lling in all your responses on the Answer Sheet and the examination has concluded, you should hand over to the Invigilator only the Answer Sheet. You are permitted to take away with you the Test Booklet. 9. Sheets for rough work areGS appended in the TestSCORE Booklet at the end. 10. Penalty for wrong answers: THERE WILL BE PENALTY FOR WRONG ANSWERS MARKED BY A CANDIDATE IN THE OBJECTIVE TYPE QUESTION PAPERS. (a) There are four alternatives for the answer to every question. For each question for which a wrong answer has been given by the candidate, one-third of the marks assigned to that question will be deducted as penalty. (b) If a candidate gives more than one answer, it will be treated as a wrong answer even if one of the given answers happens to be correct and there will be same penalty as above to that question. (c) If a question is left blank, i.e., no answer is given by the candidate, there will be no penalty for that question.

DO NOT OPEN THIS TEST BOOKLET UNTIL YOU ARE ASKED TO DO SO

1 TPT2019/B-1/012019/52 1. Match the following pairs: Which of the above statements is/are correct? Folk Dances Concerned State (a) 1 only A. Karakattam 1. Gujarat (b) 2 only B. Taakala 2. Tamil Nadu (c) Both 1 and 2 C. Misra Raas 3. Maharashtra (d) Neither 1 nor 2 Select the correct answer using the code given below: 5. With reference to Swadesh Darshan A B C Scheme, consider the following (a) 2 3 1 statements: 1. It is launched by Ministry of Culture with (b) 3 1 2 the aim to develop theme based tourist (c) 2 1 3 circuit. (d) 1 2 3 2. These tourist circuits will be developed on the principles of high tourist value, competitiveness and sustainability in an 2. With reference to Bagru Block printing, integrated manner. consider the following statements: Which of the above statements is/are correct? 1. It is a traditional technique of printing with natural colour done in Gujarat. (a) 1 only 2. The main colors used in Bagru are Red (b) 2 only and Black. (c) Both 1 and 2 3. Traditionally, motifs printed at Bagru are (d) Neither 1 nor 2 large with bold lines. Which of the above statements are correct? 6. Consider the following statements: (a) 1 and 2 only 1. Archeological Survey of (ASI) was founded in 1861 by Alexander (b) 1 and 3 only Cunningham. (c) 2 and 3 only 2. The protection and maintenance of (d) 1, 2 and 3 monuments, declared as of national importance is taken up by Geological Survey of India. 3. With reference to Maithili language consider the following statements: Which of the above statements is/are correct? 1. The language has been accorded a (a) 1 only constitutional status under the 8th (b) 2 only Schedule of the constitution. (c) Both 1 and 2 2. Mithilakshar or Tirhuta is the script of the language Maithili. (d) Neither 1 nor 2 Which of the aboveGS statements is/are correct?SCORE 7. With reference to Adopt a Heritage (a) 1 only project consider the following (b) 2 only statements: (c) Both 1 and 2 1. Under this project , Monument Mitra will be selected among the private sector (d) Neither 1 nor 2 company, public sector company and individuals. 4. Consider the following statements: 2. No funds are given by Ministry of 1. The Ministry of Culture has organized Tourism. Sanskriti Kumbh at Prayagraj, Uttar Which of the above statements is/are correct? Pradesh recently. (a) 1 only 2. UNESCO has inscribed Kumbh Mela on the list of Intangible Cultural Heritage. (b) 2 only

2 (c) Both 1 and 2 11. With reference to Khirki mosque (d) Neither 1 nor 2 consider the following statements: 1. It is built with rubble stone. 8. With reference to Badshahi 2. The mosque was built by Khan-i-Jahan Ashoorkhana consider the following Junan Shah, the Prime Minister of Firoz statements: Shah Tughluq. 1. It was built by Muhammed Quli Qutb 3. It is an example of closed mosque in Shah. northern India. 2. It was a house of mourning of Sunni Which of the above statements are correct? muslims. (a) 1 and 2 only 3. During the rule of the Mughal Emperor Aurangzeb, the Ashoorkhana was (b) 2 only converted into a bandikhana. (c) 2 and 3 only Which of the above statements are correct? (d) 1, 2 and 3 (a) 1 and 2 only (b) 1 and 3 only 12. Which of the following is/are correct (c) 2 and 3 only regarding Kalamkari painting? (d) 1, 2 and 3 1. It depicts only the stories from puranas. 2. It is concentrated primarily in Andhra 9. With reference to Konark Sun Temple Pradesh. consider the following statements: 3. Kalamkari paintings use only natural dyes. 1. It was built by King Narasimhadeva I, the great ruler of Satavahana dynasty. Select the correct answer using the code given 2. It is a classic illustration of the below: style of Hindu temple architecture. (a) 1 only 3. This temple is also known as Black (b) 1 and 2 only Pagoda. (c) 2 and 3 only Which of the above statements are correct? (d) 1, 2 and 3 (a) 1 only (b) 3 only 13. ‘Togalu Gombeyatta, Tholu Bommalata (c) 2 and 3 only and Ravanachhaya’ are which form of (d) 1, 2 and 3 puppetry that is practiced in India? (a) Shadow puppetry 10. Consider the following statements (b) Rod puppetry regarding Rani-ki-vav:GS SCORE (c) Glove puppetry 1. It is an intricately constructed stepwell situated in Gujarat. (d) String puppetry 2. The stepwell is located on the banks of Sabarmati River. 14. Consider the following statements 3. It was built by Queen Udayamati of the regarding mithila paintings: Solanki Dynasty. 1. It originated in small village known as Which of the above statements are correct? Maithili in Bihar. (a) 1 and 2 only 2. The various styles of Mithila painting include Bharni, Tantrik, Katchni, Godna, (b) 1 and 3 only and Kohbar. (c) 3 only 3. The central themes of Mithila paintings (d) 1, 2 and 3 are based on nature.

3 Which of the above statements are correct? 18. With reference to Ambubachi Mela (a) 1 and 2 only consider the following statements: (b) 3 only 1. The festival is also known as Mahakumbh of the East. (c) 2 and 3 only 2. It symbolises the fertility cult of goddess (d) 1, 2 and 3 Kamakhya. Which of the above statements is/are correct? 15. With reference to Kambala, consider the following statements: (a) 1 only 1. It is traditional slush track buff alo race. (b) 2 only 2. It is held annually in the coastal regions of (c) Both 1 and 2 Tamil Nadu. (d) Neither 1 nor 2 3. The sports season generally starts in November and lasts till March. 19. With reference to World Hindi Which of the above statements are correct? conference consider the following (a) 1 and 2 only statements: (b) 1 and 3 only 1. It is major event organised biennially by Ministry of Culture. (c) 2 and 3 only 2. The 11th World Hindi Conference (WHC) (d) 1, 2 and 3 was organized in Mauritius. 3. The theme of the Conference will be 16. Consider the following statements: “Vaishvik Hindi Aur Bharatiy Sanskriti“. 1. Walmiki and Malhar languages are Which of the above statements are correct? categorised endangered. 2. Malhar belongs to “North Dravidian” (a) 1 only subgroup of Dravidian family of (b) 3 only languages. (c) 2 and 3 only 3. Walmiki is spoken in district of Koraput of Odisha and on bordering districts of (d) 1, 2 and 3 Andhra Pradesh. Which of the above statements are correct? 20. Consider the following statements (a) 2 only regarding Guru Padmasambhava: 1. He is considered to be the founder of (b) 1 and 3 only Nyingma tradition. (c) 2 and 3 only 2. He was also known as Guru Rinpoche is (d) 1, 2 and 3 widely venerated as a “Second Buddha”. Which of the above statements is/are correct? 17. With reference GS to Nongkrem DanceSCORE (a) 1 only Festival consider the following statements: (b) 2 only 1. It is celebrated for the harvest thanksgiving (c) Both 1 and 2 by the Khasi tribe. (d) Neither 1 nor 2 2. It is the most important festival of the Khyrim state. 21. Which of the following is/ are not Which of the above statements is/are correct? present in a stupa? (a) 1 only 1. Medhi (b) 2 only 2. Shaft (c) Both 1 and 2 3. Abacus (d) Neither 1 nor 2 4. Hermika

4 Select the correct answer using the code given 25. Which of the following are the below: benefi ts of World heritage Sites Status (a) 1 and 2 only accorded by UNESCO? (b) 2 and 3 only 1. It enables access to global project (c) 3 only management resources. (d) 4 only 2. It facilitates creating partnerships between government, the private sector, and NGOs to achieve conservation goals. 22. Consider the following statements regarding various types of Paintings 3. Site becomes protected under the Geneva in India: Convention against destruction or misuse during wartime. 1. “What happens to human life after death” is a theme used in Paitkar paintings Select the correct answer using the code given 2. Warli painting is also known as Snake below: Painting (a) 1 and 2 only Which of the above statements is/are correct? (b) 2 and 3 only (a) 1 only (c) 3 only (b) 2 only (d) 1, 2 and 3 (c) Both 1 and 2 (d) Neither 1 nor 2 26. Which of the following statements is/are correct about Thumri, a semi- 23. Consider the following statements classical Indian music? regarding Sharqi style of 1. It was inspired by the Bhakti movement. Architecture: 2. The language of the compositions is 1. Sharqi style of architecture avoided the usually Hindi or Awadhi dialect. use of minars. 2. A unique feature of the buildings here is Select the correct answer using the code given the use of bold and forceful characters below: painted on huge screens. (a) 1 only Which of the above statements is/are correct? (b) 2 only (a) 1 only (c) Both 1 and 2 (b) 2 only (d) Neither 1 nor 2 (c) Both 1 and 2 (d) Neither 1 nor 2 27. Consider the following pairs: Folk Theatre Regions of Popularity 24. Consider the following statements regarding PahadiGS Style of Painting: SCORE1. Bhavai Gujarat and Rajasthan 1. Gita Govinda and Bhagwata Purana were one of the popular subjects of kangra 2. Daskathia Assam school of Paintings 3. Kariyila Himachal Pradesh 2. A famous group of paintings known as 4. Maach Odisha ‘Twelve months’ belongs to Dogra School of Painting. Which of the above pairs are correctly Which of the above statements is/are correct? matched? (a) 1 only (a) 1 and 2 only (b) 2 only (b) 2 and 4 only (c) Both 1 and 2 (c) 1 and 3 only (d) Neither 1 nor 2 (d) 1, 2 and 3 only

5 28. Which of the following is not the Which of the above statements are correct? criterial requirements for a language to (a) 1 and 2 only be accorded the status of a “Classical (b) 2 only Language in India”? (c) 2 and 3 only (a) High antiquity of its early texts/recorded history over a period of 1500–2000 years; (d) 1, 2 and 3 (b) A body of ancient literature/texts, which is considered a valuable heritage by 32. Kuthiyottam ritual is performed in generations of speakers; which Indian State? (c) The literary tradition be original and (a) Kerala not borrowed from another speech (b) Karnataka community; (c) Andhra Pradesh (d) None (d) Tamil Nadu

29. Which of the following statements 33. Consider the following statements regarding European Architectures regarding Mahamasthakabhisheka: styles in India is incorrect? 1. It is the head anointing ceremony which (a) The Portuguese brought with them the is performed once in 12 years to the Iberian style of architecture monolithic statue of Lord Bahubali. (b) The French brought the concept of urban 2. It was built by Chandragupta Maurya. city planning with them. Which of the above statements is/are correct? (c) The British brought with them the Gothic (a) 1 only style of architecture (b) 2 only (d) The Cartesian grid plans and scientifi c (c) Both 1 and 2 architectural designs were the prominent features of Iberian Architecture (d) Neither 1 nor 2

30. Consider the following pairs: 34. With reference to Chithirai festival consider the following statements: Sun Temples in India State 1. It is celebrated every year at the Meenakshi 1. Modhera Sun Temple Gujarat Temple in Madurai, Tamil Nadu. 2. Konark Temple Odisha 2. The festival symbolizes the unity of 3. Dakshinaark Temple Himachal saivitism and vaishnavism. Pradesh Which of the above statements is/are correct? 4. Martand Sun Temple Maharastra (a) 1 only Which of the above pairs are correctly (b) 2 only matched? GS SCORE(c) Both 1 and 2 (a) 1 and 2 only (d) Neither 1 nor 2 (b) 2 and 4 only (c) 1 and 3 only 35. Consider the following statements (d) 1, 2 and 3 only regarding Char Dham Yatra: 1. The Yatra begins from Gangotri. 31. With reference to Vishishtadvaita 2. The Chardham Development Plan is the consider the following statements: project of the Ministry of Road Transport and Highways. 1. It advocates Bhakti to attain God. Which of the above statements is/are correct? 2. It is a non-dualistic school of Vedanta philosophy. (a) 1 only 3. Its proponent was Ramananda. (b) 2 only

6 (c) Both 1 and 2 Which of the above statements are correct? (d) Neither 1 nor 2 (a) 1 and 2 only (b) 1 and 3 only 36. Match the following: (c) 2 and 3 only Harvest Festivals Concerned State (d) 1, 2 and 3 A. Nuakhai 1. West Bengal B. Nabanna 2. Assam 40. With reference to PRASAD Scheme consider the following statements: C. Bhogali Bihu 3. Odisha 1. It was launched by Ministry of Culture. Select the correct answer using the code given below: 2. It focuses on the development and beautifi cation of the identifi ed pilgrimage A B C destinations.

(a) 3 1 2 Which of the above statements is/are incorrect? (b) 2 1 3 (a) 1 only (c) 3 2 1 (b) 2 only (d) 1 3 2 (c) Both 1 and 2 37. Pakke Paga Hornbill Festival is been (d) Neither 1 nor 2 declared as the state festival of which of the following states? 41. Which of the following personalities has not received Sahitya Akademi (a) Nagaland Award? (b) Arunachal Pradesh (a) Ramdhari Singh Dinkar (c) Manipur (b) R.K Narayan (d) Mizoram (c) Harivansh Rai Bachchan (d) Gulzar 38. With reference to Sadharan Brahmo Samaj consider the following statements: 42. Which of the following features belong to Vijaynagar School of Temple 1. It was founded by Ram Mohan Roy. Architecture? 2. It believes in the immortality of the human (a) Goupurams, which were previously soul. present on the front side, were now built Which of the above statements is/are correct? on all the sides. (a) 1 only (b) The concept of secular buildings inside the temple premises was also (b) 2 only GS SCOREintroduced during this period (c) Both 1 and 2 (c) Temples have more than one Mandapas (d) Neither 1 nor 2 (d) All of the above

39. Consider the following statements 43. Which of the following statements regarding Sangeet Natak Akademi: is/are correct about kuchipudi dance 1. It is the nodal agency of the Ministry of form? Culture. 1. The Kuchipudi dance style is a 2. It has launched the second phase of manifestation of the divine elements in the Web Campaign ‘Sanjhi-Mujh Mein the human body. Kalakar’. 2. A Kuchipudi recital is generally 3. It is an initiative to document and promote accompanied with Carnatic music in the Intangible Cultural Heritage (ICH). Telugu Language.

7 Select the correct answer using the code given 47. With reference to Cultural Institutions below: in India, which of the following (a) 1 only statements is/are correct? (b) 2 only 1. Crafts Council of India or CCI is a non-profi t organisation that promotes activities to (c) Both 1 and 2 preserve and develop handicraft industry (d) Neither 1 nor 2 in India. 2. The Indira Gandhi National Centre For 44. Which of the following statements is/ The Arts (IGNCA) is an autonomous institution that concentrates on research, are correct about Samkhya school of conservation, display and dissemination of Philosophy? arts. 1. Samkhya School argued that salvation Select the correct answer using the code given could be attained through the acquisition below; of knowledge (a) 1 only 2. This school believed in dualism, i.e. the soul and the matter are separate entities. (b) 2 only (c) Both 1 and 2 Select the correct answer using the code given below: (d) Neither 1 nor 2 (a) 1 only 48. Which of the following statements (b) 2 only regarding Sahitya Academi is (c) Both 1 and 2 incorrect? (d) Neither 1 nor 2 (a) Apart from 22 languages listed in 8th schedule, Sahitya Academi gives 45. Consider the following statements: recognition to English and Rajasthani 1. Chaitanya Mahaprabhu was an ardent (b) Bhasha Samman is awarded by Sahitya Krishna devotee and popularized the Academi for signifi cant contribution to the chanting of “Hare Rama, Hare Krishna”. languages recognised by the Academi. 2. Guru Nanak was inspired by Kabir Das and (c) Premchand Fellowship is awarded by he was the proponent of Nirguna School Sahitya Academi to those who have contributed to the literature in Hindi. Which of the above statements is/are correct? (d) None (a) 1 only (b) 2 only 49. Consider the following pairs: (c) Both 1 and 2 Religious Founder (d) Neither 1 nor 2 Movement GS SCORE1. Ahmadiya Mirza Ghulam 46. With reference to Bhakti Movement, Movement Ahmad which of the following statements is 2. Faraizi Haji Shariatullah incorrect? Movement 3. Aligarh Sir Sayyid Ahmad (a) Bhakti movement originated in the Movement Khan Southern parts of India, especially Tamil Nadu between 7th and 12th century; Which of the above pairs are correctly matched? (b) Ramanand was the fi rst bhakti saint to use Hindi for the propagation of his message (a) 1 only (c) Bijak refers to the composition of saint (b) 1 and 2 only Kabir. (c) 2 and 3 only (d) None (d) 1, 2 and 3

8 50. Consider the following statements Which of the above statements is/are correct? regarding Folk paintings in India: (a) 1 only 1. The themes of paintings are (b) 2 only mainly inspired from and Vaishnava cult, (c) Both 1 and 2 2. Kalighat paintings depicted religious (d) Neither 1 nor 2 note but later themes expressed social sentiments.

**********

GS SCORE

9 52

TARGET PT - 2019

CURRENT AFFAIRS (ART AND CULTURE)

Answer Key

Q. 1 (a) Q. 14 (a) Q. 27 (c) Q. 40 (a)

Q. 2 (c) Q. 15 (b) Q. 28 (d) Q. 41 (d)

Q. 3 (c) Q. 16 (d) Q. 29 (d) Q. 42 (d)

Q. 4 (c) Q. 17 (c) Q. 30 (a) Q. 43 (b)

Q. 5 (d) Q. 18 (c) Q. 31 (a) Q. 44 (c)

Q. 6 (a) Q. 19 (c) Q. 32 (a) Q. 45 (c)

Q. 7 (c) Q. 20 (c) Q. 33 (a) Q. 46 (d)

Q. 8 (b) GSQ. 21 (b) SCOREQ. 34 (c) Q. 47 (c)

Q. 9 (c) Q. 22 (a) Q. 35 (b) Q. 48 (b)

Q. 10 (b) Q. 23 (c) Q. 36 (a) Q. 49 (d)

Q. 11 (d) Q. 24 (a) Q. 37 (b) Q. 50 (c)

Q. 12 (c) Q. 25 (d) Q. 38 (b)

Q. 13 (a) Q. 26 (c) Q. 39 (d)

DO NOT OPEN THIS TEST BOOKLET UNTIL YOU ARE ASKED TO DO SO

1 TPT2019/B-1/012019/52 1. Correct option: (a) 2. Correct option: (c) Explanation: Explanation: Folk Dances presented during Republic Bagru Block Printing Day Parade 2019  Statement 1 is incorrect: It is a traditional  Option (a) is correct: technique of printing with natural colour Folk Dances Concerned State done by the Chippa community in Bagru village of Rajsthan. A. Karakattam 2. Tamil Nadu  Statement 2 is correct: The main colors B. Taakala 3. Maharashtra used in Bagru are Red and Black. C. Misra Raas 1. Gujarat  Statement 3 is correct: Traditionally, motifs printed at Bagru are large with bold Karakattam folk dance: Tamil Nadu lines. The motifs include wild fl owers,  It is an ancient folk dance of Tamil Nadu buds, leaves and printed geometrical performed in praise of the rain goddess patterns. Mariamman.  Some other important traditional block  The performers balance a pot on their printing techniques in India head.  Gujarat: Ajrakh Print  Traditionally, this dance is categorized  Rajasthan: Sanganeri, Ajrakh, Dabu into two types- Aatta Karakam is danced  with decorated pots on the head and Madhya Pradesh: Bagh Print, symbolizes joy and happiness. It is mainly Bherogarh Print (Batik) performed to entertain the audience.  Andhra Pradesh: Kalamkari  The Sakthi Karakam is performed only in  Calcutta, Serampur (West Bengal), temples as a spiritual off ering Varanasi and Farrukabad (Uttar Pradesh) are also important centres of Taakala Folk Dance: Maharashtra block printing in India.  It is related to the exchange of the ‘takla’  In News: The ‘Titanwala Museum’ in vegetable among local tribal groups. Bagru, Rajasthan was inaugurated to Misra Raas of Gujarat: preserve the art of traditional Bagru block printing.  Raas popularly known as dandiya raas is one of the most popular folk dances of Gujarat. 3. Correct option: (c)  Mishra raas/Gop raas is a variant of this Explanation: genre. It is performed by both men and Maithili Language women. It is not based on any criteria of caste or profession.  Statement 2 is correct: Mithilakshar or Tirhuta is the script of the language North Eastern state folk dances Maithili. presented GS SCORE  It is mainly spoken in India (Bihar,  Monpa of Arunachal Pradesh: It is a Jharkhand etc.) and Nepal. traditional dance form of the Monpa  tribe, who inhabit parts of West The oldest form of Mithilakshar is found Kameng District and Tawang District in the Sahodara stone inscriptions of in Arunachal Pradesh. This dance is 950 AD. performed during the Losar Festival,  Vidyapati Thakur was the fi rst writer which marks the tribe’s New Year. to use Maithili as a literary language, his poetic works and songs are full of  Mamita of Tripura: It is performed on devotion to the God Shiva. the occasion of Mamita festival, which is harvest festival of Tripuri People.  In the 14th century, Jyotishwar enriched the literature in a holistic way. His work  Satoiya Nitya of Assam Dhurt Samagam is very popular in  Tamang Seto of Sikkim Maithili.

2  In the 20th century, Baba Nagarjun,  Haridwar (banks of Ganga), Ramanand Renu contributed in the fi eld  Prayagraj/Allahabad (Confl uence of of prose writing. Ganga, Yamuna and Saraswati),  In 1910 the fi rst Maithili organization, the  Nashik (banks of Godavari) Maithili Mahasabha came into being for the development of Maithili and Maithils.  Ujjain (banks of Shipra).  Statement 1 is correct: The language has  Kumbh Mela can be traced back in been accorded a constitutional status in history through the writings of 7th the 8th Schedule of the constitution. century Chinese traveller Hiuen Tsang who came to India during the rule of  The lack of use of Mithilakshar script has Harshavardhan. The festival was also become the reason for the decline in the popularised among people by the eighth Maithli language. century saint Shankaracharya.  The Ministry of Human Resource  At Prayagraj, Kumbh Mela held at every Development has decided to implement 6 years and Maha Kumbh at every 12 the recommendations of a committee for years. Earlier they were known as Ardh the promotion and protection of Maithili Kumbh and Kumbh. But this year the language and its scripts. government has announced the Ardh  The committee has suggested establishing Kumbh will be known as Kumbh and a Script and Manuscript Centre at Kumbh as Maha Kumbh. Darbhanga and creating of audio-visual teaching material for mithilashkar script. 5. Correct option: (b) 4. Correct option: (c) Explanation: Explanation: Swadesh Darshan Scheme  Sanskriti Kumbh and Kumbh Mela Statement 1 is incorrect: Ministry of Tourism (MoT) launched the Scheme for  Statement 1 is correct: The Ministry of integrated development of theme based Culture had organized Sanskriti Kumbh, tourist circuits in the country in 2014-15 a 29 days cultural extravaganza at Kumbh  Mela area, Prayagraj, Uttar Pradesh. Objective is to develop theme-based tourist circuits in the country. These  It is being conducted with intent to tourist circuits will be developed on showcase the rich Cultural Heritage of the principles of high tourist value, India in all its rich and varied dimensions competitiveness and sustainability in an viz. Performing Arts, Folk, Tribal and integrated manner. Classical art forms, Handicrafts, Cuisines, Exhibitions etc. all in one place. Features of Swadesh Darshan Scheme:  Kumbh Mela is one of the oldest and  Statement 2 is correct: 100% centrally largest congregationsGS in the SCORE world funded for the project components which is attended by millions of people undertaken for public funding. irrespective of all caste, creed, sex and  To leverage the voluntary funding region. available for Corporate Social  Statement 2 is correct: The signifi cance Responsibility (CSR) initiatives of Central of Kumbh Mela has been enhanced at the Public Sector Undertakings and corporate Global level as the UNESCO has inscribed sector. Kumbh Mela on the list of Intangible  Funding of individual project will vary from Cultural heritage for humanity in state to state and will be fi nalised on the 2017. basis of detailed project reports prepared  The Kumbha Mela is held four times by PMC (Programme Management over the course of 12 years, in rotation Consultant). PMC will be a national between four Hindu pilgrimage places on level consultant to be appointed by the four sacred rivers: Mission Directorate.

3  A National Steering Committee (NSC)  The 125-year-old Old High Court will be constituted with Minister in Building in Nagpur, Maharashtra, charge of M/O Tourism as Chairman, to  Two Mughal-era monuments in Agra steer the mission objectives and vision of (U.P.): the scheme.  Haveli of Agha Khan  A Mission Directorate headed by the  Member Secretary, NSC as a nodal Hathi Khana offi cer will help in identifi cation of projects  The ancient Neemrana Baori in Alwar, in consultation with the States/ UTs Rajasthan governments and other stake holders.  The Group of Temples at Bolangir, Odisha 6. Correct option: (a)  The Vishnu Temple, Pithoragarh, Explanation: Uttarkhand Archeological Survey of India (ASI) 7. Correct option: (c)  The ASI under the Ministry of Culture Explanation: is the premier organization for the archaeological researches and protection ‘Adopt a Heritage’ project of the cultural heritage of the nation. Sl.No. Name of Monument State  Maintenance of ancient monuments and archaeological sites and remains of 1. Red Fort national importance is the prime concern 2. Gandikota Fort Andhra of the ASI. Pradesh  Besides it regulates all archaeological 3. Area surrounding Uttrakhand activities in the country as per the Gangotri Temple and provisions of the Ancient Monuments Trail to Gaumukh and Archaeological Sites and Remains 4. Mt. Stok Kangri Trek, Jammu and Act, 1958. Lakakh Kashmir  It also regulates Antiquities and Art 5. Jantar Mantar Delhi Treasure Act, 1972. 6. Surajkund Haryana  Statement 1 is correct: It was founded in 1861 by Alexander Cunningham who 7. Qutub Minar Delhi also became its fi rst Director-General. 8. Ajanta Caves Maharashtra  Statement 2 is incorrect: Under Section 9. Leh Palace, Leh Jammu & 4 of the Ancient Monuments and Kashmir Archaeological Sites and Remains Act, 1958, ancient monuments or 10. Hampi (Hazara Rama Karnataka archaeological sites which are of historical, Temple) archaeological GS or artistic interest SCORE and  Statement 1 is correct: It’s a joint which have been in existence for not less collaborative eff ort of The Ministry of than 100 years may be declared as of Tourism, Ministry of Culture, Archeological national importance. Survey of India (ASI) and State/UTs Governments to develop the heritage  The protection and maintenance of sites/ monuments and making them monuments, declared as of national tourist-friendly. importance is taken up by ASI by way of  structural repairs, chemical preservation Under the project private sector companies, and environmental development around public sector companies and individuals with best vision for the heritage site will the monument which is a regular and on- be selected through a bidding process going process. (Vision Bidding). Successful bidders will  ASI has declared 6 monuments as be tagged as Monument Mitra. monuments of national importance in  Statement 2 is correct: These ‘Monument 2018. The 6 monuments are as follows: Mitras’ are expected to use Corporate

4 Social Responsibility (CSR) funds for  He is a contemporary of Tulsidas, providing amenities like toilets, drinking Mirabai and Surdas. His poetry is bound water, accessibility for the disabled, to earth and revels in the universality of signage, audio guides etc. No funds are love and mystic experiences. given by Ministry of Tourism.  During his reign, Jean Baptiste Tavernier  As of now government has put up a list visited and wrote about walking into the of over 93 ASI monuments under this Qutb Shahi tombs complex where carpets project. were laid out and whoever walked in was served pulao.  Recently Ministry of Culture informed Parliament that ten monuments have been adopted under ‘Adopt a Heritage’ 9. Correct option: (c) (Apni dharohar Apni pehchan) project. Explanation: Konark Sun Temple 8. Correct option: (b)  Built in the 13th century, the Konark Explanation: temple was conceived as a gigantic ’s 17th century Badshahi chariot of the Sun God, with 12 pairs of Ashoorkhana: exquisitely ornamented wheels pulled by seven horses.  Statement 1 is correct: The 400-year old  Ashoorkhana was built by Muhammed The name Konark derives from the Quli Qutb Shah. combination of the Sanskrit words Kona (corner or angle) and Arka (the sun).  Statement 2 is incorrect: It was a house  Statement 1 is incorrect: It was built by of mourning, where large congregations King Narasimhadeva I, the great ruler of of Shia Muslims gathered in memory of Eastern Ganga dynasty. the martyrdom of Imam Hussain during the month of Muharram.  Statement 2 is correct: Also called the Surya Devalaya, it is a classic illustration  The building consists of a Kaman, Naqqar of the Odisha style of Hindu temple Khana (where ceremonial drums are architecture. beaten), Niyaz Khana (where visitors are fed), Sarai Khana (place to rest), Abdar  The temple is included in UNESCO World Khana (drinking water is stored), the Heritage Site in 1984 for its architectural chabutra (platform) and a guard room. greatness and also for the sophistication and abundance of sculptural work.  Statement 3 is correct: During the rule of the Mughal Emperor Aurangzeb, in 1178  The temple is perfect blend of Kalinga Hijri (about 1764 AD), the Ashoorkhana architecture, heritage, exotic beach and was converted into a bandikhana. For 80 salient natural beauty. years, it was used as a stable where horses  It is protected under the National were kept, before it was restored as a Framework of India by the Ancient place of worship by the then mutawalli Monuments and Archaeological Sites Mir Nawazish Ali Khan. and Remains (AMASR) Act (1958) and  The eight wooden pillars were its Rules (1959). reconstructed GS and the alams SCORE were  The Konark is the third link of Odisha’s reinstalled. During this time, the main Golden Triangle. The fi rst link is entrance, Bab-e-Faiz-e-Imam-e-Alamiyan, Jagannath Puri and the second link is was also constructed. (Capital city of Odisha). Fact File:  Statement 3 is correct: This temple was also known as ‘Black Pagoda’ due to Muhammad Quli Qutb Shah its dark color and used as a navigational  He was the fi fth sultan of the Qutb Shahi landmark by ancient sailors to Odisha. dynasty of Golkonda who ascended to Similarly, the Jagannath Temple in Puri the throne in 1580. was called the “White Pagoda”.  He founded the city of Hyderabad and  It remains a major pilgrimage site for built its architectural centerpiece, the Hindus, who gather here every year for Charminar. He also built Charkaman the Chandrabhaga Mela around the archways. month of February.

5 10. Correct option: (b)  The lower storey consists only of a series of basement cells. Explanation:  Untidy bastions occupy its four corners, Rani- ki- vav: imparting it the look of a fortifi ed  Statement 1 is correct: It is an intricately building. constructed stepwell situated in Patan,  The gateways of three sides, except on the Gujarat. west, are edged by narrowed minarets;  Statement 2 is incorrect: The stepwell is the main entrance is from the east. located on the banks of Saraswati River.  Corresponding with the openings of cells  Statement 3 is correct: It was built on the lower storey, the upper storey by Queen Udayamati of the Solanki contains holed windows (khirkis), which Dynasty in the 11th-century as a memorial have given it its present name. to her husband King Bhima I.  The pillared courtyard is divided into  Rani Ki Vav was built in the complex twenty - fi ve squares, fi ve on each side, ‘Maru-Gurjara’ architectural style with each square consisting of nine smaller an inverted temple and seven levels of squares. stairs and holds more than 500 principal sculptures.  The larger squares, together with the square in the centre of the courtyard, are  It highlights the sanctity of water as it is each covered with a cluster of nine small designed as an inverted temple under the low domes. earth’s surface.  Among the remaining squares, four on  The central theme is the Dasavataras, the diagonals are left uncovered to admit or ten incarnations of Vishnu, including Buddha. The avatars are accompanied by light, while the others are covered with sadhus, Brahmins, and apsaras (celestial fl at roofs. dancers).  Statement 3 is correct: This ingenious  It was added to the list of UNESCO’s way of covering the courtyard is repeated World Heritage Sites on June 22, 2014. only in one other mosque erected by the same builder. These two are the only  The site bagged the title of the ‘Cleanest examples of closed mosques in northern Iconic Place’ in India at the Indian India. Sanitation Conference 2016.  The Reserve Bank of India (RBI) had 12. Correct option: (c) released a new lavender Rs. 100 currency note. The banknote highlights the rich Explanation: and diverse cultural heritage of India as it prominently displays a photograph of Kalamkari Art ‘Rani-ki-vav’ (The Queen’s Stepwell), an  Kalamkari literally translates into “pen 11th century architectural wonder. craft”; with ‘kalam’ meaning pen and ‘kari’ meaning art. It is among the most 11. Correct option: (d) beautiful traditional Indian art forms GS SCOREand involves block printing or hand Explanation: printing, typically done on pieces of Khirki Mosque cotton fabrics.   This mosque lies on the southern periphery Statement 1 is incorrect: It depicts of the village Khirki, New Delhi. deities and scenes taken from the great Hindu epics- Ramayana, Mahabharata,  Statement 2 is correct: The mosque was Puranas and mythological classics. built by Khan-i-Jahan Junan Shah, the Prime Minister of Firoz Shah Tughluq  Statement 3 is correct: The unique (1351-88) and is believed to be one of feature of the Kalamkari art is that it the seven mosques built by him. makes use of only natural colours or vegetable dyes.  Statement 1 is correct: Built with rubble stone, the Khirki Mosque is double-  Statement 2 is correct: Kalamkari art storeyed. and printing is concentrated primarily

6 in Andhra Pradesh, particularly in colorful shadows for the viewers who sit Kalahasti and Machilipatnam, and a in front of the screen. This tradition of few other smaller regions of the state. shadow puppets survives in Odisha Kerala, Andhra Pradesh, Karnataka, Maharashtra  Srikalahasti was the main center of and Tamil Nadu. kalamkari art for a very important reason: it received a constant supply of clean river  Togalu Gombeyatta, Karnataka water.  These puppets are mostly small in  There are two identifi able styles of size. The puppets however diff er in Kalamkari art in India: size according to their social status, for instance, large size for kings and  In the Machilipatnam style of religious characters and smaller size Kalamkari, motifs are essentially for common people or servants. printed with hand-carved traditional blocks with intricate detailing painted  Tholu Bommalata, Andhra Pradesh by hands.  The puppets are large in size and  On the other hand, Srikalahasti style have jointed waist, shoulders, elbows of painting draws inspiration from the and knees. They are colored on both Hindu mythology describing scenes sides. Hence, these puppets throw from the epics and folklore. This colored shadows on the screen. The style holds a strong religious connect music is dominantly infl uenced by the because of its origin in the temples. classical music of the region and the theme of the puppet plays are drawn  The Srikalahasti style is characterized from the Ramayana, Mahabharata and by one very important feature: free Puranas. hand drawing.  Ravanachhaya, Odisha  The procedure begins with the craftsman treating the cotton cloth  The puppets are in one piece and have using mordant and sketching the no joints. Hence the manipulation design outline with black colour or requires great dexterity. They are not ‘kasami’. colored, hence throw opaque shadows on the screen. The puppets are made  The only other colours used for of deer skin and are conceived in bold fi lling the outlined sketch are those dramatic poses. Apart from human obtained from natural plants: indigo, and animal characters, many props green, red and vibrant yellow. such as trees, mountains, chariots, etc.  Therefore, every piece of the are also used. Although, Ravanachhaya kalahasti kalamkari art is quite puppets are smaller in size-the largest unique and absolutely authentic. not more than two feet have no jointed  A ‘Kalamkari art museum’ has been limbs, they create very sensitive and opened at Pedana in Andhra Pradesh. lyrical shadows. The museum traces the history of the art and displays theGS process of extraction SCORE of 14. Correct option: (a) natural colours from various sources. Explanation: 13. Correct option: (a) Mithila Painting  Explanation: Mithila painting is folk painting practiced in Mithila region of India (especially Shadow Puppets Bihar) and Nepal.  Option (a) is correct: Shadow puppets  It is also known as Madhubani paintings are fl at fi gures, cut out of leather, which which mean ‘forest of honey’. has been treated to make it translucent.  Shadow puppets are pressed against It has been mentioned in ancient Indian the screen with a strong source of light texts like Ramayana. behind it. The manipulation between the  Statement 1 is correct: It originated in light and the screen make silhouettes or small village known as Maithili in Bihar.

7  Initially, women from village drew paintings  Statement 3 is correct: The sports season on freshly plastered mud walls of their generally starts in November and lasts home as illustration of their thoughts, till March. hopes and dreams using fi ngers and  twigs forming two dimensional imagery The contest generally takes place between using paste of powdered rice. two pairs of buff aloes, each pair race in two separate wet rice fi elds’ tracks,  With time, it became part of festivities controlled by a whip-lashing farmer. and special events like marriage. It also used colors derived from vegetables and  In the traditional form of Kambala, buff alo plants. racing was non-competitive and buff alo pairs ran one by one in paddy fi elds.  Now they are also painted on cloth, handmade paper and canvas.  Besides, there was also ritualistic approach  Statement 3 is incorrect: The central as some agriculturist’s race their buff aloes themes of Mithila paintings are based as a means to thank god for protecting on Hindu Gods and Goddesses. their animals from disease.  Its main theme is supported by traditional  But in recent times, Kambala has become geometric patterns. an organised rural sport.  Some of main attributes of these paintings are double line border, abstract-like 16. Correct option: (d) fi gures of deities, ornate fl oral patterns, bold use of colors and bulging eyes and Explanation: jolting nose of faces of fi gures. Walmiki and Malhar Languages  Statement 2 is correct: The various  A linguist from the University of styles of Mithila painting include Bharni, Hyderabad (UoH) has stumbled on two Tantrik, Katchni, Godna, and Kohbar, languages called Walmiki and Malhar which were historically painted only by both predominantly in the remote regions women from the upper strata in caste of Odisha. system, which used to make them on mud walls on special occasions.  They were discovered by Prof Panchanan  Mithila painting has been accorded the Mohanty as part of activity of Centre for coveted GI (geographical Indication) Endangered Languages and Mother status as it has remained confi ned to Tongue Studies. compact geographical area and skills  The professor had collected some data have been passed on through centuries, and did preliminary analysis of these but content and the style have largely languages and published paper in remained same. proceedings of XX Annual Conference of  Madhubani railway station got complete Foundation for Endangered Languages, makeover with Mithila paintings. The UK. makeover project was unique initiative of Indian Railways to experiment with local  Statement 1 is correct:The languages artists voluntarilyGS painting works in SCOREshort are categorised `endangered’ as the span of two months. number of people speaking is small. These languages are spoken in Andhra 15. Correct option: (b) Pradesh and Odisha.  Only a few people living in the forest and Explanation: hilly areas speak these languages. Kambala Walmiki:  Statement 1 is correct and Statement 2  is incorrect:: Kambala is traditional slush Statement 3 is correct It is spoken in track buff alo race that is held annually district of Koraput of Odisha and on in coastal districts of Karnataka to bordering districts of Andhra Pradesh. entertain rural people of the area.  It is an isolate language and does  Slushy/marshy paddy fi eld track is used not belonged to particular family of for Kambala. languages.

8  Its name is interesting and indicative as  The men dance by holding the sword in speech of the community claims to be their right hand and whisks in their left. descent from great Indian saint-poet The sword is used to symbolize a man’s Valmiki, who is credited to have written defense of himself and his family. Men wear one version of epic Ramayana. dhotis, full sleeved shirts, embroidered sleeveless coats and turbans. Malhar:  Prayers are off ered to Ka Pah Syntiew  It is spoken in a remote and isolated hamlet and U Suid Nia Tong Syiem. The prayer about 165 km away from Bhubaneswar, is off ered for good harvest, peace and Odisha. prosperity for all the community people.  Now it is spoken by community consisting of about 75 speakers including children. 18. Correct option: (c)  Statement 2 is correct: It belongs to Explanation: “North Dravidian” subgroup of Dravidian family of languages. Ambubachi Mela:  It has close affi nities with other North  Statement 2 is correct: It symbolises the Dravidian languages like Malto and Kurux fertility cult of goddess Kamakhya. spoken in Jharkhand, West Bengal and  There is no idol in the temple; the goddess Bihar. is worshipped in the form of a yoni-like stone over which a natural spring fl ows. 17. Correct option: (c)  Also known as Siddha Kubjika, the Explanation: goddess is a Hindu tantric goddess of desire who evolved in the Himalayan hills. Nongkrem Dance Festival She is also identifi ed as Kali and Maha  Statement 1 is correct: Nongkrem Dance Tripura. Festival (held annually in November) is a  Statement 1 is correct: The festival is fi ve day festival which is celebrated for the also known as ‘Mahakumbh of the East’ harvest thanksgiving by the Khasi tribe. as it draws lakhs of devotees from all over  Statement 2 is correct: The Nongkrem the world. Dance is the most important festival of Kamakhya Temple: the Khyrim state.  Kamakhya, atop Nilachal Hills in  The festival is associated mostly with Guwahati, is one of 51 shaktipeeths or goat sacrifi ce and is held at Smit, which seat of Shakti followers, each representing is around 15 kilometres to the southwest a body part of the Sati, Lord Shiva’s of Shillong. companion.  The Nongkrem word means “Goat  The temple’s sanctum sanctorum Killing Ceremony”. The word Nongkrem (garbhgriha) houses the yoni — female was used because in the past years, the genital — symbolized by a rock. tribes used to celebrate this festival at  Nongkrem, whichGS is the headquarters SCORE of Kamakhya Temple denotes the spot where Khyrim state. Sati used to retire in secret to satisfy her amour with Shiva, and it was also the place  Drums and Pipes are played continuously where her yoni fell after Shiva danced with during this festival to mark the occasion. the corpse of Sati.  The traditional dance is performed by  The only ones that avoid the temple dancers who are the young men and are the descendants of the medieval women of the tribe. The youngsters get Koch royalty, who had reconstructed dressed up in exquisite attire. The silver Kamakhya temple in 1565. or gold crowns worn by the tribes on their  This is because the goddess is believed heads signify the glory and dignity of the to have cursed the royalty after the king Khasi society. and his brother Chilarai — one of Assam’s  The dance performed by the man is known revered generals — had secretly watched as “Ka Shad Mastieh”. her dance.

9  There are legends about the goddess  The fi rst conference was held from dancing when Kendukoli, a priest during 10 to 12 January 1975 in Nagpur, Naranarayan’s reign, performed with Maharashtra and was inaugurated by his eyes shut. then Prime Minister Indira Gandhi. Since  Naranarayan and Chilarai convinced then, ten such Conferences have been the priest to allow them to watch the held in diff erent parts of world. goddess’s dance.  In the fi rst conference, then Prime Minister  Incensed, the goddess punished the of Mauritius Seewoosagur Ramgoolam priest besides cursing the duo and their was Chief Guest and it was attended by descendants with doom if they visited the 122 delegates from 30 countries. Kamakhya temple ever.  In remembrance of the fi rst conference, Tuloni Biya: every year 10th January is being observed as World Hindi Day.  The ritualistic fair celebrating the goddess’ period is one of the reasons why the taboo  The 10 edition of World Hindi Conference associated with menstruation is less in was held in , Madhya Pradesh Assam compared to other parts of India. (India) in 2015 with the theme of “Hindi  The attainment of womanhood of girls in Jagat-Vistar and Sambhavnaye”. Assam is celebrated with a ritual called ‘Tuloni Biya’, meaning small wedding. 20. Correct option: (c)  Ambubachi Mela is also an occasion Explanation: to promote awareness on menstrual hygiene. Guru Padmasambhava  An International Conference on 8th century 19. Correct option: (c) Himalayan sage Guru Padmasambhava Explanation: was held in New Delhi.  11th World Hindi Conference (WHC) The conference was organised as part of events to commemorate 50-years of  Statement 2 is correct: The 11th formalization of diplomatic ties between World Hindi Conference (WHC) was India and Bhutan. organized in August 2018 in Mauritius  by the Ministry of External Aff airs, Guru Padmasambhava was born in India Government of India in association with and travelled all across Himalayan region the Government of Mauritius. in the 8th century to spread Buddhism and Buddhist teachings.  Statement 3 is correct: The theme of the Conference was “Vaishvik Hindi Aur  Guru Padmasambhava is highly revered in Bharatiy Sanskriti“. Bhutan.  The conference was aimed at providing  Statement 2 is correct: He was also common platform to several Hindi known as Guru Rinpoche is widely scholars, writers and laureates from venerated as a “second Buddha” by diff erent parts of the world to contribute adherents of Tibetan Buddhism in Tibet, the language. GS SCORE Nepal, Bhutan, the Himalayan states of  Its venue was at Swami Vivekanand India, and elsewhere. International Convention Centre,  Statement 1 is correct: Guru Pailles Mauritius. Padmasambhava is also considered to  The conference covered both classical be the founder of Nyingma tradition, and modern elements of Hindi and also oldest of the four major schools of Tibetan saw participation delegates and scholars Buddhism. of Hindi from all over the world. World Hindi Conference: 21. Correct option: (b)  Statement 1 is incorrect: The World Explanation Hindi Conference is major event organised every three years by MEA to increase the Stupa Architecture Design and its Related popularity of Hindi globally. Terms

10  A circular terrace (medhi). The terrace—  This style of architecture also came to be surrounded by a similar three-bar railing— known as Sharqi style and also avoided supports the anda and raises it off the the use of minars, as the Pathan style. ground (black highlights); it likely served  as a platform for ritual circumambulation. A unique feature of the buildings here is the use of bold and forceful characters  Anda -large hemispherical dome where painted on huge screens in the centre and a casket containing buddha relics(dhatu) side bays of the prayer hall. was placed in small central chamber  Inner wall – unburnt brick, outer wall – 24. Correct option: (a) burnt brick Explanation  Harmika – square pavilion on the fl attened summit of the dome Kangra School  Chatra – umbrella built on the top of the  After the decline of the Mughal empire, dome many artists trained in the Mughal style  Pradakshinapatha – path for clockwise migrated to the Kangra region as they got circumambulation surrounded by a fence patronage by Raja Govardhan Singh in built encircling the stupa 1774. It led to the birth of Guler- Kangra School of paintings. It fi rst evolved in  Torana – gateways having sculptures depicting Jataka tale Guler then came to Kangra. This school reached its zenith under the patronage of Raja Sansar Chand. His paintings were 22. Correct option: (a) marked with a sensuality and intelligence Explanation that the other schools lack.  Paitkar Painting The popular subjects were the Gita Govinda, Bhagwata Purana, Satsai of  Practiced by the tribal people of Jharkhand, Biharilal and Nal Damyanti. Love scenes Paitkar paintings or scroll paintings are of Krishna was very prominent theme. All considered one of the ancient schools of the paintings had another worldly feel painting in the country. This old form of about them. Another very famous group painting has cultural association with Ma Mansa, one of the most popular of paintings is the ‘Twelve months’ where goddesses in tribal household. the artist tried to bring forth the eff ect of the twelve months on the emotions  These paintings are linked to the social and of human beings. This emotive style was religious customs including giving alms popular till the 19th century. and holding yajnas. The common theme of Paitkar paintings is ‘What happens to human life post death’. Although an 25. Correct Option: (d) ancient art, it is on the verge of extinction Explanation given the rate of its decline. Benefi ts of the World Heritage Site Manjusha Painting Status  This art form belongsGS to Bhagalpur regionSCORE  of Bihar. It is also known as Angika art, Brings international attention to the need where ‘ang’ refers to one of the Mahajan for the preservation and conservation of Pada. Since snake motifs are always the site. present, it is also called snake painting.  Brings tourism to the site ensuring economic benefi ts. 23. Correct option: (c)  UNESCO can provide funds for restoration, Explanation preservation, and training, if required. Jaunpur School (Sharqi Style of  Promotes close ties with the United Architecture) Nations system and the prestige and support it provides.  Patronised by the Sharqi rulers, Jaunpur became a centre of great art and cultural  Enables access to global project activity. management resources.

11  Facilitates creating partnerships between Pradesh. Generally staged during village government, the private sector, and NGOs fairs and festivals, the performance occurs to achieve conservation goals. through the night and comprises of a  Site becomes protected under the Geneva series of small plays and skits. Convention against destruction or misuse  Maach is the folk theatre of the region of during wartime. Malwa in Madhya Pradesh. It originated in Ujjain around 17th century A.D. and 26. Correct Option: (c) was based on mythological themes. Later, romantic folk tales were included into Explanation its repertoire. The unique feature of this form is the dialogues, which are delivered Thumri in the form of couplets known as Rangat  It is based on mixed ragas and is Dohas. commonly considered to be semi classical Indian music. 28. Correct Option: (d)  The compositions are either romantic or Explanation devotional in nature.  The following criteria were laid down to  This was inspired by the Bhakti movement determine the eligibility of languages so much that the text usually revolves to be considered for classifi cation as a around the girl’s love for Krishna. “Classical Language”.  The language of the composition is  High antiquity of its early texts/ usually Hindi or Awadhi dialect or the Braj recorded history over a period of Bhasha dialect. 1500–2000 years;  The main Gharanas of thumri are based  A body of ancient literature/texts, in Banaras and Lucknow and one of the which is considered a valuable heritage most timeless voice of singing thumri is of by generations of speakers; Begum Akhtar who is a household name for her husky voice and boundless range  The literary tradition be original and in singing. not borrowed from another speech community; 27. Correct Option: (c)  The classical language and literature being distinct from modern, there may Explanation also be a discontinuity between the Folk Theatre Regions of Popu classical language and its later forms larity or its off shoots. (a) Bhavai Gujarat and  Languages so far declared to be Classical Rajasthan language are: (b) Daskathia Odisha  Tamil in the year 2004 (c) Kariyila Himachal Pradesh  Telugu in the year 2008 (d) Maach GS Malwa SCORE Malayalam in the year 2013  Bhavai is a popular folk theatre form of  Sanskrit in the year 2005 Gujarat and Rajasthan, mainly in the  Kannada in the year 2008 regions of Kutchh and Kathiawar. This form incorporates an extensive use of  Odia in the year 2014 dance to narrate a series of small plays, known as Vesha or Swanga, each with 29. Correct Option: (d) its own plot. The theme of the play is generally romantic. Explanation   Daskathia is a form of folk theatre popular The Portuguese brought with them the in the region of Odisha. Iberian style of architecture. They initially built trading posts and ware houses,  Kariyila is another form of open-air theatre which were later remodelled into fortifi ed that is popular in the foothills of Himachal town along the coastlines

12  The French brought the concept of urban  Ramanuja, the main proponent of city planning with them. The French Vishishtadvaita philosophy contends towns of Pondicherry and Chandernagore that the Prasthanatrayi (“The three (now Chandannagar, West Bengal) were courses”), namely the Upanishads, the built using the Cartesian grid plans and Bhagavad Gita, and the Brahma Sutras are scientifi c architectural designs. to be interpreted in a way that shows this unity in diversity, for any other way would  The British brought with them the Gothic violate their consistency. style of architecture. It merged with the Indian architecture and resulted in the  There are three key principles of Indo-Gothic style of architecture. Post- Vishishtadvaita: 1911, a new style of architecture known as  Tattva: The knowledge of the 3 real the Neo-Roman architecture emerged. entities namely, jiva (living souls; the sentient); ajiva (the nonsentient) 30. Correct Option: (a) and Ishvara (Vishnu-Narayana or Parahbrahman, creator and controller Explanation of the world).  Modhera Sun Temple Gujarat  Hita: The means of realization, as  Konark Temple Odisha through bhakti (devotion) and prapatti (self-surrender).  Dakshinaark Temple Bihar  Purushartha: The goal to be  Martand Sun Temple Kashmir attained, as moksha or liberation from bondage. 31. Correct option: (a) Explanation: 32. Correct option: (a) Vishishtadvaita (Qualifi ed Non-dualism) Explanation:  Statement 3 is incorrect: Its proponent Kuthiyottam Ritual and Pongala Festival was Rāmānuja.  Option (a) is correct: It is usually  The basic theory is that “jīvātman is a performed every year during the Pongala part of Brahman, and hence is similar, festival at the Attukal Bhagavathy but not identical. Brahman, matter and the Temple in Thiruvananthapuram, individual souls are distinct but mutually Kerala. The temple is popularly known as inseparable entities”. the “Women Sabarimala”.   Statement 1 is correct: Vishishtadvaita According to Kerala’s Department of advocates Bhakti to attain God. Tourism, the Attukal Pongala festival is the largest congregation of women for a  It is one of the most popular schools of festival in the world. the Vedanta school of Hindu philosophy.  Pongala, which means ‘to boil over’, is a  Vedanta literally means the end of the ritual in which women prepare a pudding . made from rice, jaggery, coconut and  GS SCOREplantains cooked together, and off er it Statement 2 is correct: VishishtAdvaita (literally “Advaita with uniqueness; to the ‘Bhagavathy’, also referred to as qualifi cations”) is a non-dualistic school ‘Attukalamma’ goddess. of Vedanta philosophy.  The ritual can only be performed by  It is non-dualism of the qualifi ed whole, women. in which Brahman alone exists, but is  The Attukal Pongala Mahotsavam is a 10 characterized by multiplicity. days festival which falls during Malayalam  It can be described as qualifi ed monism month of Kumbham. or qualifi ed non-dualism or attributive  In 2009, the Attukal Pongala festival monism. entered the Guinness Book of Records as the largest gathering of women.  It is a school of Vedanta philosophy which believes in all diversity subsuming to an  Nearly 1,000 young boys undertake a underlying unity. seven-day penance before Pongala day.

13  These boys are said to represent the  It does so by re-enacting the wedding wounded soldiers of the goddess. of Lord Sundareswarar (Lord Shiva)  The boys have to observe strict discipline and Goddess Meenakshi (Lord Vishnu’s and stay inside the temple for seven sister). days.  The festival includes a procession of  The rigors include sleeping on the fl oor, Kallazhagar avtar of Lord Vishnu strict diet restrictions, and bathing three entering Vaigai River. times a day.  He travels from his abode in Azhagarmalai  They also have to prostrate 1,008 times to get his sister married to Lord before the deity. Sundareshwarar.   The ritual reportedly involves piercing This journey from his abode to his sister’s the child’s side with a small hook and wedding, on a golden chariot, is the knotting a thread through it to symbolize legend behind celebrating the Chithirai their bond with the Goddess. Festival. Sri Meenakshi Sundareswarar Temple 33. Correct option: (a)  It is historic Hindu temple located on Explanation: southern bank of Vaigai River in temple city of Madurai, Tamil Nadu. Mahamastakabhisheka:  It is dedicated to Meenakshi, a form of  Statement 1 is correct: Parvati, and her consort, Sundareswar, a Mahamasthakabhisheka is the head form of Shiva. anointing ceremony which is performed once in 12 years to the 57 feet tall  It was built during reign of Pandyas. monolithic statue of Lord Bahubali  It houses 14 gopurams (gateway towers), (Gomateshwara) at Shravanbelagola, ranging from 45–50m in height and tallest Karnataka. is southern tower, 51.9 metres high.  It is one of biggest festivals for Jain  It also has two golden sculptured vimanas, community and it honors Lord Bahubali, shrines over garbhagrihas (sanctums) of a revered Jain icon. main deities.  According to Jain texts, Bahubali had  The Sri Meenakshi Sundareswarar Temple attained liberation from cycle of births and deaths (moksha) at Mount Kailash in Madurai (Tamil Nadu) was adjudged and is revered as a liberated soul (Siddha) as the cleanest iconic place in country by Jains. under Swachhta Hi Seva (cleanliness is service) programme in 2017.  Bahubali is also called Gommateshwara because of Gommateshwara statue dedicated to him and as lord 35. Correct option: (b) Kammateswara from an inscription. Explanation:  Statement 2 is incorrect: Gommateshwara Chardham Yatra statue was builtGS by Ganga dynastySCORE minister and commander Chavundaraya  As per Hindu traditions, Char Dham Yatra in around 981 AD. opens the gates of salvation by washing away all the sins. 34. Correct option: (c)  The term Chardham is believed to be coined by Shri Adi Shankaracharya, Explanation: which means four abodes of God or Holy Chithirai Festival Destinations.  Both the statements are correct: The  Statement 1 is incorrect: The Yatra Chithirai festival is celebrated every year begins from Yamunotri, then the pilgrims at the Meenakshi Temple in Madurai, head to Gangotri, thereafter to Kedarnath Tamil Nadu during the month of April for and fi nally to Badrinath to complete the symbolizing the unity of saivitism and Chardham Yatra. Yamunotri is the source vaishnavism. of the sacred Yamuna River.

14  According to Hindu Mythology, Yamuna festival: Payesh (Kheer) made from the was the sister of Yamraj and she was newly harvested rice and Nabanna fair. promised by Yamraj that whoever takes  Nuakhai: It is an age old harvest the holy dip in the Yamuna River will not celebration in Odisha. Locally ‘nua’ be taken to Yamlok and they would attain means new and ‘khai’ means food. This salvation. is not only a popular harvest festival in  Yamunotri is also the seat of Goddess India, but also celebrated to appreciate Yamuna. Gangotri i.e. Gomukh is the birth the passing away of the past and evil days place of the Holy River Ganga. while welcoming the new and beautiful with open arms. The festival is also known  The stone on which King Bhagirath did as Nuakhai Parab or Nuakhai Bhetghat. meditation to bring Goddess Ganga to Key attractions of Nuakhai festival: The earth is located near Ganga Temple in delicious Arsaa Pitha (sweet pancakes). Gangotri.  Bhogali Bihu: Every year in January,  A temple dedicated to Goddess Ganga the entire state of Assam showcases is located in this pious area. Kedarnath enthusiasm and delight in celebrating is one of the twelve jyotirlingas of Lord Bhogali Bihu. The farmers of Assam Shiva and the temple is believed to be celebrate and cherish the eff orts of restored by Adi Shankaracharya. cultivation and reap the benefi ts. The  Badrinath is considered as the most sacred celebration starts one night before with among the four shrines and is located on Uruka—the community feast. On the the left bank of the Alaknanda River. day of Bihu, the mejis or pavilion made  The temple at Badrinath is dedicated to of clay and hay are burnt. Local women Lord Vishnu, the protector and preserver wear stunning mukhlas and participate of the divine Hindu Trinity (Brahma, in group songs and dance. Also known Vishnu, and Shiva). as Magh Bihu, this is an exotic and most vibrant name on the list of harvest festivals  It is also believed that Adi Shankaracharya of India. Key attractions of Bihu festival: got freedom from the process of Bihu dance, bullfi ght, bird fi ght and Sunga reincarnation at Badrinath. Pitha, Til Pitha and Laru.  Statement 2 is correct: The Chardham Development Plan is the project of 37. Correct option: (b) the Ministry of Road Transport and Highways. Explanation: Pakke Paga Hornbill Festival (PPHF) 36. Correct option: (a)  Option (b) is correct: The government Explanation: of Arunachal Pradesh has declared the Pakke Paga Hornbill Festival (PPHF)–the Harvest Festivals of India state’s only conservation festival, as a  Option (a) is correct: “state festival”. Harvest Festivals Concerned State  The fi rst-ever PPHF was held on January GS SCORE16–18, 2015. A. Nuakhai 3. Odisha  The organizers had a number of objectives B. Nabanna 1. West Bengal in mind- C. Bhogali Bihu 2. Assam  to recognise the role played by the  Nabanna: This is one of the most resident Nyishi tribe in conserving celebrated traditions of Bengal, where hornbills in the Pakke Tiger Reserve new rice is harvested with sheer joy and (PTR), stocked in homes. Farmers from Bengal  to devise alternative sources of income cheerfully participate in this harvest ritual for a region that relies on hunting and in the Bengali month of Agrahayan and logging, and off er the fi rst grains to Goddess  while thanking her for all blessings. It is to tell the rest of the country about the celebrated in the months of November and wonders of the PTR and its surrounding December. Key attractions of Nabanna areas.

15  As part of the festival, visitors are treated  It accepts, respects, and uses of to cultural programmes, food stalls, plays, scriptures of the world, (not as infallible) sightseeing, river and village walks, bird- as ancient records of the moral and watching, screenings of short fi lms as well spiritual experiences. as local tribal sports and dances.  It is involved in various educational, social,  Pakke Tiger Reserve medical and other welfare activities.  Pakke Tiger Reserve, also known as Pakhui Tiger Reserve, is a Project Tiger 39. Correct option: (d) reserve in the East Kameng district of Arunachal Pradesh in northeastern Explanation: India. Sangeet Natak Akademi (SNA)  Previously it was declared a Wildlife  Statement 2 is correct Sangeet Natak sanctuary in 2001, and was declared a Akademi (SNA) launched the second Tiger Reserve in 2002. phase of the Web Campaign ‘SANJHI-  This Tiger Reserve has won India MUJH MEIN KALAKAR’. Biodiversity Award 2016 in the category  Statement 3 is correct It is an initiative to of ‘Conservation of threatened document and promote the Intangible species’ for its Hornbill Nest Adoption Cultural Heritage (ICH) and diverse Programme. cultural traditions of the country by direct public-participation. 38. Correct option: (b)  The literal meaning of the term SANJHI is Explanation: “to share” and “to partner”. Sadharna Bhramo Samaj (SBS)  The fi rst phase of this campaign  It is a general community of worshipers was earlier initiated in the month of of one God, formed in May 1878, traces November 2018 during the festival back its roots to The Brahmo Samaj. season of .  Statement 1 is incorrect: It was formed  The Second Phase: by the followers of Keshav Chandra Sen  (Brahmo Samaj India) after he violated The second phase of this campaign the Brahmo Marriage Act by marrying has its main focus area like the folk, his 13 year old daughter with Maharaja of traditional, customary, socially events Cooch-Bihar. and ritualistic art forms – woven as  a cultural fabric around the harvest Anandamohan Bose was its fi rst festival season of January. President.  It is widely celebrated as various  In 1891, it opened the Das Ashram, a nomenclatures like , welfare institution of untouchables, and Pongal, Lohri, Bhogali Bihu, Torgya, the Brahmo Girls School of Calcutta, and Uttarayan, Attukkal Pongal, etc. also founded small hospitals, orphanages, a leper asylum.GS SCORE It intends to bring forth such forms that are aligned with the domains of intangible  Statement 2 is correct: Its basic principles are: cultural heritage as per the Convention of Safeguarding the ICH under UNESCO:  It believes in the existence and  personality of God, Oral traditions and expressions, including language as a vehicle of the  It believes in the immortality of the intangible cultural heritage; human soul.  Performing arts;  It does not believe in any particular  book or collections of books as the Social practices, rituals and festival one infallible revelation of divine truth, events; love, and fi nal authority.  Knowledge and practices concerning  It does not believe in specifi c nature and the universe; incarnation the Deity.  Traditional craftsmanship.

16 40. Correct option: (a) 1959 Ramdhari Singh Dinkar Hindi Explanation: 1960 R.K Narayan English PRASAD Scheme 1968 Harivansh Rai Bachchan Hindi  It was launched in 2014-15 by Union Ministry of Tourism. 1956 Amrita Pritam Punjabi  It aims at integrated development of 1988 Vikram Seth English pilgrimage destinations in planned, 1975 Kaifi Azmi Urdu prioritised and sustainable manner to provide complete religious tourism 2004 Ghulam Nabi Firaq Kashmiri experience. 1967 Buddhadeb Basu Bengali  It focuses on the development and 2012 K. Satchidanandan Malayalam beautifi cation of the identifi ed pilgrimage destinations.  This is an honour given to those who achieve brilliance in literature. This award Objectives: was instituted in 1954 and is given by the  Harness pilgrimage tourism for its direct ‘Sahitya Akademi’ that is the National and multiplier eff ect upon employment Academy of Letters of our country. generation and economic development.  Enhance tourist attractiveness in 42. Correct option: (d) sustainable manner by developing world class infrastructure in the religious Explanation destinations. Vijaynagar School of Temple  It also seeks to promote local art, culture, Architecture handicraft, cuisine, etc. The features of the temples were: Funding:  The walls of the temples were highly  Under it, Ministry of Tourism provides decorated with carvings and geometrical Central Financial Assistance (CFA) patterns. to State Governments for promoting tourism at identifi ed destinations.  Goupurams, which were previously  For components within public funding present on the front side, were now built under this scheme, Central Government on all the sides. will provide 100% fund.  Monolithic rock pillars  For improved sustainability of project,  Generally, temple pillars have a mythical it also seeks to involve Public Private creature Yali (Horse) engraved in them Partnership (PPP) and Corporate Social Responsibility (CSR) as well.  The enclosing walls were larger  Union Government has included Gangotri  More than one mandaps were built in and Yamunotri in Uttrarakhand, each temple. Amarkantak GSin Madhya PradeshSCORE  The central mandap came to be known as and Parasnath in Jharkhand under Pilgrimage Rejuvenation and Spiritual, kalyan mandap. Heritage Augmentation Drive (PRASAD)  The concept of secular buildings inside to develop pilgrimage and heritage the temple premises was also introduced destinations in the country. With the during this period. new additions, the number of sites under PRASAD has now reached to 41 in 25  Temple complex is enclosed by states. boundaries  Example: Vittalsami temple, Lotus Mahal, 41. Correct Option: (d) Virupaksha temple in Hampi, Raghunatha Temple in Hampi etc. Rock-cut idol of Explanation Narasimha on Shesha (snake) near Hampi Sahitya Akademi Award is a marvel in itself.

17 43. Correct Option: (b) He was inspired by Kabir Das and he was the proponent of Nirguna School. Explanation He emphasized “Nam Japna”, that is Some of the features of Kuchipudi dance repeated utterances of God’s name to are: feel his presence. However, it has to be  noted that Sikhism is not an extension of It involves diffi cult foot movements, and Bhakti movement. He also criticized the is generally a team performance. discriminatory caste practices and wished  Most of the Kuchipudi recitals are based for equality of all. Everybody irrespective on stories of Bhagwata purana but have a of their caste or creed was welcome to secular theme. There is a predominance come and have meals in the community of Shringaara ras. kitchens (langars). His teachings can be found in Guru Granth Sahib, the revered  The dance involves all three components Sikh scripture. of classical dances: Nritta, Nritya and Natya. It is similar to Bharatnatyam but Chaitanya Mahaprabhu (1486-1534) has its own features.  He was an ardent Krishna devotee  The Kuchipudi dance style is a and popularized the chanting of “Hare manifestation of the earthly elements in Rama, Hare Krishna”. He made famous the human body. the practice of worshipping Radha and Krishna together. He gave the  Both Lasya and Tandava elements are Achintaya Bheda-Abheda philosophy. important in the Kuchipudi dance form. He was a Saguna and popularized “Kirtans”  A Kuchipudi recital is generally (religious songs) as a form of worshipping accompanied with Carnatic music; Violin God. He was very popular in Eastern India and Mridgangam being the principal and lived most of his life in Puri, Odisha. instruments. The recital is in Telugu He wrote “Siksastakam”, a text in Sanskrit, language. where he elaborated his philosophy.

44. Correct Option: (c) 46. Correct option: (d) Explanation Explanation  This is the oldest school of philosophy Bhakti Movement and was founded by Kapil Muni who is  supposed to have written the Samkhya The Bhakti movement originated in the Sutra. The word ‘Samkhya’ or ‘Sankhya’ Southern parts of India, especially Tamil literally means ‘count’. Nadu between 7th and 12th century; it slowly percolated to the northern belt by  Samkhya schools argued that salvation the end of 15th century. could be attained through the acquisition  of knowledge In South India, there were two main groups of Bhakti saints: Nayanars (Shiva  This school believed in dualism or devotees) and Alvars (Vishnu Devotees) dvaitavada, i.e. the soul and the matter who preached devotion to God as a are separate entities. This concept is means of salvation and disregarded the the basis of GS all real knowledge. SCORE This austerities preached by Buddhism and knowledge can be acquired through three Jainism. Most of their poetries were main concepts: focused on the love between the devotee  Pratyaksha: Perception and God.   Anumana: Inference Ramanand was the fi rst bhakti saint to use Hindi for the propagation of his message  Shabda: Hearing in 14th Century, which popularized his preaching among local people. 45. Correct Option: (c) Explanation 47. Correct Option: (c) Guru Nanak (1469-1539) Explanation   He was born in Talwandi (Pakistan). He The IGNCA is an autonomous institution had spiritual inclination from childhood. that concentrates on research,

18 conservation, display and dissemination function of this organisation was to work of arts. Although they concentrate on the as a national organisation to promote visual and performing art, yet they also literary culture in India, to foster and promote critical and creative literature. co-ordinate literature in all the Indian languages and to overall promote the  The Crafts Council of India or CCI is a national unity of the country. non-profi t organisation that promotes activities to preserve and develop  It is an autonomous organisation, which handicraft industry in India. Kamala undertakes literary activities in over 24 Chattopadhyay who wanted to help the Indian languages. Apart from the 22 craft persons to get regular work and languages that are mentioned in the recognition for their craft, established the Constitution of India, the Sahitya Academi CCI in 1976. Although their headquarter lives recognition to two more languages: is located in , they have over English and Rajasthani. They have ten State Councils that are affi liated to several awards and fellowships that honor the parent organisation. They achieved the great contribution made to linguistic a global platform after being affi liated to development by writers the World Crafts Council.  Bhasha Samman is awarded by Sahitya Akademi for signifi cant contribution to 48. Correct Option: (b) the languages not recognised by the Academi. Explanation  The Government of India established the 49. Correct Option: (d) “National Academy of Letters” or the Sahitya Academi in 1954. The primary Explanation

Movement Founder Reasons Ahmadiya Mirza Ghulam Ahmad who It started in Punjab to create a community of people who would Movement called himself a Mahdi (guide) uphold true Islamic values. Faraizi Movement (19th Haji Shariatullah began the He called for the coming back of century) movement. pure Islam and urged Muslims to perform the obligatory duties of Islam called Faraiz. He wanted people not to go to saints and follow their rituals. Tariqh-i- Muhammadiya Sayyid Ahmad Barelvi It was essentially an armed movement to establish a proper Movement Islamic state. Aligarh Movement Sir Sayyid Ahmad Khan He propagated modern education for the Muslims,which GS SCOREwould allow them towork with the British, and inturn the Muslims will alsoprosper.

50. Correct option: (c)  The base of the painting is treated cloth while the colours used Explanation comes from natural sources including Pattachitra burnt coconut shells, Hingula, Ramaraja  and lamp black. No pencil or charcoal A traditional painting of Odisha, the is used, rather brush is used to draw name Pattachitra comes from a Sanskrit the outlines in red or yellow after which word patta, which means canvas/cloth the colours are fi lled. The background is and chitra means picture. The paintings decorated with foliage and fl owers and show a mix of classical and folk elements, the paintings have a intricately worked with a bias towards the latter. frame. Once the fi nal lines are drawn, the

19 painting is given a coating of lacquer to and squirrel hair. The painted fi gures has a give it a glossy fi nish. plaque-like eff ect on neutral background given the shaded contours and articulated Kalighat Painting movements.  In 19th century, Kalighat paintings were  Originally, the paintings depicted religious done by the rural migrants who settled note, especially Hindu Gods and around the Kalighat temple in the then British Goddesses. Over time, these paintings capital (Calcutta). Watercolours were used on mill paper using brushes made of calf were used to express social sentiments.

GS SCORE

20 57

TARGET PT - 2019

CURRENT AFFAIRS (ENVIRONMENT)

Time Allowed: 1 Hrs. Maximum Marks: 100

Roll No.:

INSTRUCTIONS

1. IMMEDIATELY AFTER THE COMMENCEMENT OF THE EXAMINATION, YOU SHOULD CHECK THAT THIS TEST BOOKLET DOES NOT HAVE ANY UNPRINTED OR TORN OR MISSING PAGES OR ITEMS, ETC. IF SO, GET IT REPLACED BY A COMPLETE TEST BOOKLET. 2. Please note that it is the candidate’s responsibility to encode and fi ll in the Roll Number carefully and without any omission or discrepancy at the appropriate places in the OMR Answer Sheet. Any omission/ discrepancy will render the Answer Sheet liable for rejection. 3. You have to enter your Roll Number on the Test Booklet in the Box provided alongside. DO NOT write anything else on the Test Booklet. 4. While writing name and Roll No. on the top of the OMR Sheet in appropriate boxes use “ONLY BLACK BALL POINT PEN”. 5. This Test Booklet contains 50 items (questions). Each item comprises four responses (answers). You will select the response which you want to mark on the Answer Sheet. In case you feel that there is more than one correct response, mark the response which you consider the best. In any case, choose ONLY ONE response for each item. 6. You have to mark all your responses ONLY on the separate OMR Answer Sheet provided. 7. All items carry equal marks. (2 marks each) 8. After you have completed fi lling in all your responses on the Answer Sheet and the examination has concluded, you should hand over to the Invigilator only the Answer Sheet. You are permitted to take away with you the Test Booklet. 9. Sheets for rough work areGS appended in the TestSCORE Booklet at the end. 10. Penalty for wrong answers: THERE WILL BE PENALTY FOR WRONG ANSWERS MARKED BY A CANDIDATE IN THE OBJECTIVE TYPE QUESTION PAPERS. (a) There are four alternatives for the answer to every question. For each question for which a wrong answer has been given by the candidate, one-third of the marks assigned to that question will be deducted as penalty. (b) If a candidate gives more than one answer, it will be treated as a wrong answer even if one of the given answers happens to be correct and there will be same penalty as above to that question. (c) If a question is left blank, i.e., no answer is given by the candidate, there will be no penalty for that question.

DO NOT OPEN THIS TEST BOOKLET UNTIL YOU ARE ASKED TO DO SO

1 TPT2019/B-1/012019/57 1. Recently, Madhya Pradesh government 2. The last census in the year 2015 showed has sought the revival of Cheetah the population of 1027 Asiatic Lions in Gir Reintroduction Project of 2009. Protected Area Network. Consider the following statements in 3. At present Gir National Park and Wildlife this regard: Sanctuary is the only abode of the Asiatic 1. The cheetah has been declared extinct in lion. India. Which of the statements given above is /are 2. Shahgarh Landscape in Rajasthan was correct? found to be the most suitable area for (a) 1 only the cheetahs as its forests are not very dense to restrict the fast movement of the (b) 1 and 2 only spotted cat. (c) 3 only 3. With the reintroduction of the cheetah, these dryland ecosystems of India will (d) 1, 2 and 3 have a chance to return to their natural state. 4. Match the following pairs: Which of the statements given above is /are A. Ice-sheet 1.Permanent fl oating correct? sheets of ice that (a) 3 only connect to a landmass. (b) 1 and 3 only B. Ice-shelve 2. Mass of glacial land ice extending more (c) 2 and 3 only than 50,000 sq km. (d) 1, 2 and 3 C. Ice-berg 3. Pieces of ice that formed on land and 2. Consider the following Statements fl oat in an ocean. with reference to the World Sustainable Development Summit 2019: Select the correct answer using the code given below: 1. The World Sustainable Development Summit (WSDS) is the annual fl agship (a) A–1; B-2; C-3 event of The Energy and Resources (b) A-2; B-1; C-3 Institute (TERI). (c) A-3; B-2; C-1 2. WSDS 2019 focussed on the theme, ‘Partnerships for a Resilient Planet’. (d) A-3; B-1; C-2 Which of the statements given above is /are correct? 5. Consider the following statements (a) 1 only regarding the Trishna Gas Project: (b) 2 only 1. The National Wildlife Board has given its approval for the Trishna Gas project of (c) Both 1 and 2 GS SCORE GAIL which falls in the Trishna Wildlife (d) Neither 1 nor 2 Sanctuary of Tripura. 2. The National Board for Wild Life is chaired 3. The Ministry of Environment, Forest by Union Minister of Environment, Forest and Climate Change, Government of and Climate Change. India has launched the “Asiatic Lion Which of the statements given above is /are Conservation Project”. Consider the incorrect? following statements in this regard: (a) 1 only 1. The total budget of the project will be funded from the Centrally Sponsored (b) 2 only Scheme- Development of Wildlife Habitat (c) Both 1 and 2 (CSS-DWH) with the contributing ratio being 50:50 of Central and State share. (d) Neither 1 nor 2

2 6. Air pollution is one of the biggest (c) 1 and 2 only global environmental challenges (d) 2 and 3 only of today. Consider the following statements with reference to situation 8. Recently, Palu city in the Indonesian of Air pollution in India: island of Sulawesi, was hit by a 1. At present, the levels of Sulphur Dioxide, Tsunami. Consider the following Nitrogen Dioxide and Particulate Matter statements in this regard: PM are above the prescribed National 10 1. This Tsunami was the result of Vertical Ambient Air Quality Standards. Movement in the Earth’s Crust. 2. The National Clean Air Programme 2. The shape of the Palu Bay played a role in (NCAP), a fi ve-year action plan with 2019 amplifi cation of force of wave. as the fi rst year is launched to tackle the 3. The retreating of Sea water is an important increasing air pollution problem warning sign of a Tsunami. 3. NCAP sets a tentative target of 20-30% Which of the above statements is / are reduction in concentrations of PM and 10 correct? PM2.5 by 2024, with 2017 as the base year for comparison. (a) 2 and 3 only Which of the statements given above is /are (b) 2 only correct? (c) 1 and 2 only (a) 1 and 3 only (d) 1 and 3 only (b) 2 and 3 only th (c) 2 only 9. The 24 Conference of the Parties to the United Nations Framework (d) 1, 2 and 3 Convention on Climate Change (COP24) took place in Katowice 7. Consider the following statements Poland in 2018. Consider the following regarding various schemes to reduce statements in this regard: carbon emission from Farming: 1. Nations have agreed to converge on a 1. Crop Diversifi cation Scheme is being Rule Book that will specify how countries implemented only in Punjab, Haryana and will agree to take forward commitments st Western Uttar Pradesh to diversify area taken at the 21 COP in Paris. from water guzzling crop like paddy to 2. All the members of the conference alternate crops like maize, pulses, oilseeds, have unanimously welcomed the cotton & agro-forestry plantation. Intergovernmental Panel on Climate Change (IPCC) report on 1.5°C. 2. Bringing Green Revolution to Eastern India (BGREI) is being implemented in 3. Poland is not a landlocked Country. seven eastern states namely Assam, Which of the statements given above is /are Bihar, Chhattisgarh, Jharkhand, Odisha, correct? West Bengal and eastern Uttar Pradesh GS SCORE(a) 3 only to address the constraints limiting the productivity of rice based cropping (b) 1 and 3 only systems. (c) 1 and 2 only 3. The Paramparagat Krishi Vikas Yojana (d) 2 only (PKVY) is being implemented in 7 North Eastern States for development of certifi ed organic production in a value chain mode 10. Consider the following statements to link growers with consumers. regarding Akademik Lomonosov: Which of the statements given above is /are 1. It is world’s fi rst fl oating Solar Power Plant correct? made Operational by Russia. 2. Russia will primarily use it to power oil (a) 1 only rigs in remote areas of Artic region where (b) 1 and 3 only Russia is pushing to drill for oil and gas.

3 Which of the statements given above is /are 1. Soil biodiversity is the variety of life that incorrect? exists within the soil, including bacteria, fungi, earthworms and termites. (a) 1 only 2. As per the Global Soil Biodiversity Atlas (b) 2 only prepared by Wildlife Institute of India (c) Both 1 and 2 (WII), India faces a grave danger to soil biodiversity. (d) Neither 1 nor 2 Which of the statements given above is /are incorrect? 11. Which of the following Particularly (a) 1 only Vulnerable Tribal Groups (PVTGs) are not found in Andaman and Nicobar (b) 2 only Islands? (c) Both 1 and 2 1. Great Andamanese (d) Neither 1 nor 2 2. Onge 3. Shompens 14. Consider the following statements regarding Shifting Cultivation: 4. Juang 1. Shifting cultivation is an agricultural 5. Sentinele system in which a piece of land is cleared 6. Nicobarese and then abandoned followed by several years of farming until the soil loses Select the correct answer using the code given fertility. below: 2. Shifting Cultivation is also known as Jhum (a) 2 and 5 only Cultivation in Southern States of India (b) 3 and 4 only 3. This practice increases surface run off of rainwater leading to soil erosion. (c) 4 only Which of the statements given above is /are (d) 1, 6 and 4 only correct? (a) 1 only 12. Zoological Survey of India (ZSI) has (b) 1 and 2 only published a report titled: ‘Faunal Diversity of Bio-geographic Zones: (c) 1 and 3 only Islands of India’. Consider the (d) 1, 2 and 3 following statements in this regard: 1. Andaman and Nicobar Islands, which 15. Consider the following statements comprises of 572 islands is home to more regarding Black Carbon: than 10% of country’s fauna species. 1. Black carbon is a component of particulate 2. 11o channel separates Andaman and matter formed by the incomplete Nicobar islands. combustion of fossil fuels, wood and GS SCOREother fuels. 3. Zoological Survey of India publishes Red Data book on Indian Flora and Fauna. 2. Black carbon is a long-lived climate pollutant with a lifetime of several years. Which of the above statements is /are 3. When deposited on ice and snow, black correct? carbon and co-emitted particles increases (a) 1 only the surface albedo. (b) 1 and 3 only Which of the statements given above is /are correct? (c) 1 and 2 only (a) 1 only (d) 1, 2 and 3 (b) 1 and 2 only 13. Consider the following statements (c) 1 and 3 only with reference to Soil Biodiversity: (d) 1, 2 and 3

4 16. Consider the following statements Which of the statements given above is /are regarding Glacial Lakes: correct? 1. A Glacial Lake Outburst Flood (GLOF) is a (a) 1 only release of melt water from a moraine. (b) 2 and 3 only 2. Moraines are elongated, teardrop-shaped (c) 3 only hills of rock, sand, and gravel that formed under moving glacier ice (d) 1, 2 and 3 3. Kedarnath Tragedy was caused by a GLOF 19. Recently, Mangalajodi Ecotourism Trust has won prestigious United Which of the statements given above is /are Nations World Tourism Organisation correct? (UNWTO) Award. Consider the (a) 1 only following statements in this regard: (b) 1 and 2 only 1. Mangalajodi Ecotourism Trust is government owned and managed venture (c) 3 only on banks of Chilika Lake in Odisha. (d) 1, 2 and 3 2. Chilika Lake is the 2nd largest coastal lagoon in India after Pulikat Lake. 17. Consider the following statements 3. Lagoons are shallow bodies of water that regarding Carbon Sinks: provide access to the sea. 1. A carbon sink is anything that absorbs 4. The headquarter of World Tourism more carbon than it releases as carbon Organization is located in Geneva, dioxide. Switzerland. 2. India has committed a target of creating Which of the statements given above is /are an additional carbon sink equivalent to 2.5 correct? billion to 3 billion tonnes carbon dioxide (a) 1 and 2 only by 2030 in COP 24. (b) 1, 3 and 4 only Which of the statements given above is /are correct? (c) 3 only (a) 1 only (d) 2, 3 and 4 only (b) 2 only 20. Consider the following Statements (c) Both 1 and 2 regarding Floating Treatment (d) Neither 1 nor 2 Wetlands: 1. Floating Treatment Wetlands are small 18. Consider the following statements artifi cial platforms which are useful for regarding Zero Budget Natural the removal of contaminants from the Freshwater Lakes. Farming: GS SCORE 2. They work on the process of 1. It is a Farming practice, very similar to Bioremediation. Organic Farming that believes in natural growth of crops without adding any 3. Recently, a Floating Treatment Wetland (FTW) was inaugurated on World Wetlands fertilizers and pesticides or any other Day in Neknampur Lake in . foreign elements. Which of the statements given above is /are 2. Under this practice, the farmers are going correct? to have no costs at all. 3. The four pillars of ZBNF are: Jeevamrutha- (a) 1 only a fermented microbial culture; (b) 1 and 2 only Beejamrutha-a treatment used for seeds; (c) 1, 2 and 3 Acchadana – Mulching and Whapasa – moisture. (d) None

5 21. Consider the following statements 3. India’s total forest and tree cover is around regarding Olive Ridley Turtles: 35% of the Total geographical area of the country. 1. They are most abundant of all sea turtles found in the world. Select the correct answer using the code given 2. Olive Ridley Turtle is recognized as below: Vulnerable by IUCN Red list. (a) 2 only 3. The government of Orissa has declared (b) 1 and 2 only Rushikulya rookery a Protected Areadue to the consistent arrival of lakhs and lakhs (c) 3 only of Olive Ridley turtles. (d) 1, 2 and 3 Which of the statements given above is /are correct? 24. Consider the following statements (a) 2 only with reference to National Biogas and (b) 1 and 2 only Manure Management Programme (NBMMP). (c) 1 and 3 only 1. It is a Central Sector Scheme. (d) 1, 2 and 3 2. The scheme provides for setting up of Family Type Biogas Plants mainly for rural 22. A two day international conference and semi-urban households on Sustainable Biofuels was jointly 3. The process of biogas generation is called organized by Government of India Anaerobic digestion (AD). and Biofuture platform. Consider the Which of the statements given above is /are following statements in this regard: correct? 1. Mission Innovation (MI) is a global (a) 1 only initiative to accelerate global clean energy innovation with the objective to make (b) 2 only clean energy widely aff ordable. (c) 1 and 3 only 2. Ministry of New and Renewable Energy (d) 1, 2 and 3 is the nodal agency coordinating and steering the activities of Mission Innovation in India 25. Recently, the United Nations World 3. Fourth-generation biofuels are a carbon Water Development Report (WWDR) negative source of fuel. 2018 was released. Consider the following statements in this regard: Which of the statements given above is /are correct? 1. The development of the WWDR is coordinated by the World Water (a) 2 only Assessment Programme (WWAP), a joint (b) 1 and 2 only eff ort of the UN agencies and entities which make up UN-Water (c) 1 and 3 only GS SCORE 2. UN Water is the single UN entity dedicated (d) 1, 2 and 3 exclusively to water issues. 3. The report highlights that Global demand 23. As per the Draft National Forest for water has increased six-fold over the Policy 2018, released by Ministry past 100 years and continues to grow at of Environment, Forest and climate the rate of 1% each year. Change, Which of the following Which of the above statements is /are statements is/are correct? correct? 1. The current adopted Forest Policy of India (a) 2 only is National Forest Policy, 1952. (b) 3 only 2. The new draft Forest Policy seeks to establish a National Board of Forestry and (c) 1 and 3 only State Boards of Forestry. (d) 2 and 3 only

6 26. Consider the following statements 3. Another main reason for the dust storms regarding the One Planet One City to be catastrophic was because the wind Challenge: hit at night. 1. It’s a friendly competition, organized Which of the statements given above is /are by World Wide Fund for Nature (WWF) correct? to mobilize cities to deliver on the Paris (a) 1 only Agreement. (b) 2 only 2. The logo of WWF is a Black and White Rhinoceros. (c) 1 and 3 only 3. Pune was selected as National Winner in (d) 1, 2 and 3 2017-2018 edition of WWF’s One Planet City Challenge (OPCC). 29. Consider the following statements Which of the statements given above is /are regarding Suva Expert Dialogue which correct? took place recently: (a) 1 only 1. The dialogue was organised to minimize (b) 1 and 2 only and address the loss and damages associated with the adverse eff ects of (c) 3 only Shipping Activities. (d) 2 and 3 only 2. It took place in Bonn, Germany. 3. The Expert Dialogue was set up in 27. Consider the following statements with response to request made by developing reference to Solar Geo Engineering: countries in Conference of Parties 23rd 1. Solar Geo-engineering is a theoretical meet (COP23). approach to refl ect some amount of Which of the statements given above is /are inbound sunlight back out into space. correct? 2. Most countries are willing to implement Solar Geo Engineering Methods like (a) 1 only Stratospheric aerosol injection, Marine (b) 1 and 2 only cloud brightening as these methods have negligible harmful eff ects. (c) 3 only Which of the statements given above is /are (d) 2 and 3 only correct? (a) 1 only 30. Consider the following statements with reference to SAWEN, an (b) 2 only intergovernmental Agency. (c) Both 1 and 2 1. SAWEN is an inter-governmental Weather (d) Neither 1 nor 2 forecasting Network of South Asian GS SCOREcountries. 28. Around 100 people were recently 2. All the members of SAARC are members killed by a massive dust storm in of SAWEN. the Northern Part of India. With 3. SAWEN Secretariat is located in reference to it, consider the following Kathmandu, Nepal. statements: Which of the statements given above is /are 1. Dust Storms are an ensemble of particles correct? of dust or sand energetically lifted to great heights by a strong and turbulent wind. (a) 1 only 2. According to Meteorologists, it was an (b) 2 only intense upward movement of air, known (c) 1 and 3 only as upburst which produced strongest wind gust. (d) 2 and 3 only

7 31. Consider the following statements Which of the statements given above is /are with reference to Composite Water correct? Management Index 2018: (a) 1 only 1. It is developed by Ministry of Drinking (b) 1 and 2 only water and Sanitation to assess and (c) 3 only improve the performance of States/ Union Territories in effi cient management (d) 1, 2 and 3 of water resources. 2. The reporting states have been divided 34. Consider the following statements into two special groups – ‘North Eastern regarding impact of climate change in and Himalayan states’ and ‘Other States’. South Asia: 3. The report ranks Gujarat as number one in 1. Asian Development Bank has recently the reference year (2016-17) and in North released South Asia’s Hotspots Report to Eastern and Himalayan States, Himachal assess the Impact of climate change on Pradesh has been adjudged number 1 in Living Standards in South Asian Region. 2016-17. 2. The report identifi es climate “hotspots” Which of the statements given above is /are as a small area of the Earth’s crust where correct? an unusually high heat fl ow is associated with volcanic activity. (a) 1 and 2 only 3. As per the Report, if preventive measures (b) 2 only are taken along the lines of the Paris (c) 2 and 3 only Agreement, India’s average annual temperatures are expected to rise by (d) 1, 2 and 3 1-2°C by 2050. Which of the statements given above is /are 32. Consider the following statements correct? regarding Dead Zones: (a) 1 and 3 only 1. Dead Zones or Hypoxia are areas in the (b) 1 and 2 only ocean of very high Salinity in which animal life suff ocates and dies. (c) 3 only 2. Recently, scientists have found a huge (d) 1, 2 and 3 dead zone in Bay of Bengal Which of the statements given above is /are 35. The India Meteorological Department correct? (IMD) has recently launched new (a) 1 only Ensemble Prediction Systems (EPS) to provide probabilistic weather (b) 2 only forecasts. Consider the following (c) Both 1 and 2 statements in this regard: (d) Neither 1 nor 2GS SCORE1. The new systems have a resolution of 12 km Grid Scale, which is currently 23 km scale grid. 33. Consider the following statements regarding the National Disaster Risk 2. The EPS involves the generation of multiple forecasts using slightly varying Index: initial conditions. 1. It is made by the Union Home Ministry 3. India Meteorological Department comes with support from the United Nations under the Ministry of Science and Development Programme (UNDP). Technology. 2. Maharashtra has been ranked at the top Which of the statements given above is /are of the list of Indian states, vulnerable to correct? natural disasters (a) 2 only 3. Among the Union Territories, Puducherry is the most vulnerable to such disasters. (b) 2 and 3 only

8 (c) 1 and 2 only 2. It is the fi rst interval in Earth’s geological history that coincided with a major cultural (d) 1, 2 and 3 event. 3. The IUGS is an inter-governmental 36. According to recent study, arsenic organization devoted to international contamination in paddy is rising cooperation in the fi eld of geology. from ground water in West Bengal. Consider the following statements in Which of the statements given above is /are correct? this regard: (a) 1 and 2 only 1. Arsenic occurs naturally in the environment in both organic and inorganic forms. (b) 1 only 2. According to WHO, Maximum contaminant (c) 2 and 3 only level (MCL) for arsenic in drinking water is (d) 1, 2 and 3 50 parts per Billion (ppb). 3. Arsenic cannot be removed by boiling the water. 39. Consider the following statements regarding the recently launched Deep Which of the statements given above is /are Ocean Mission: correct? 1. It is launched by Union Ministry of Earth (a) 1 only Sciences. (b) 2 only 2. Exclusive Economic Zone of a concerned (c) 2 and 3 only country extends from the baseline to 120 nautical miles from the coast. (d) 1 and 3 only 3. A Submersible Vehicle and a Desalination plant are the two key projects planned 37. Consider the following statements under Deep Ocean Mission. with reference to the Magnetic Field of the Earth: Which of the statements given above is /are correct? 1. Earth’s magnetic fi eld is caused due to the diff erential rate of spinning of Inner (a) 3 only and Outer Core. (b) 1 and 2 only 2. It takes around 200,000 to 300,000 years (c) 1 and 3 only for Earth’s magnetic fi eld to fl ip polarity. (d) 1, 2 and 3 3. The reversal of the Magnetic Polarity of Earth could lead to its Destruction. 40. India has prepared its National REDD+ Which of the statements given above is /are Strategy in line with its commitment to correct? Paris Agreement on Climate Change. (a) 1 only Consider the following Statements in (b) 1 and 2 only this regard: (c) 2 and 3 only GS SCORE1. The strategy report has been prepared by Indian Institute of Forest Management, (d) 1, 2 and 3 Bhopal. 2. REDD+ simply means, “Reducing 38. International Union of Geological Emission from Deforestation and Forest Sciences (IUGS) has offi cially Degradation”. accepted the new phase in Earth’s Which of the statements given above is /are geological history and designated correct? it as Meghalayan Age. Consider the following statements in this regard: (a) 1 only 1. It has been named after a stalactite which (b) 2 only was found from a cave in the Indian state (c) Both 1 and 2 of Meghalaya that helped defi ne climatic events 4,200 years ago. (d) Neither 1 nor 2

9 41. With reference to the Brazzaville Which of the statements given above is /are Declaration, consider the following correct? statements: (a) 1 only 1. It is aimed to protect the Peruvian Amazon (b) 1 and 2 only region in South America, world’s largest tropical peatland. (c) 1, 2 and 3 2. Democratic Republic of Congo (DRC) and (d) None the Republic of Congo have jointly signed the declaration. 44. Consider the following statements 3. Peatlands are wetlands that contain a regarding El Nino: mixture of decomposed organic material, partially submerged in a layer of water, 1. During El Nino year, a surface low pressure lacking oxygen develops in the region of northern Australia and Indonesia and a high pressure system Which of the statements given above is /are correct? over the coast of Peru. 2. El Nino and Indian monsoon are inversely (a) 1 and 3 only related. (b) 2 only Which of the statements given above is /are (c) 3 only correct? (d) 1, 2 and 3 (a) 1 only (b) 2 only 42. An IISc study claims that Heat Waves have increased in frequency in the (c) Both 1 and 2 recent decades in India, consider the (d) Neither 1 nor 2 following statements in this regard: 1. A Heat Wave is a period of abnormally 45. Consider the following statements high temperatures, more than the normal regarding Cyclone Mora: maximum temperature that occurs during the summer season in the South-Eastern 1. Cyclone Mora got its name from parts of India. Thailand 2. Heat wave is considered if maximum 2. It was a result of a deep depression temperature of a station reaches at least developed in the Bay of Bengal 40°C or more for plains, 37°C or more for 3. Life period of a tropical cyclone over the coastal stations at least 30°C or more for north Indian Ocean is 5-6 days hilly regions. Which of the statements given above is /are Which of the statements given above is /are correct? correct? (a) 1 only (a) 1 and 2 only (b) 2 only (b) 1 and 3 only (c) Both 1 and 2 GS SCORE(c) 3 only (d) Neither 1 nor 2 (d) 1, 2 and 3

43. Consider the following statements 46. Consider the following Statements: with reference to Flammable Ice: 1. The Central Government has approved the 1. Flammable ice (also known as methane development of a Turtle Sanctuary along hydrate) consists of methane trapped with a River Biodiversity Park in Kanpur, within water crystals. under Namami Gange Programme. 2. Methane Hydrate is highly fl ammable and 2. National Ganga River Basin Authority energy intensive Fuel. (NGRBA) is the implementation arm of the 3. India has successfully produced natural gas National Mission for Clean Ganga (NMCG) from methane hydrate in an experimental under the Environment Protection Act project in the Indian Ocean. 1986.

10 Which of the statements given above is /are (c) 3 only correct? (d) 1, 2 and 3 (a) 1 only (b) 2 only 49. Consider the following statements: (c) Both 1 and 2 1. 8th Regional 3R Forum in Asia and the (d) Neither 1 nor 2 Pacifi c was held in Cuttack, Odisha, India. 2. It was co-organized by Ministry of Housing 47. Consider the following statements: and Aff airs, Japan’s Environment Ministry, 1. United Nation (UN) Water released the and United Nations Centre for Regional annual World Water Development Report Development (UNCRD). 2018, titled ‘Waste Water – The Untapped 3. It was launched with objective of Resource’ integrating 3Rs– reduce, reuse and recycle 2. UN Water is the only single UN entity in policy, planning and development. dedicated exclusively to water issues. Which of the statements given above is /are Which of the statements given above is /are correct? correct? (a) 2 only (a) 1 only (b) 2 and 3 only (b) 2 only (c) Both 1 and 2 (c) 1, 2 and 3 (d) Neither 1 nor 2 (d) None

48. Consider the following Statements 50. Consider the following statements: regarding Persistent Organic 1. 16th biennial International Energy Forum Pollutants (POPs): Ministerial Meeting (IEF16) 2018 was 1. Persistent Organic Pollutants (POPs) are hosted in New Delhi organic substances that remain intact for 2. The theme chosen for IEF16 Ministerial exceptionally long periods of time and was ‘Global Energy Transition an Enhanced are toxic to both humans and wildlife Role for the Dialogue’. 2. POPs bio-magnify throughout the food chain and bio-accumulate in organisms. 3. IEF’s Secretariat is headquartered at Riyadh, Saudi Arabia 3. Rotterdam Convention is a global treaty to protect human health and the Which of the statements given above is /are environment from harmful eff ects of correct? POPs (a) 2 only Which of the statements given above is /are correct? GS SCORE(b) 2 and 3 only (a) 1 only (c) 1, 2 and 3 (b) 1 and 2 only (d) 1 and 3 only

**********

11 57

TARGET PT - 2019

CURRENT AFFAIRS (ENVIRONMENT)

Answer Key

Q. 1 (b) Q. 14 (c) Q. 27 (a) Q. 40 (d)

Q. 2 (a) Q. 15 (a) Q. 28 (c) Q. 41 (c)

Q. 3 (c) Q. 16 (a) Q. 29 (d) Q. 42 (b)

Q. 4 (b) Q. 17 (c) Q. 30 (d) Q. 43 (b)

Q. 5 (c) Q. 18 (c) Q. 31 (b) Q. 44 (b)

Q. 6 (b) Q. 19 (c) Q. 32 (b) Q. 45 (d)

Q. 7 (c) Q. 20 (b) Q. 33 (b) Q. 46 (d)

Q. 8 (a) GSQ. 21 (b) SCOREQ. 34 (c) Q. 47 (d)

Q. 9 (b) Q. 22 (c) Q. 35 (c) Q. 48 (b)

Q. 10 (a) Q. 23 (a) Q. 36 (d) Q. 49 (b)

Q. 11 (c) Q. 24 (d) Q. 37 (b) Q. 50 (d)

Q. 12 (a) Q. 25 (c) Q. 38 (c)

Q. 13 (b) Q. 26 (a) Q. 39 (c)

DO NOT OPEN THIS TEST BOOKLET UNTIL YOU ARE ASKED TO DO SO

1 TPT2019/B-1/012019/57 1. Correct Option: (b) Red List. The Ministry of Environment, Forest and Climate Change, India has Explanation: launched the “Asiatic Lion Conservation  Statement 1 is correct: A Cheetah is a big Project” with an aim to protect and cat and is a member of the family Felidae. conserve the world’s last ranging The cheetah was declared extinct in India free population of Asiatic Lion and its in 1952. It is the only large mammal associated ecosystem. The total budget to have been declared extinct in our of the project for 3 years will be funded country in recorded history. from the Centrally Sponsored Scheme-  Development of Wildlife Habitat (CSS- Statement 2 is incorrect: Nauradehi DWH) with the contributing ratio being Santuary of Madhya Pradesh was 60:40 of Central and State share. found to be the most suitable area for the cheetahs as its forests are not very  Statement 2 is incorrect: The last census dense to restrict the fast movement of in the year 2015 showed the population the spotted cat. Besides, the prey base of 523 Asiatic Lions in Gir Protected for cheetahs is also in abundance at the Area Network. sanctuary.  Statement 3 is correct: At present Gir  Statement 3 is correct: The cheetah is National Park and Wildlife Sanctuary is the fl agship species of the grasslands, the only abode of the Asiatic lion. scrublands and open forests. Therefore, with the reintroduction of the cheetah, these dryland ecosystems of India will 4. Correct Option: (b) have a chance to return to their natural Explanation: state. Wildlife Institute of India started this ambitious Cheetah Reintroduction  Ice sheet is a mass of glacial land ice Project in 2009. The project focused extending more than 50,000 square on relocating cheetah from Namibia kilometers (20,000 square miles). The in Africa to Nauradehi sanctuary in two ice sheets on Earth today cover Madhya Pradesh. most of Greenland and Antarctica. If the Greenland Ice Sheet melted, scientists 2. Correct Option: (a) estimate that sea level would rise about 6 meters (20 feet). If the Antarctic Ice Explanation: Sheet melted, sea level would rise by  Statement 1 is correct: The World about 60 meters (200 feet) Sustainable Development Summit is  the annual fl agship event of The Energy Ice shelves are permanent fl oating sheets and Resources Institute (TERI). World of ice that connect to a landmass. Most of Sustainable Development Summit is the world’s ice shelves hug the coast of continuing the legacy of Delhi Sustainable Antarctica. Development Summit (DSDS) which was  Icebergs are pieces of ice that formed initiated in 2001 with the aim of making on land and fl oat in an ocean or lake. ‘sustainable development’ a globally Icebergs come in all shapes and sizes, shared goal. from ice-cube-sized chunks to ice  Statement 2 is incorrect: WSDS 2019 islands the size of a small country. The will focus onGS the theme, ‘Attaining SCORE North Atlantic and the cold waters the 2030 Agenda: Delivering on our surrounding Antarctica are home to Promise’. TERI is an independent, most of the icebergs on Earth. multi-dimensional organization, with capabilities in research, policy, consultancy and implementation. It aims to usher 5. Correct Option: (c) transitions to a cleaner and sustainable Explanation: future through the conservation and effi cient use of Earth’s resources.  Statement 1 is incorrect: The National Wildlife Board has given its approval for 3. Correct Option: (c) the Trishna Gas project of ONGC which falls in the Trishna Wildlife Sanctuary in Explanation: the Gomati district of Tripura. ONGC has  Statement 1 is incorrect: Asiatic Lions are discovered 10-12 gas bearing wells in listed as ‘Endangered’ under the IUCN the Trishna Wildlife sanctuary.

2  Statement 2 is incorrect: National Board a sub-scheme of Rashtriya Krishi Vikas for Wild Life is a statutory organization Yojna (RKVY) is being implemented in constituted under the Wildlife Protection seven eastern states namely Assam, Bihar, Act, 1972. It is an apex body to review Chhattisgarh, Jharkhand, Odisha, West all wildlife-related matters and approves Bengal and eastern Uttar Pradesh from projects in and around national parks 2010-2011 to address the constraints and sanctuaries. The National Board for limiting the productivity of rice based Wild Life is chaired by the Prime Minister, cropping systems Union Minister of Environment, Forest  Statement 3 is incorrect: The and Climate Change is the vice- Paramparagat Krishi Vikas Yojana chairman of the Board. (PKVY), launched in 2015, is an extended component of Soil Health Management 6. Correct Option: (b) (SHM) under the Centrally Sponsored Explanation: Scheme (CSS), National Mission on Sustainable Agriculture (NMSA). PKVY  Statement 1 is incorrect: The levels of aims at supporting and promoting sulphur dioxide and Nitrogen Dioxide organic farming, in turn resulting in are within the prescribed National improvement of soil health. It is being Ambient Air Quality Standards but the implement in whole of India.

levels of PM10 exceed the prescribed National Ambient Air Quality Standards in most of the cities. 8. Correct Option: (a)  Statement 2 is correct: National Clean Explanation: Air Programme (NCAP) was launched  Statement 1 is incorrect: Tsunamis more by Union Minister of Environment, Forest commonly follow vertical movement in and Climate Change. Collaborative the crust, which disrupts the overlying and participatory approach involving water and can generate massive waves relevant Central Ministries, State crashing onshore. But the 7.5-magnitude Governments, local bodies forms the earthquake appears to be the result of crux of the Programme. The NCAP will be what’s known as a strike-slip fault, which a mid-term, fi ve-year action plan with takes place as two blocks of crust grind 2019 as the fi rst year. against one another, largely in a horizontal  Statement 3 is correct: NCAP sets the direction. tentative national level target of 20%–  Statement 2 is correct: The narrow shape 30% reduction of PM2.5 and PM10 of Palu bay concentrated and amplifi ed concentration by 2024 taking 2017 the force of the wave. It funneled the as the base year for the comparison of energy and concentrate it at the tip and concentration. that focused the tsunami. It is also a very a deep bay, which means the tsunami 7. Correct Option: (c) could move at high speed.  Explanation: Statement 3 is correct: A tsunami’s trough, the low point beneath the wave’s  Statement 1GS is correct: SCORECrop crest, often reaches shore fi rst. When it Diversifi cation Programme (CDP), a does, it produces a vacuum eff ect that sub scheme of Rashtriya Krishi Vikas sucks coastal water seaward and exposes Yojna (RKVY), is being implemented in harbor and sea fl oors. This retreating of the Original Green Revolution States of sea water is an important warning sign of Punjab, Haryana and Western Uttar a tsunami, because the wave’s crest and Pradesh from 2013-14 to diversify area its enormous volume of water typically hit from water guzzling crop like paddy shore fi ve minutes or so later. to alternate crops like maize, pulses, oilseeds, cotton & agro-forestry plantation. It has been continued in 9. Correct Option: (b) 2014-15 Explanation:  Statement 2 is correct: Bringing Green  Statement 1 is correct: The 2018 United Revolution to Eastern India (BGREI), Nations Climate Change Conference

3 that took place between 2 and 15 11. Correct Option: (c) December 2018 in Katowice, Poland, is Explanation: the 24th Conference of the Parties to the United Nations Framework Convention  The population of the Andamand on Climate Change (COP24). One of the and Nicobar islands includes six main outcome of COP24 is that Nations particularly vulnerable tribal groups will try to converge on a rule book that (PVTGs) — Great Andamanese, Onge, will specify how countries will agree to Jarawa, Sentinelese, Nicobarese and take forward commitments taken at the Shompens. 21st COP in Paris. This ‘rulebook’ can  Juang is a PVTG belonging to the State be called as the detailed “operating of Odisha. manual” of the 2015 Paris Agreement.  Particularly Vulnerable Tribal Groups:  Statement 2 is incorrect: The members 75 tribal groups have been categorized of the conference did not agree to categorized by Ministry of Home “welcome” the Intergovernmental Panel Aff airs as Particularly Vulnerable Tribal on Climate Change (IPCC) report on 1.5°C. Groups (PVTG)s. PVTGs reside in 18 States The US, Saudi Arabia, Russia and Kuwait and UT of A&N Islands. The Ministry of refused to “welcome” the IPCC report Tribal Aff airs implements the Scheme of “Development of Particularly  Statement 3 is correct: Poland is Vulnerable Tribal Groups (PVTGs)” bordering Germany to the west, the exclusively for them. Czech Republic and Slovakia to the south, Ukraine and Belarus to the east, 12. Correct Option: (a) and Lithuania and the Russian enclave of Kaliningrad to the northeast. Its northern Explanation: frontier constitutes the Baltic Sea.  Statement 1 is correct: Andaman and Nicobar Islands comprises of 572 islands, 10. Correct Option: (a) islets and rocky outcrops. It comprises only 0.25% of India’s geographical Explanation: area but is home to more than 10% of  Statement 1 is incorrect: Russia’s country’s fauna species. Thus, presence Akademik Lomonosov, the world’s fi rst of such large number of species in such small area makes A&I Islands one of the fl oating nuclear power plant (FNPP) richest ecosystems and biodiversity hot has become operational. It is constructed spots in India. by a nuclear power fi rm Rosatom. For power generation, it has been fi tted with  Statement 2 is incorrect: Ten degree two modifi ed KLT-40 naval propulsion Channel separates the islands of Andaman nuclear reactors (each of 35 MW and Nicobar. capacity) together providing up to 70  Statement 3 is incorrect: Zoological MW of electricity and 300 MW of heat. Survey of India (ZSI) and Botanical It is named after Russian Academician Survey of India (BSI) under the Ministry Mikhail Lomonosov. of Environment, Forest and Climate  GS SCOREChange conduct surveys of threatened Statement 2 is correct: The low-capacity, and important species of fauna and fl ora mobile (fl oating) nuclear power plant respectively of the country. Zoological can produce enough electricity to power Survey of India (ZSI) is India’s apex town of 200,000 residents living in Russia’s organization on animal taxonomy. far-fl ung northernmost Artic region Its objective is to promote survey, where large amounts of electricity exploration, research and documentation is not needed and construction of on various aspects of animal taxonomy in conventional power station based on Indian subcontinent. coal, gas and diesel is complicated and costly. It can save upto 50,000 tonnes of 13. Correct Option: (b) carbon dioxide emissions per year. The project is part of Russia’s greater aims Explanation: to secure rich deposits of oil and gas in  Statement 1 is correct: Soil biodiversity North Pole region in Artic. is the variety of life that exists within

4 the soil, including bacteria, fungi, Batra South-eastern Rajasthan earthworms and termites. A teaspoon of topsoil typically contains a vast range Podu Andhra Pradesh of diff erent species and up to 6 billion microorganisms. The maintenance Kumari Hilly Region of the Western of soil biodiversity is essential to both Ghats of Kerala the environment and to agricultural industries. Kaman, Vinga Odisha and Dhavi  Statement 2 is incorrect: According to recently released Global Soil Biodiversity  Statement 3 is correct: Shifting cultivation Atlas prepared by World Wide Fund for causes in long term problem of land Nature (WWF), India among nations that degradation and threat to ecology of face grave danger to soil biodiversity. the region at large. Burning of forests The atlas was released as part of WWF’s results in emissions of greenhouse biennial Living Planet Report (LPR) gases (GHGs) such as CO2, NO2. It also 2018. India’s soil biodiversity is in grave increases surface run off of rainwater peril according to the report. The key leading to soil erosion. aspect of this year’s report was threat to soil biodiversity and pollinators, the two key drivers of biodiversity. 15. Correct Option: (a)  World Wide Fund for Nature (WWF) Explanation: is international non-governmental  Statement 1 is correct: Black carbon is organization working fi eld of the a potent climate-warming component wilderness preservation and reduction of particulate matter formed by the of human impact on the environment. incomplete combustion of fossil fuels, It is world’s largest conservation wood and other fuels. The complex organization with over fi ve million mixture of particulate matter resulting supporters worldwide, working in more from incomplete combustion is often than 100 countries. referred to as soot. Black carbon and its co-pollutants are key components 14. Correct Option: (c) of fi ne particulate matter (PM2.5) air pollution Explanation:  Statement 2 is incorrect: Black carbon  Statement 1 is correct: Shifting cultivation is a short-lived climate pollutant with is an agricultural system which involves a lifetime of only days to weeks after clearing of a piece of land followed by several years of wood harvesting or release in the atmosphere. During this farming until the soil loses fertility. Once short period of time, black carbon can the land becomes inadequate for crop have signifi cant direct and indirect impacts production, it is left to be reclaimed on the climate, glacial regions, agriculture by natural vegetation, or sometimes and human health converted to a diff erent long term  Statement 3 is incorrect: When cyclical farmingGS practice. This system SCORE of deposited on ice and snow, black carbon agriculture is often practised at the level and co-emitted particles reduce surface of an individual or family. albedo (the ability to refl ect sunlight)  Statement 2 is incorrect: In north east and heat the surface. The Arctic and India, it is called as jhum cultivation. glaciated regions such as the Himalayas are particularly vulnerable to melting as Jhum North-eastern India a result. Vevar and Bundelkhand Region (Madhya Dahiyaar Pradesh) 16. Correct Option: (a) Deepa Bastar District (Madhya Explanation: Pradesh) x Statement 1 in correct: A glacial lake outburst fl ood (GLOF) is a release of Zara and Erka Southern States meltwater from a moraine- or ice-dam glacial lake due to dam failure. GLOFs often

5 result in catastrophic fl ooding downstream, without adding any fertilizers and with major geomorphic and socioeconomic pesticides or any other foreign elements. impacts But it is diff erent from organic farming. x Statement 2 in incorrect: Moraines are accumulations of dirt and rocks that have  Diff erence Between Organic Farming fallen onto the glacier surface or have been and Zero Budget Natural Farming pushed along by the glacier as it moves.  In organic farming, organic fertilizers Drumlins are elongated, teardrop-shaped hills of rock, sand, and gravel that formed and manures like compost, under moving glacier ice. They can be up to 2 vermicompost, cow dung manure, kilometers (1.25 miles) long. etc. are used whereas in ZBNF, no external fertilizers are added to soil x Statement 3 in incorrect: The term ‘Glacial or give to plants whatsoever, instead, Lake Outburst Flood’ was making news decomposition of organic matter because of rumours that the Kedarnath by microbes and earthworms is tragedy of last year was caused by a GLOF. encouraged. However, it was confi rmed later that this  tragedy was not caused due to GLOF but Organic farming still requires basic due to a combination of several factors agro practices like plowing, tilting, viz. early rainfall, movement of southwest mixing of manures, weeding, etc. to monsoon winds, and the formation of a be performed whereas In ZBNF there temporary lake. no plowing, no tilting of soil and no fertilizers, and no weeding is done just the way it would be in natural 17. Correct Option: (c) ecosystems Explanation:  Organic farming is still expensive due to the requirement of bulk manures,  Statement 1 is correct: A carbon sink is and it has an ecological impact on anything that absorbs more carbon than surrounding environments; whereas, it releases as carbon dioxide. In climate natural agriculture is an extremely negotiations, this temporary reduction of low-cost farming method, completely carbon dioxide in the atmosphere is also known as negative emissions. United molding with local biodiversity. Nations accepts the planting trees or  Statement 2 is incorrect: The word reducing deforestation is equivalent to Zero Budget refers to zero net cost of reducing emissions from burning fossil production of all crops. It doesn’t mean fuels. The carbon stock in India is roughly that the farmer is going to have no 7 billion tonnes, equivalent to 25.66 costs at all, but rather that any costs billion tonnes of carbon dioxide. The will be compensated for by income average annual increment of carbon stock from intercrops, making farming a close is 35 million tonnes which is equivalent to to zero budget activity. Farmers need 128.33 million tonnes carbon dioxide not purchase fertilizers and pesticides  Statement 2 is correct: India has in order to ensure the healthy growth of committed a GS target of creating SCORE an crops. additional carbon sink the equivalent  Statement 3 is correct: The four pillars of 2.5 billion to 3 billion tonnes carbon of ZNBF are: dioxide by 2030 in COP 24. But India now  anticipates that it might not be able Jeevamrutha- a fermented microbial to meet that through forests alone. It culture, which acts as a catalytic is now looking at the soil of catchment agent that promotes the activity of areas as an additional alternative. microorganisms in the soil;  Beejamrutha-a treatment used for 18. Correct Option: (c) seeds, to protect young roots from fungus as well as from soil-borne and Explanation: seedborne diseases.  Statement 1 in incorrect: Zero Budget  Acchadana – Mulching protects top Natural Farming is a farming practice soil during cultivation and does not that believes in natural growth of crops destroy it by tilling.

6  Whapasa – Whapasa is the condition nutrients is widely practiced around the where there are both air molecules and world. Their roots spread through the water molecules present in the soil fl oating islands and down into the water creating dense columns of roots 19. Correct Option: (c) with lots of surface area.  Explanation: Statement 2 is correct: The natural ability of plants and microbes to absorb  Statement 1 is incorrect: Mangalajodi nutrients (such as phosphorus and Ecotourism Trust is community owned nitrogen) and break down contaminants and managed venture promoted by RBS through biological processes is known as Foundation India and Indian Grameen bioremediation. Services on banks of Chilika Lake in Odisha.  Statement 3 is incorrect: The Floating  Treatment Wetland (FTW) was inaugurated Statement 2 is incorrect: Chilika Lake on World Wetlands Day (February 2) is largest coastal lagoon or brackish in Neknampur Lake in Hyderabad to water lake in India and Asia and second clean and purify the polluted waterbody. largest lagoon in the world. It is located Cleaning agents planted on FTW at mouth of Daya River, fl owing into Bay include vetivers, cattalis, canna, of Bengal. It was designated as the fi rst bulrush, citronella, hibiscus, fountain “Ramsar Site” of India. grass, fl owering herbs, tulsi and  Statement 3 is correct: A lagoon is a ashvagandha. body of water separated from larger bodies of water by a natural barrier. 21. Correct Option: (b)  Statement 4 is incorrect: The World Tourism Organization (UNWTO) is Explanation: the United Nations specialized agency  Statement 1 is correct: The Olive ridley responsible for the promotion of turtles are the smallest and most responsible, sustainable and universally abundant of all sea turtles found in the accessible tourism. UNWTO’s membership world, inhabiting warm waters of the includes 158 countries, 6 territories and Pacifi c, Atlantic and Indian oceans. over 500 affi liate members representing  the private sector, educational institutions, Statement 2 is correct: Their vulnerable tourism associations and local tourism status comes from the fact that they authorities. Its headquarters are located nest in a very small number of places, in Madrid, Spain. and therefore any disturbance to even one nest beach could have huge repercussions on the entire population. 20. Correct Option: (b)  Statement 3 is Incorrect: The Explanation: Government of Odisha is planning to declare Rushikulya rookery a Protected Area. Of the three mass nesting sites GS SCOREalong the Odisha coast – Gahirmatha, Rushikulya and the Devi River mouth, only Gahirmatha is legally protected.

22. Correct Option: (c) Explanation:  Statement 1 is correct: Mission  Innovation (MI) is a global initiative of 23 Statement 1 is correct: Floating countries and the European Commission treatment wetlands (FTWs) or islands (on behalf of the European Union) working are small artifi cial platforms that allow to reinvigorate and accelerate global clean aquatic emergent plants to grow. The energy innovation with the objective to use of wetlands for bioremediation to make clean energy widely aff ordable. capture and remove contaminants and India is a member of MI. Department

7 of Biotechnology also has established  Statement 3 is incorrect: Getting India Sustainable Biofuel Innovation Challenge to have at least 33% of its area under (SBIC) under MI. A two day international forest has been a long standing goal of conference on Sustainable Biofuels is the government since 1988. India State jointly being organized by Department of Forest Report 2017 says that India has of Biotechnology, Govt. of India and about 7,08,273 square kilometres of forest, Biofuture platform. which is 21.53% of the geographic area of the country (32,87,569 sq. km).  Statement 2 is incorrect: Ministry of Science and Technology is the nodal agency coordinating and steering the 24. Correct Option: (d) activities of Mission Innovation in India Explanation:  Statement 3 is correct: Generations of  Statement 1 is correct: National Biogas Biofuels: and Manure Management Programme  First Generation: First-generation is a Central Sector Scheme. Under Central biofuels rely on food crops as their sector schemes, it is 100% funded by the feedstock Union government and implemented by the Central Government machinery.  nd Second Generation: 2 generation Ministry of New and Renewable Energy biofuels come from non-food is implementing the National Biogas biomass. and Manure Management Programme  Third Generation: Third-generation (NBMMP) in all the States and UTs of the biofuels seek to improve the country. feedstock. Designing oilier crops, for  Statement 2 is correct: The scheme example, could greatly boost yield. which provides for setting up of Family  Fourth Generation: Fourth-generation Type Biogas Plants mainly for rural technology combines genetically and semi-urban households. A family optimized feedstocks, which are type biogas plant generates biogas from designed to capture large amounts organic substances such as cattle –dung, of carbon, with genomically and other bio-degradable materials such synthesized microbes, which are as biomass from farms, gardens, kitchens made to effi ciently make fuels. Key and night soil wastes etc. to the process is the capture and  Statement 3 is correct: The process of

sequestration of CO2, a process that biogas generation is called anaerobic renders fourth-generation biofuels a digestion (AD) Anaerobic digestion carbon negative source of fuel. is a series of biological processes in which microorganisms break down 23. Correct Option: (a) biodegradable material in the absence of oxygen. Explanation:  Statement 1 is incorrect: India is one 25. Correct Option: (c) of the few countries which have a forest Explanation: policy since 1894.GS The policy was revisedSCORE in 1952 and again in 1988. National  Statement 1 is correct: The WWDR is an Forest Policy, 1988 aims to ensure annual and thematic report that focuses environmental stability and maintenance on diff erent strategic water issues each of ecological balance. year. It aims to provide decision-makers  with the tools to implement sustainable use Statement 2 is correct: As per the Draft of our water resources. The development Policy, A National Board of Forestry of the WWDR is coordinated by the headed by the central minister in-charge World Water Assessment Programme of forests and State Boards of Forestry (WWAP), a joint eff ort of the UN headed by state minister in-charge of agencies and entities which make up forests will be established for ensuring UN-Water. inter-sectoral convergence, simplifi cation  of procedures, confl ict resolution and Statement 2 is incorrect: There periodic review. is no single UN entity dedicated

8 exclusively to water issues. Over 30 which were selected as national fi nalists in UN organizations carry out water and 2017-2018 edition of WWF’s One Planet sanitation programmes, refl ecting the fact City Challenge (OPCC). However, Rajkot that water issues run through all of the emerged as the National Winner in UN’s main focus areas. UN-Water’s role 2017-2018 edition of WWF’s One Planet is to coordinate so that the UN family City Challenge (OPCC). ‘delivers as one’ in response to water related challenges. 27. Correct Option: (a)  Statement 3 is correct: Explanation:  Important highlights of the Report:  Statement 1 is correct: Solar  Global demand for water has increased radiation management (SRM or solar six-fold over the past 100 years and geoengineering) is a theoretical approach continues to grow at the rate of 1% to reducing some of the impacts of climate each year. change by refl ecting a small amount of  Demand for water is projected to rise inbound sunlight back out into space. faster in developing countries. Some SRM techniques:  The report highlights that more than  Stratospheric aerosol injection - 5 billion people could suff er water When very large volcanoes erupt shortages by 2050. they blast millions of tonnes of refl ective sulphate particles into  The report thus emphasises the the stratosphere. These particles importance of nature-based circulate the planet on the powerful solutions. stratospheric winds, refl ecting away  Pollution has worsened the water a small amount of inbound sunlight bodies and water is expected to and cooling the planet for a year or deteriorate further in the coming two two. Stratospheric aerosol ejection decades. would seek to replicate this eff ect,  Water scarcity can lead to civil unrest, with aeroplanes or balloons used mass migration and even to confl ict to inject refl ective aerosol particles within and between countries. into the upper atmosphere.  Marine cloud brightening - Large 26. Correct Option: (a) areas of ocean are covered in marine stratus clouds. Scientists have Explanation: proposed that spraying tiny droplets of seawater into these clouds could make them lighter and more refl ective. Whiter, brighter clouds refl ect more sunlight back out into space, and would help cool the planet. GS SCORE Statement 2 is incorrect: It is in the early stages of research, but it is already a controversial topic. It is clear that SRM has the potential to be very helpful or  Statement 1 is correct: World Wide very damaging for those people and Fund for Nature (WWF) organizes One species most threatened by climate Planet City Challenge (OPCC) annually change, but it is very unclear what its to mobilize cities to deliver on the Paris Agreement. It supports and celebrate 100 full eff ects would be. cities with action plans for 1.5°C by 2020.  Statement 2 is incorrect: The black-and- 28. Correct Option: (c) white panda is the logo of World Wide Explanation: Fund for Nature.  Statement 1 is correct: Dust Storms are  Statement 3 is incorrect: Panaji, Pune, an ensemble of particles of dust or sand and Rajkot were the three Indian cities energetically lifted to great heights by

9 a strong and turbulent wind. The recent is an inter-governmental wildlife law dust storm which took lives of atleast 100 enforcement support body of South people, stretched from the western state Asian countries. It promotes regional of Rajasthan to the eastern state of cooperation to combat wildlife crime in Uttar Pradesh and hit Delhi, which lies South Asia. SAWEN is now a legitimate between them. intergovernmental organization with  Statement 2 is incorrect: A Downburst endorsement of the SAWEN Statute by is characterized by intense, downward fi ve countries namely Sri Lanka, India, air movements during thunderstorms. Nepal, Pakistan and Bangladesh. They can produce some of the strongest  ‘Wildlife’ means all fauna and fl ora. wind gusts of all weather patterns. As the downward air hits the ground, it  ‘Fauna’ are animals and birds, such moves outward and back up. While as tigers and falcons, but also include downbursts can span distances over fi sh. 200 miles, some are more concentrated,  ‘Flora’ are plants, such as orchids or impacting ground only a mile across. cacti, but also include timber and non-  Statement 3 is correct: The main timber forest products, some of which reason the these dust storms were so are illegally traded at very signifi cant catastrophic was because of the time levels when the strongest winds hit - at night,  as people were sleeping indoors. Most CITES (the Convention on of deaths were because of the collapse International Trade in Endangered of buildings and other structures. Species of Wild Fauna and Flora) – It is an international agreement between governments. Its aim is to ensure that 29. Correct Option: (d) international trade in specimens of wild Explanation: animals and plants does not threaten their survival.  Statement 1 is incorrect: Suva is an expert dialogue to explore a wide range  Statement 2 is correct: It members are of information, inputs and views on - Afghanistan, Bangladesh, Bhutan, ways for facilitating the mobilization and India, Maldives, Nepal, Pakistan and Sri securing of expertise, and enhancement Lanka (All the SAARC Nations). of support, including fi nance, technology  and capacity-building, for averting, Statement 3 is Correct: SAWEN operates minimizing and addressing loss and its activities from the Secretariat based in damage associated with the adverse Kathmandu, Nepal. eff ects of climate change.  Statement 2 is correct: It took place in 31. Correct Option: (b) Bonn, Germany. Explanation:  Statement 3 is correct: COP 23  requested the UN Secretariat, under the Statement 1 is incorrect: The National guidance of the Executive Committee of Institute for Transforming India (NITI) the Warsaw International Mechanism Aayog has developed the Composite for Loss andGS Damage (the Executive SCORE Water Management Index (CWMI) to Committee) and the Chair of the Subsidiary enable eff ective water management in Body for Implementation (SBI) to organize Indian states in the face of this growing such an Expert Dialogue. crisis.  Key Outcome - One of the outcomes  The CWMI is the fi rst comprehensive will be a table capturing the potential collection of country-wide water sources of support to avert, minimize, data in India. It is aimed at promoting and address loss and damage. competitiveness among states, driving them toward eff ective water 30. Correct Option: (d) governance  Statement 2 is correct: For the purposes Explanation: of analysis, the reporting states were  Statement 1 is incorrect: South Asia divided into two special groups – Wildlife Enforcement Network (SAWEN) ‘North Eastern and Himalayan states’

10 and ‘Other States’, to account for the made by the Union Home Ministry diff erent hydrological conditions across with support from the United Nations these groups. Development Programme (UNDP).  Statement 3 is incorrect: The report ranks  The index factors in exposure of Gujarat as number one in the reference population, agriculture and livestock year (2016-17), followed by Madhya and environmental risk in drawing up Pradesh, Andhra Pradesh, Karnataka the rankings. and Maharashtra. In North Eastern and  The disasters taken into consideration Himalayan States, Tripura has been include cyclones, earthquakes, adjudged number 1 in 2016-17 followed landslides, etc. by Himachal Pradesh, Sikkim and Assam.  A “disaster” is defi ned as per the  In terms of incremental change in specifi cations in Disaster Management index (over 2015-16 level), Rajasthan Act, 2005. Accordingly, a “disaster” holds number one position in means a catastrophe, mishap, general States and Tripura ranks at calamity or grave occurrence in any fi rst position amongst North Eastern area. and Himalayan States.  This could arise from natural or man-made causes, or by accident or 32. Correct Option: (b) negligence. Explanation:  Statement 2 is correct: Maharashtra  Statement 1 is incorrect: “Dead zone” is has been ranked at the top of the list a more common term for hypoxia, which of Indian states, vulnerable to natural refers to a reduced level of oxygen in the disasters, followed by West Bengal, Uttar water. Such a low oxygen concentration Pradesh, and Madhya Pradesh, according suff ocates the animal life. to the National Disaster Risk Index.  Hypoxic zones can occur naturally,  Statement 3 is incorrect: Among the but these areas are also created or Union Territories, Delhi is the most enhanced by human activity. vulnerable to such disasters.  Nutrient pollution is the primary cause of those zones created by humans. Excess nutrients that run off land or are piped as wastewater 34. Correct Option: (c) into rivers and coasts can stimulate Explanation: an overgrowth of algae, which then sinks and decomposes in the  Statement 1 is incorrect: World Bank has water. The decomposition process recently released South Asia’s Hotspots consumes oxygen and depletes the Report to assess the Impact of climate supply available to healthy marine life. change on Living Standards in South Asian Region.  One of the largest dead zones forms in the Gulf of Mexico every spring.  Statement 2 is incorrect: The report identifi es climate “hotspots” as defi ned  Statement 2GS is correct: Scientists SCORE geographical areas where living standards including those from India have found a will be most adversely impacted by huge ‘dead zone’ of an estimated 60,000 changes in average weather. square kilometres in Bay of Bengal.  Statement 3 is correct:  In recent times, warming of the atmosphere through climate change  The report looks at two scenarios is predicted to lead expansion of namely Climate-sensitive and ‘dead zones’ in the ocean. Carbon-intensive.  Climate-Sensitive - It represents 33. Correct Option: (b) a future “in which some collective action is taken to limit greenhouse gas Explanation: emissions and global annual average  Statement 1 is correct: National Disaster temperatures increase 2.4°C by 2100 Risk index is currently in a draft form relative to pre-industrial levels.

11  Carbon-Intensive - It represents a  Arsenicosis is the medical word for future in which no actions are taken arsenic poisoning, which occurs due to reduce emissions and global annual to accumulation of large amounts of average temperatures increase 4.3°C arsenic in the body. by 2100 relative to pre-industrial  Statement 2 is incorrect: The guideline levels. value or maximum contaminant level  If preventive measures are taken (MCL) for arsenic in drinking water is along the lines of the Paris Agreement, 10 ppb (according to WHO) followed India’s average annual temperatures are by most of the developed countries. In expected to rise by 1-2°C by 2050. developing countries including India and Bangladesh, 50 ppb is considered as  If no measures are taken, average the accepted level for arsenic in drinking temperatures in India are predicted to water. increase by 1.5-3°C by 2050.  Statement 3 is correct: Arsenic cannot be removed by boiling as it is NOT a volatile 35. Correct Option: (c) substance. Rather, its concentration increases as water evaporates during Explanation: boiling.  Statement 1 is correct: The Indian  “Volatile” refers to a substance that Meteorological Department IMD has vaporizes readily. commissioned two very high resolution (12 km grid scale) state-of-the-art global Ensemble Prediction Systems 37. Correct Option: (b) (EPS) for generating operational 10-days Explanation: probabilistic forecasts of weather.  Statement 1 is correct: Creation of  Statement 2 is correct: The EPS involves Earth’s Magnetic Field the generation of multiple forecasts using slightly varying initial conditions.  The probabilistic forecasts of severe weather events at 12 km grid scale across India will greatly help the disaster management.  Statement 3 is incorrect: India Meteorological Department IMD comes under the Ministry of Earth Sciences.

36. Correct Option: (d) Explanation:  The planet’s inner core is made of  Statement 1 is correct: Arsenic (As) is an solid iron. Surrounding the inner core odourless and tasteless metalloid widely is a molten outer core. distributed in theGS earth’s crust. It hasSCORE an  atomic number of 33. The Earth itself spins on its axis. The inner core spins as well, and it spins at  Arsenic occurs naturally in the a diff erent rate than the outer core. environment in both organic (arsenic  atoms bonded with carbon) and This creates a dynamo eff ect and this inorganic (no carbon) forms. is what creates the Earth’s magnetic fi eld.  Inorganic arsenic is the type  Statement 2 is correct: Shifting in associated with more adverse health Magnetic Poles of Earth eff ects for humans.   The magnetic fi eld varies over time as Arsenic can get into the human the Molten Liquid changes its spinning body through drinking water as speed. As the magnetic fi eld changes, well as eating food that has been the position o magnetic poles change contaminated with arsenic. accordingly.

12  Earth has settled in the last 20 million Epoch, which refl ects everything that years into a pattern of a pole reversal has happened over the past 11,700 about every 200,000 to 300,000 years. years  Statement 2 is correct: The Meghalayan  As the lava solidifi es, it creates a is unique because it is the fi rst interval record of the orientation of past in Earth’s geological history that magnetic fi elds much like a tape coincided with a major cultural event, recorder records sound. as agricultural societies struggled to  Statement 3 is incorrect: Impact of recover from the shift in climate. Polarity Reversal  The droughts over a 200-year period  This is proof that a polarity reversal resulted in human migrations in Egypt, would not aff ect the rotation axis of Greece, Syria, Palestine, Mesopotamia, Earth the Indus valley and the Yangtze river valley.  Fossil records from hundreds of past  magnetic polarity reversals; indicate Statement 3 is correct: About that polarity reversal would not lead International Union of Geological to any dramatic eff ects. Scienes (IUGS)   Reversals are a Rule, Not an The IUGS is an international non- exception governmental organization devoted to international cooperation in the fi eld of geology. 38. Correct Option: (c)  IUGS is a joint partner with UNESCO Explanation: for the International Geoscience Programme (IGCP) and they also participate in the Global Network of National Geoparks (GGN).

39. Correct Option: (c) Explanation:  Statement 1 is correct: Union Ministry of Earth Sciences has unveiled blueprint of Deep Ocean Mission (DOM). For this  Statement 1 is incorrect: Scientists have mission, Centre has drawn up a fi ve-year, created a new phase in Earth’s geological Rs. 8,000 crore plan to explore deep history and named it Meghalayan, after recesses of the ocean. a stalagmite from a cave in the Indian  The focus will be on technologies for state of Meghalaya that helped defi ne deep-sea mining, underwater vehicles, climatic events 4,200 years ago, marking underwater robotics and ocean climate the beginning ofGS the phase that continues SCORE change advisory services. till today.  Statement 2 is incorrect: An Exclusive  A stalagmite is an upward-growing Economic Zone or EEZ is a zone in the mound of mineral deposits that have sea approved by the UN Convention on precipitated from water dripping onto the Law of the Sea (UNCLOS). Over this the fl oor of a cave zone, a nation-state has exclusive special  A stalactite is an icicle-shaped rights for exploring and using the marine formation that hangs from the ceiling resources. It extends from the baseline of a cave. to two hundred nautical miles from the coast of the concerned country.  The Meghalayan Age began with a mega global drought that devastated  The Exclusive Economic Zone (EEZ) ancient agricultural civilisations from allotted to India (2.2 million sq.km) in Egypt to China. It is part of a longer the international waters will be covered period known as the Holocene under this mission.

13  It is to be noted that EEZ is approved world’s largest tropical peatlands, from by UNCLOS, but the right to explore unregulated land use and prevent its deep sea minerals like Poly- Metallic drainage and degradation. Nodules is granted by International  Statement 2 is incorrect: The Democratic Seabed Authority ISA. Republic of Congo (DRC), the Republic  Statement 3 is correct: Under this of Congo and Indonesia jointly signed mission, the key deliverables are: the Brazzaville declaration  Off shore desalination plant that will  Statement 3 is correct. The equivalent work with tidal energy, and of three years of global greenhouse gas  Developing a submersible vehicle emissions are stored in the Congo Basin, that can go to a depth of at least 6,000 emissions that could be released if the metres with three people on board. peatlands are degraded or the natural wetlands drained. 40. Correct Option: (d) 42. Correct Option: (b) Explanation: Explanation:  Statement 1 is incorrect: Complying  with the UNFCCC decisions on REDD+, Statement 1 is incorrect: A Heat India has prepared its National REDD+ Wave is a period of abnormally high Strategy. temperatures, more than the normal maximum temperature that occurs  The strategy report has been prepared during the summer season in the by Indian Council of Forestry North-Western parts of India. Heat Research & Education (ICFRE). Waves typically occur between March  The Indian Council of Forestry and June, and in some rare cases even Research and Education (ICFRE) is an extend till July. autonomous organisation under the  Statement 2 is correct: The Indian Ministry of Environment and Forests Meteorological Department (IMD) and Climate Change, Government of has given the following criteria for India. Headquartered in Dehradun. Heat Waves : Heat Wave need not be  Statement 2 is incorrect: REDD i.e considered till maximum temperature of “Reducing emissions from deforestation a station reaches atleast 40°C for Plains and forest degradation” was fi rst and atleast 30°C for Hilly regions. introduced in UNFCCC as a climate change mitigation option. 43. Correct Option: (b)  With India’s intervention for inclusion Explanation: of policy approach of conservation and sustainable management of forests,  Statement 1 is correct: Methane hydrate, the concept of “forest conservation, also known as “gas hydrate” or “fl ammable sustainable management of forests and ice,” is an ice-like substance composed enhancement GSof forest carbon stocks SCORE in of methane gas and water. These are developing countries” was added and trapped beneath permafrost and ocean the concept is now collectively referred sediment where low temperature and as ‘REDD+’. moderate pressure combine to trap methane in this specifi c way. 41. Correct Option: (c)  Statement 2 is correct: The methane hydrate is highly fl ammable and energy- Explanation: intensive fuel as one cubic metre of the  Statement 1 is incorrect: The Brazzaville compound can releases about 160 cubic Declaration is signed to boost the metres of gas. It can break down into coordination eff orts to protect the benefi ts water and methane after temperature is provided by peatland ecosystems. It is raised or pressure is lowered. It is likely aimed to protect the Cuvette Centrale to be the world’s last great source of region in the Congo Basin, the carbon-based fuel.

14  Statement 3 is incorrect: China has pressure system in tropics when the successfully produced natural gas central pressure falls by 5 to 6 hPa from from methane hydrate, also known as the surrounding and maximum sustained “fl ammable ice”, in an experimental wind speed reaches 34 knots (about 62 project in the South China Sea (SCS). kmph). Cyclone Mora Formation - A India, Canada and US are also believed to deep depression in the Bay of Bengal be looking at hydrates as an alternative intensifi ed into a tropical cyclone named energy source Mora. Life period of a tropical cyclone over the north Indian Ocean is 5-6 days 44. Correct Option: (b) Naming of a Cyclone Explanation:  World Meteorological Organization (WMO) maintains rotating lists of names  Statement 1 is incorrect: In a normal which are appropriate for each Tropical year, a surface low pressure develops Cyclone basin. in the region of northern Australia and Indonesia and a high pressure system  There is a strict procedure to determine a over the coast of Peru. As a result, the list of tropical cyclone names in an ocean trade winds over the Pacifi c Ocean move basin(s) by the Tropical Cyclone Regional strongly from east to west. Body responsible for that basin(s) at its annual/biennial meeting.  The easterly fl ow of the trade winds carries warm surface waters westward & along  For the Indian Ocean region, the coast of Peru, cold bottom nutrient deliberations for naming cyclones began rich water wells up to the surface to in 2000 and a formula was agreed upon replace the warm water that is pulled to in 2004. Eight countries in the region - the west. Bangladesh, India, Maldives, Myanmar,  Oman, Pakistan, Sri Lanka and Thailand In an El Niño year, the normal low - all contributed a set of names which are pressure system is replaced by a weak assigned sequentially whenever a cyclonic high in the western Pacifi c (the southern storm develops. oscillation). This change in pressure pattern causes the trade winds to be reduced = Weak Walker Cell. Sometimes 46. Correct Option: (d) Walker Cell might even get reversed. Explanation:  This reduction allows the equatorial  Statement 1 is incorrect: In order to counter current (current along doldrums) protect the rich aquatic biodiversity of river to accumulate warm ocean water along Ganga from escalating anthropogenic the coastlines of Peru and Ecuador pressures, development of a Turtle  This accumulation of warm water causes sanctuary in Allahabad along with a the thermocline to drop in the eastern River Biodiversity Park at Sangam have part of Pacifi c Ocean which cuts off the been approved under Namami Gange upwelling of cold deep ocean water along programme. the coast of Peru  Statement 2 in incorrect: National  Statement 2 GS is correct. The warmerSCORE Mission for Clean Ganga(NMCG) acts as waters had a devastating eff ect on implementation arm of National Ganga marine life existing off the coast of Peru River Basin Authority(NGRBA) which and Ecuador. Severe droughts occur in was constituted under the provisions Australia, Indonesia, India and southern of the Environment (Protection) Act Africa. The most prominent droughts in (EPA),1986 India – six of them – since 1871 have been  The aims and objectives of NMCG is El Nino droughts, including the recent to accomplish the mandate of National ones in 2002 and 2009 Ganga River Basin Authority (NGRBA) of  To ensure eff ective abatement of 45. Correct Option: (d) pollution and rejuvenation of the Explanation: river Ganga by adopting a river basin approach to promote inter-sectoral  All the statements are correct: A co-ordination for comprehensive tropical cyclone (TC) is a rotational low- planning and management

15  To maintain minimum ecological Organic Pollutants (POPs) is a global fl ows in the river Ganga with treaty to protect human health and the the aim of ensuring water quality environment from highly dangerous, and environmentally sustainable long-lasting chemicals by restricting and development. ultimately eliminating their production, use, trade, release and storage. Rotterdam 47. Correct Option: (d) Convention was signed to promote shared responsibility and cooperative Explanation: eff orts among Parties in the international  Statement 1 is incorrect: UN Water trade of certain hazardous chemicals in released the annual World Water order to protect human health and the Development Report 2018, titled ‘Nature- environment from potential harm Based Solutions for Water’. It was launched 19 March 2018 during the 8th 49. Correct Option: (b) World Water Forum, and in conjunction to the World Water Day, demonstrates Explanation: how nature-based solutions (NBS)  Statement 1 is incorrect: The Eighth off er a vital means of moving beyond Regional 3R Forum in Asia and Pacifi c business-as-usual to address many was held in Indore, Madhya Pradesh. of the world’s water challenges while The forum provides strategic platform simultaneously delivering additional for sharing best practices in 3R areas, benefi ts vital to all aspects of sustainable including new and emerging issues development. of concern in waste management. Its  Statement 2 is incorrect: There members include governments, aid is no single UN entity dedicated agencies, international agencies, exclusively to water issues. Over 30 private sector entities, research bodies, UN organizations carry out water and NGOs and other relevant parties. sanitation programmes, refl ecting the fact  Statement 2 is correct: It was co- that water issues run through all of the organized by Ministry of Housing and UN’s main focus areas. UN-Water’s role Aff airs, Japan’s Environment Ministry, is to coordinate so that the UN family and United Nations Centre for Regional ‘delivers as one’ in response to water Development (UNCRD). related challenges  Statement 3 is correct: The Regional 3R 48. Correct Option: (b) Forum in Asia and Pacifi c was launched in 2009 in Tokyo, Japan with objective Explanation: of integrating 3Rs– reduce, reuse  Statement 1 is correct: Persistent and recycle in policy, planning and organic pollutants (POPs) are organic development. chemical substances that are recognized as a serious, GS global threat to humanSCORE 50. Correct Option: (d) health and to ecosystems. They remain intact for exceptionally long periods of Explanation: time (many years) and are toxic to both  Statement 1 is correct: The Government humans and wildlife. of India hosted 90+ delegations including  Statement 2 is correct: Biomagnifi cation 50 energy ministers, 30 CEOs, 12 heads refers to the tendency of pollutants to of international organisations at the concentrate as they move from one trophic 16th biennial International Energy level to the next. In bioaccumulation Forum Ministerial Meeting (IEF16) in there is an increase in concentration of New Delhi on 10-12 April 2018 with the a pollutant from the environment to the support of the People’s Republic of China fi rst organism in a food chain. and the Republic of Korea as co-hosts.  Statement 3 is incorrect: The  Statement 2 is incorrect: The meeting Stockholm Convention on Persistent was held under the theme: “The Future

16 of Global Energy Security - Transition,  Statement 3 is correct: The Secretariat Technology, Trade and Investment”. is the administrative organ of the IEF. Providing the opportunity for attendees Governed by a Secretary General to discuss relevant issues that currently responsible to the Executive Board of the aff ect global energy markets, the IEF Forum the Secretariat is a permanent Ministerial is the largest gathering of body. Based in the Diplomatic Quarter energy ministers in the world. of Riyadh, Saudi Arabia the Secretariat has been headquartered there since its inception in December 2003.

**********

GS SCORE

17 59

TARGET PT - 2019

CURRENT AFFAIRS (SCIENCE & TECHNOLOGY)

Time Allowed: 1 Hrs. Maximum Marks: 100

Roll No.:

INSTRUCTIONS

1. IMMEDIATELY AFTER THE COMMENCEMENT OF THE EXAMINATION, YOU SHOULD CHECK THAT THIS TEST BOOKLET DOES NOT HAVE ANY UNPRINTED OR TORN OR MISSING PAGES OR ITEMS, ETC. IF SO, GET IT REPLACED BY A COMPLETE TEST BOOKLET. 2. Please note that it is the candidate’s responsibility to encode and fi ll in the Roll Number carefully and without any omission or discrepancy at the appropriate places in the OMR Answer Sheet. Any omission/ discrepancy will render the Answer Sheet liable for rejection. 3. You have to enter your Roll Number on the Test Booklet in the Box provided alongside. DO NOT write anything else on the Test Booklet. 4. While writing name and Roll No. on the top of the OMR Sheet in appropriate boxes use “ONLY BLACK BALL POINT PEN”. 5. This Test Booklet contains 50 items (questions). Each item comprises four responses (answers). You will select the response which you want to mark on the Answer Sheet. In case you feel that there is more than one correct response, mark the response which you consider the best. In any case, choose ONLY ONE response for each item. 6. You have to mark all your responses ONLY on the separate OMR Answer Sheet provided. 7. All items carry equal marks. (2 marks each) 8. After you have completed fi lling in all your responses on the Answer Sheet and the examination has concluded, you should hand over to the Invigilator only the Answer Sheet. You are permitted to take away with you the Test Booklet. 9. Sheets for rough work areGS appended in the TestSCORE Booklet at the end. 10. Penalty for wrong answers: THERE WILL BE PENALTY FOR WRONG ANSWERS MARKED BY A CANDIDATE IN THE OBJECTIVE TYPE QUESTION PAPERS. (a) There are four alternatives for the answer to every question. For each question for which a wrong answer has been given by the candidate, one-third of the marks assigned to that question will be deducted as penalty. (b) If a candidate gives more than one answer, it will be treated as a wrong answer even if one of the given answers happens to be correct and there will be same penalty as above to that question. (c) If a question is left blank, i.e., no answer is given by the candidate, there will be no penalty for that question.

DO NOT OPEN THIS TEST BOOKLET UNTIL YOU ARE ASKED TO DO SO

1 TPT2019/B-1/012019/59 1. Consider the following statements 4. Which of the following statements regarding Fixed Dose Combination about Apsara Nuclear reactor is/are Drugs (FDCs): correct? 1. FDCs contain two or more active 1. It is India’s indigenous cryogenic research pharmaceutical ingredients, distributed reactor. in variable doses and targeting multiple 2. The reactor burns enriched uranium in the diseases. form of aluminium alloyed curved plates 2. FDCs are useful for chronic conditions Select the correct answer using the code given especially, when multiple disorders co- below: exist. (a) 1 only Which of the above statements is/are correct? (b) 2 only (a) 1 only (c) Both 1 and 2 (b) 2 only (d) Neither 1 nor 2 (c) Both 1 and 2 5. Recently Anti-Nuclear Medical kit (d) Neither 1 nor 2 was in news, consider the following statements about it: 2. With reference to Higgs Boson, 1. It is India’s fi rst medical kit for protection consider the following statements: against nuclear warfare or radioactive 1. The Higgs boson is an elementary particle leakage, produced recently in a joint collaboration with Russia. produced by the quantum excitation of the Higgs fi eld. 2. It can protect, but cannot heal the radiation damage on the skin 2. It was the last hold-out particle remaining hidden during the quest to check the Which of the above statements is/are accuracy of the Standard Model of incorrect? Physics. (a) 1 only Which of the above statements is/are correct? (b) 2 only (a) 1 only (c) Both 1 and 2 (b) 2 only (d) Neither 1 nor 2 (c) Both 1 and 2 6. What is iLint ? (d) Neither 1 nor 2 (a) It is the world’s fi rst hydrogen-powered passenger train 3. Which of the following statements is/ (b) It is a robot manufactured on the lines of are correct about NASA’s ICESat-2? Artifi cial Intelligence 1. It aims to measureGS the width of icy particlesSCORE (c) It the new prototype of Internet of Things on the planets in the Kuiper belt by Google 2. Researchers are going to use the (d) It is the world’s largest Fly-ash plant. information to study sea level rise resulting from melting ice sheets and glaciers, and 7. With reference to Dickinsonia, to improve sea ice and climate forecasts consider the following statements: Select the correct answer using the code given 1. It is the earliest plant on geological below: record. (a) 1 only 2. It has been found in Chotanagpur region. (b) 2 only Which of the above statements is/are correct? (c) Both 1 and 2 (a) 1 only (d) Neither 1 nor 2 (b) 2 only

2 (c) Both 1 and 2 11. Recently Cyclone-30 India’s biggest (d) Neither 1 nor 2 cyclotron facility becomes operational. It will be: 8. Consider the following statements 1. Used to predict cyclones through a range of sensors and networks regarding Astra Missile, tested 2. The fi rst and only cyclotron facility in India recently: to produce Germanium 68 radioisotope 1. It is beyond visual range Surface to Air Which of the above statements is/are correct? Missile. (a) 1 only 2. It has a range of 100-150 kms. (b) 2 only Which of the above statements is/are correct? (c) Both 1 and 2 (a) 1 only (d) Neither 1 nor 2 (b) 2 only (c) Both 1 and 2 12. Consider the following statements (d) Neither 1 nor 2 regarding Facial Recognition technology: 9. With reference to HOPE Probe, 1. It allows the passengers to move across diff erent sections of the airport using consider the following statements: automatic biometric boarding procedure 1. It is a Chinese space mission aimed at instead of manual boarding passes learning about life on other planets in verifi cation. post-Milky Way galaxy 2. India has the fi rst end-to-end face 2. It works on the principle of echo, used in recognition-based walk through experience in Asia and largest in the SONARS. world. Which of the above statements is/are correct? Which of the above statements is/are correct? (a) 1 only (a) 1 only (b) 2 only (b) 2 only (c) Both 1 and 2 (c) Both 1 and 2 (d) Neither 1 nor 2 (d) Neither 1 nor 2

10. Recently, researchers from University 13. Which of the following is the correct of Melbourne, Australian have description of the term EyeROV discovered three variants of multidrug- TUNA? resistant superbug in bacteria known (a) It is an underwater robotic drone as Staphylococcus epidermidis, (b) It is an indigenously developed oil spill that can cause GS severe infections SCORE or cleaner even death. Consider the following (c) It is a hybrid variety of vitamin rich fi sh statements about this superbug: found recently in Eastern coast of India 1. It infects the children and infants mostly. (d) It is an underwater observatory. 2. It is found naturally on the skin of monkeys. 14. Consider the following statements Which of the above statements is/are correct? regarding India’s supercomputer named Pratyush: (a) 1 only 1. It will improve rankings of Indian (b) 2 only supercomputer from 300s to 30s in Top 500 list globally. (c) Both 1 and 2 2. It works under the Ministry of Electronics (d) Neither 1 nor 2 and Information Technology

3 Which of the above statements is/are 18. Consider the following statements incorrect? about Scorpene Submarine: (a) 1 only 1. It is a diesel-electric attack submarines equipped with anti-ship missile (b) 2 only 2. The attacks can be carried out with (c) Both 1 and 2 torpedoes both while submerged or on (d) Neither 1 nor 2 surface — in all war theatres, including the tropics. 15. With reference to National Viral Which of the above statements is/are correct? Hepatitis Control Programme, (a) 1 only consider the following statements: (b) 2 only 1. It will include all viral strains of Hepatitis (c) Both 1 and 2 excluding Hepatitis E. (d) Neither 1 nor 2 2. It aims to treat minimum of 3 lakh hepatitis cases over a period of three years. 19. Which of the following statements Which of the above statements is/are correct? about INS Chakra is/are correct? (a) 1 only 1. It is part of Indo-Russia collaboration under the Project 75 (b) 2 only 2. It is the fi rst Akula class submarine (c) Both 1 and 2 Select the correct answer using the code given (d) Neither 1 nor 2 below: (a) 1 only 16. Consider the following achievements (b) 2 only of Prof. SN Bose: (c) Both 1 and 2 1. He never received a doctorate nor was he (d) Neither 1 nor 2 awarded a Nobel Prize 2. The God Particle is named in his honour 20. Consider the following statements Which of the above statements is/are correct? about Down Syndrome: (a) 1 only 1. It is a genetic disorder (b) 2 only 2. It impacts neurological functions causing impairment of intellectual functions (c) Both 1 and 2 Which of the above statements is/are correct? (d) Neither 1 nor 2 (a) 1 only 17. Which of the following statements (b) 2 only about Mass EmbryoGS Transfer is/areSCORE (c) Both 1 and 2 correct? (d) Neither 1 nor 2 1. Farmer can get 5-6 fold increase in number of off -springs 21. With reference to Raman Eff ect, 2. Off -springs born will be free from consider the following statements: diseases 1. It is a change in the wavelength of light that occurs when a light beam is defl ected Select the correct answer using the code given by molecules. below: 2. The discovery was awarded with the (a) 1 only Nobel Prize in Physics. (b) 2 only Which of the above statements is/are correct? (c) Both 1 and 2 (a) 1 only (d) Neither 1 nor 2 (b) 2 only

4 (c) Both 1 and 2 25. With reference to the Nerve Agents, (d) Neither 1 nor 2 consider the following statements: 1. They are among the most lethal form of chemical weapons. 22. With reference to Machine to Machine (M2M) Communication, consider the 2. Novichok is a type of Nerve Agent. following statements: Which of the above statements is/are correct? 1. Unlike Person-to-Person (P2P) (a) 1 only communication, M2M technologies (b) 2 only involve communication between two devices. (c) Both 1 and 2 2. It enables only wireless devices to talk to (d) Neither 1 nor 2 each other using sensors. 26. Consider the following statements Which of the above statements is/are correct? about Tuberculosis in India: (a) 1 only 1. India has set the target to eliminate (b) 2 only Tuberculosis by 2030. (c) Both 1 and 2 2. Elimination of a disease is defi ned by the World Health Organisation (WHO) as (d) Neither 1 nor 2 there should be less than 1 case of TB for a population of a million people. 23. Which of the following statements Which of the above statements is/are about MILAN Exercise are correct? incorrect? 1. It is congregation of littoral navies (a) 1 only conducted biennially by Indian Navy. (b) 2 only 2. It was recently started under aegis of the (c) Both 1 and 2 Andaman and Nicobar Command. (d) Neither 1 nor 2 3. In total, 16 countries participated in MILAN 2018. 27. With reference to Saposhi, consider Select the correct answer using the code given the following statements: below: 1. It is a new malware that can take over (a) 1 and 2 only electronic devices and use them for Distributed Denial of Service (DDoS) (b) 2 and 3 only attacks. (c) 1 and 3 only 2. Computer Emergency Response Team (d) 1, 2 and 3 (CERT) is a Central government body that deals with cyber attack in India. Which of the above statements is/are correct? 24. Consider the following statements about recently GS inaugurated Defence SCORE (a) 1 only Industrial Corridor in Tamil Nadu: (b) 2 only 1. Tamil Nadu Defence Industrial Corridor (c) Both 1 and 2 would be a specialised facility of aero (d) Neither 1 nor 2 component manufacturing. 2. It is the country’s fi rst defence industrial 28. The Shakti Sthala Solar park, one of corridor the world’s largest solar park is setup Which of the above statements is/are correct? in which of the following states? (a) 1 only (a) Andhra Pradesh (b) 2 only (b) Karnataka (c) Both 1 and 2 (c) Tamil Nadu (d) Neither 1 nor 2 (d) Kerala

5 29. An international team of physicists 1. The disease is mainly prevalent in South have successfully created a “giant Asian countries. atom” and fi lled it with ordinary 2. It’s a zoonotic disease which transmits to atoms, creating a new state of matter human from Monkeys. termed “Rydberg polarons”. Consider Which of the above statements is/are correct? the following statements: (a) 1 only 1. These atoms are held together by a (b) 2 only strong bond and is created at very hot temperatures. (c) Both 1 and 2 2. It is an atom in which an electron has been (d) Neither 1 nor 2 kicked out to a very large orbit. Which of the above statements is/are correct? 34. Start-up States and UTs Ranking (a) 1 only Framework is : 1. Launched by Department of Industrial (b) 2 only Policy and Promotion (DIPP). (c) Both 1 and 2 2. Aiming to foster competitiveness and (d) Neither 1 nor 2 propel states/UTS to act pro-actively in start-up matters. 30. Recently, the annual Lyrid meteor 3. Based on feedback collected from Start- up ecosystem stakeholders shower is set to reach a peak. Consider the following statements about it: Which of the above statements is/are correct? 1. It is one of the oldest known meteor (a) 1 and 2 only showers. (b) 3 only 2. It is classifi ed as a high-strength shower. (c) 1 and 3 only Which of the above statements is/are correct? (d) 1, 2 and 3 (a) 1 only (b) 2 only 35. With reference to Nipah Virus, consider the following statements: (c) Both 1 and 2 1. It is a zoonotic disease that causes severe (d) Neither 1 nor 2 disease in humans only. 2. It was fi rst identifi ed in Malaysia. 31. Recently, fi rst Radio 5G Lab was 3. The virus can be transmitted through air. established in ? Which of the above statements are incorrect? (a) ISRO, Bengaluru (a) 1 and 2 only (b) IISc, Bengaluru (b) 2 and 3 only (c) CSIR, Delhi GS SCORE(c) 1 and 3 only (d) IIT, Delhi (d) 1, 2 and 3 32. What is Gravity Rat? 36. Consider the following statements (a) Malware about GRACE Mission: (b) Space Mission 1. It is a mission to map Earth’s water and ice (c) Extinct Spicy by measuring changes in Earth’s gravity fi eld. (d) None 2. It is launched by ISRO.

33. With reference to the disease Which of the above statements is/are correct? Lassa Fever, consider the following (a) 1 only statements: (b) 2 only

6 (c) Both 1 and 2 (c) Both 1 and 2 (d) Neither 1 nor 2 (d) Neither 1 nor 2

37. Which of the following statements 41. Recently, Indian IT industry’s apex about quadrivalent vaccine is/are body National Association of Software incorrect? and Services Companies (NASSCOM) 1. The vast majority of infl uenza vaccines has launched its Centre of Excellence manufactured were quadrivalent till for the Internet of Things (CoE – IoT). recently Consider the following statements 2. In India, the vaccine will be available as a about it: single dose pre-fi lled syringe 1. It aims to act as perfect collaboration for Select the correct answer using the code given innovation and high-end technologies below: 2. It is inaugrated at Gurugram, Haryana. (a) 1 only 3. It is a part of nationwide collaboration (b) 2 only involving the Ministry of Electronics and Information Technology (MeitY). (c) Both 1 and 2 Which of the above statements is/are correct? (d) Neither 1 nor 2 (a) 1 only 38. Recently KATRIN Experiment was in (b) 1 and 2 only news. It’s objective is to: (c) 2 and 3 only (a) Measure the mass of Neutrino. (d) 1, 2 and 3 (b) Develop a Drug to treat Ebola. (c) Measure Earthquake Waves. 42. Consider the following statements about World Intellectual Property (d) Study the Big-bang phenomena. Organisation (WIPO): 1. It is the global body for promotion and 39. Pinaka Rocket is : protection of Intellectual Property Rights 1. Developed by DRDO. (IPR). 2. Fired using multi-barrel rocket launcher 2. It is the specialized agencies of United Which of the above statements is/are Nations (UN). incorrect? 3. It is headquartered in Geneva, (a) 1 only Switzerland. 4. It prescribes mandatory measures to the (b) 2 only member countries. (c) Both 1 and 2 Which of the above statements are correct? (d) Neither 1 nor 2 (a) 1 and 4 only GS SCORE(b) 1, 2 and 4 only 40. Recently, Scientists from New Zealand have performed world’s fi rst-ever (c) 1, 2 and 3 only 3-D, colour X-ray on human. With (d) 1, 2, 3 and 4 reference to it, consider the following statements: 43. Consider the following statements 1. The new device is based on the traditional about 5G technology: black-and-white X-ray 1. It is a wireless communication technology 2. It gives high-resolution and high-contrast based on third-generation partnership pictures. project Which of the above statements is/are correct? 2. The 5G wireless technology will off er far (a) 1 only greater upload and download speed i.e. 100 times faster internet data speed than (b) 2 only current 4G networks.

7 Which of the above statements is/are correct? 48. Consider the following statements (a) 1 only about National Institute of Food (b) 2 only Technology Entrepreneurship and Management (NIFTEM): (c) Both 1 and 2 1. It is the brainchild of Ministry of Agriculture (d) Neither 1 nor 2 & Farmers Welfare. 2. NIFTEM will work actively for assisting in 44. Which of the following statements setting up food standards. about Ballistic Missile Interceptor Advanced Area Defence (AAD) is/are 3. It would also be an apex institution correct? in the fi eld of food technology and management, 1. It is recently tested by Bhabha Atomic Research Centre. Which of the above statements is/are 2. It is a single-stage, solid-fuelled missile. incorrect? Select the correct answer using the code given (a) 1 only below: (b) 3 only (a) 1 only (c) 1 and 2 only (b) 2 only (d) 2 and 3 only (c) Both 1 and 2 (d) Neither 1 nor 2 49. With reference to Two-Factor Authentication (2FA), consider the 45. What is Uakitite, recently in news? following statements: (a) New geometrical shape discovered. 1. Two-factor authentication methods rely (b) A new mineral discovered. on users providing a password as well as a second factor as passcode. (c) Russian made Drone. 2. It adds an additional layer of security to (d) Cancer healing Drug the authentication process by making it harder for attackers to gain access to a 46. What is Scutoid? person’s devices. (a) Newly discovered geometrical shape. Which of the above statements is/are correct? (b) An asteroid seen crossing the Earth. (a) 1 only (c) A new body organ discovered. (b) 2 only (d) Artifi cial Intelligence News Anchor. (c) Both 1 and 2 (d) Neither 1 nor 2 47. Recently, in which of the following state India’s fi rst geneticGS bank for wildlife SCORE conservation was inaugurated? 50. What is “P null”? (a) Rajasthan (a) Virus (b) Telangana (b) Blood Group (c) Maharashtra (c) Cyclone (d) Haryana (d) Asteroid

**********

8 59

TARGET PT - 2019

CURRENT AFFAIRS (SCIENCE & TECHNOLOGY)

Answer Key

Q. 1 (c) Q. 14 (b) Q. 27 (c) Q. 40 (c)

Q. 2 (c) Q. 15 (b) Q. 28 (b) Q. 41 (d)

Q. 3 (b) Q. 16 (c) Q. 29 (b) Q. 42 (c)

Q. 4 (b) Q. 17 (c) Q. 30 (a) Q. 43 (c)

Q. 5 (c) Q. 18 (c) Q. 31 (d) Q. 44 (b)

Q. 6 (a) Q. 19 (b) Q. 32 (a) Q. 45 (b)

Q. 7 (d) Q. 20 (b) Q. 33 (d) Q. 46 (a)

Q. 8 (d) GSQ. 21 (c) SCOREQ. 34 (d) Q. 47 (b)

Q. 9 (d) Q. 22 (a) Q. 35 (c) Q. 48 (a)

Q. 10 (d) Q. 23 (c) Q. 36 (a) Q. 49 (b)

Q. 11 (b) Q. 24 (a) Q. 37 (a) Q. 50 (b)

Q. 12 (c) Q. 25 (c) Q. 38 (a)

Q. 13 (a) Q. 26 (a) Q. 39 (a)

DO NOT OPEN THIS TEST BOOKLET UNTIL YOU ARE ASKED TO DO SO

1 TPT2019/B-1/012019/59 1. Correct Option: (c)  They are smaller than an atom and the wavelength of visible light, so the only Explanation: way we can detect and observe their Fixed Dose Combination Drugs (FDC) behaviour is by smashing the atomic  nucleus of particles together at intense Statement 1 is correct: An FDC is a speeds (close to the speed of light), which cocktail of two or more active drug generates vast amounts of exotic particles ingredients in a fi xed ratio of doses. that are only created at high energies.  According to US healthcare provider IMS  These collisions resemble the conditions Health, almost half the drugs sold in physicists believe existed during the time India in 2014 were FDC, making it a of the big bang. world leader in combination drugs.  Thanks to particle accelerators like the  FDCs’ popularity in India is due to Large Hadron Collider, the Relativistic advantages such as increased effi cacy, Heavy Ion Collider and the (now defunct) better compliance, reduced cost and Tevatron circular particle accelerator, simpler logistics of distribution. physicists have made a lot of progress in  FDCs have shown to be particularly useful designing a “theory of everything.” in the treatment of infectious diseases  This theory postulates how all the like HIV, malaria and tuberculosis, where subatomic particles in the universe giving multiple antimicrobial agents is the operates and how they interact to norm. comprise the Universe as we know it.  Statement 2 is correct: FDCs are also  One of the most complete models that useful for chronic conditions especially, comes anywhere near producing a “theory when multiple disorders co-exist. of everything,” is the Standard Model of In News: Fundamental Particles and Interactions, which describes how particles and forces  According to a study, of the 110 anti-TB interact. (tuberculosis) Fixed Dose Combinations  (FDCs) available in India, only 32 (less than The standard model also includes an 30%) have been approved by the Central explanation for 3 of the 4 fundamental Drugs Standard Control Organisation forces of nature on a subatomic scale. (CDSCO), the country’s drug regulator. 3. Correct Option: (b) 2. Correct Option: (c) Explanation: Explanation: NASA’s ICESat-2 Higgs Boson  Statement 1 is incorrect and 2 is  Both the statements are correct: The correct: NASA’s ICESat-2 has mapped Higgs boson is an elementary particle in melting ice sheets in Antarctica and the the Standard Model of particle physics, resulting sea level rise across the globe, produced by the quantum excitation of which could help improve climate GS SCOREforecasts. the Higgs fi eld, one of the fi elds in particle physics theory.  The satellite is measuring the height  It is named after physicist Peter Higgs, of sea ice to within an inch, tracing the who in 1964, along with six other terrain of previously unmapped Antarctic scientists, proposed the mechanism, valleys, surveying remote ice sheets, and which suggested the existence of such a peering through forest canopies and shallow coastal waters. particle.  With each pass of the ICESat-2 satellite,  Its existence was confi rmed by the the mission is adding to datasets tracking ATLAS and CMS collaborations based on Earth’s rapidly changing ice. Researchers collisions in the LHC at CERN. are ready to use the information to study  In the world of particle physics, subatomic sea level rise resulting from melting ice particles are diffi cult to observe because sheets and glaciers, and to improve sea of their size. ice and climate forecasts.

2  As the name suggests, ICESat-2 is a  By virtue of higher neutron fl ux, this reactor follow-on project. The original spacecraft will increase indigenous production or fl ew in the 2000s and pioneered the radio-isotopes for medical application laser measurement of the height of polar by about fi fty percent and would also be glaciers and sea-ice from space. But extensively used for research in nuclear the mission was plagued by technical physics, material science and radiation problems that limited its observations to shielding. just a couple of months in every year.  This development has re-emphasised  ICESat-2 will measure the average annual the capability of Indian scientists and elevation change of land ice covering Engineers to build, complex facilities Greenland and Antarctica to within for health care, science education and the width of a pencil, capturing 60,000 research. measurements every second. In News:  ICESat-2’s Advanced Topographic Laser  The Bhabha Atomic Research Centre has Altimeter System (ATLAS) measures height recommissioned an upgraded version of by timing how long it takes individual light ‘Apsara’, the country’s oldest research photons to travel from the spacecraft to reactor that was decommissioned almost Earth and back. a decade ago. Apsara is now operational as ‘Apsara-U’ on Trombay campus of Maharashtra. 4. Correct Option: (b) Explanation: 5. Correct Option: (c) Apsara Nuclear reactor Explanation:  Statement 1 is incorrect: Apsara is the Anti-nuclear medical kit oldest of India’s research reactors.  Statement 1 is incorrect: Institute of  The reactor was designed by the Bhabha Nuclear Medicine and Allied Sciences Atomic Research Center (BARC) and built (INMAS) has developed India’s fi rst with assistance from the United Kingdom indigenous medical kit for protection (which also provided the initial fuel supply against nuclear warfare or radioactive consisting of 80% enriched uranium). leakage.  Apsara fi rst went critical on 4 August  The kit will ensure protection from serious 1956. injury and aid faster healing of wounds due to nuclear warfare or radioactive  Apsara is a light water swimming pool- leakage. type reactor with a maximum power output of one megawatt thermal (MWt).  Statement 2 is incorrect: It protects and heals the radiation damage on the skin.  Statement 2 is correct: The reactor burns enriched uranium in the form of  The kit has been developed after 20 years aluminium alloyed curved plates. Fuel of work by INMAS scientists. for the reactor is supplied under contract  It has 25 items which include radioactive from the UnitedGS Kingdom, provided SCORE that protectors which can absorb 80-90% of the fuel is safeguarded. radiation, nerve gas agents, bandages  The Apsara reactor is utilized for various that absorb radiation as well as tablets experiments including neutron activation and ointments. analysis, radiation damage studies,  The kit is seen as potent alternative to forensic research, neutron radiography, similar imported kits that were till now and shielding experiments. procured from US and Russia at much higher prices.  The reactor is also used for research and the production of radioisotopes.  It has been developed for armed, paramilitary and police forces only as Apsara- Upgraded: they are fi rst ones likely to get exposed  “Apsara-upgraded”, made indigenously, to radiation during nuclear, chemical uses plate type dispersion fuel elements and biomedical (NCB) warfare or rescue made of Low Enriched Uranium (LEU). operation after nuclear accident.

3 6. Correct Option: (a) running along its body that lived on Earth 558 million years ago. Explanation:  The fossils of this animal were recently iLint discovered.  Option (a) is correct: Germany has rolled  Statement 2 is incorrect: Researchers out world’s fi rst hydrogen-powered found the fossil in a remote area near the passenger train. White Sea in the northwest of Russia.  These locomotives named iLint trains  Dickinsonia grew up to 1.4 metres in emit zero emissions, making them eco- length. friendly.  It was part of the Ediacara Biota that lived  This train technology off ers greener and on Earth 20 million years prior to the quieter alternative to diesel on non- ‘Cambrian explosion’ of modern animal electrifi ed railway lines. life.  These hydrogen trains are manufactured  The ‘Cambrian explosion’ was when by French TGV-maker Alstom and are complex animals and other macroscopic commercially running on 100km route organisms — such as molluscs, worms, between towns and cities of Cuxhaven, arthropods and sponges — began to Bremerhaven, Bremervoerde and dominate the fossil record. Buxtehude in northern Germany. Hydrogen trains 8. Correct Option: (d)  Hydrogen trains are equipped with fuel Explanation: cells that produce electricity by combining Astra Missile hydrogen with oxygen. This conversion process only emits steam and water, thus  Statement 1 is incorrect: Astra producing zero emissions. Excess energy is air to air beyond visual range produced is stored in ion-lithium batteries Missile, indigenously developed by on board train. Defence Research and Development Organisation (DRDO).  These trains also make very little noise.   More than 50 private and public sector Moreover, hydrogen fuel cells have industries are involved in development advantages over batteries. Instead of and production of diff erent sub-systems recharging, they can easily be re-fuelled of the missile. like gas or diesel engine.   It is one of the smallest weapon system It is also easier to build re-fuelling developed by DRDO, having length infrastructure for these trains at railway of 3.8-metre and weighing 154kg. It is stations. single stage solid fuelled missile and has  These trains can run for around 1,000 km payload capacity of 15 kg conventional on a single tank of hydrogen, similar to explosives. the range of diesel trains.  Statement 2 is incorrect: It can be  These trains off er attractive prospect launched from diff erent altitudes and is GS SCOREcapable of engaging targets at varying to many cities scrambling to combat air pollution. range and altitudes at both short-range targets (up to 20 km) in tail-chase mode  The only disadvantage these hydrogen and long-range targets (up to 80 km) in trains are that they are more expensive head-on mode. than fossil fuel-based trains.  It is radar homing supersonic missile having maximum speed of Mach 4 (four 7. Correct Option: (d) times speed of sound). It possesses high Explanation: Single Shot Kill Probability (SSKP) making it highly reliable. Dickinsonia  It is all-weather missile with active radar  Statement 1 is incorrect: It is the earliest terminal guidance, excellent electronic animal on geological record — a strange counter-counter measure (ECCM) oval creature with rib like segments features, smokeless propulsion and

4 process improved eff ectiveness in multi- resistant to all known antibiotics and can target scenario. cause severe infections or even death is  It has advance on-board electronic spreading undetected through hospital counter-measures that jam radar signals wards across the world. from enemy radar, making tracking of the  They have discovered three variants of missile diffi cult. multidrug-resistant superbug in bacteria  It is fi tted with terminal active radar-seeker known as Staphylococcus epidermidis in and an updated mid-course internal samples collected from 10 countries, guidance system that helps missile to including strains in Europe that cannot locate and track targets. The missile can be controlled by any drug currently in the be integrated with all fi ghter aircraft of market. IAF including Sukhoi-30 MKI, Mirage-  Scientists have found that bacterium 2000, MiG-29, Jaguar and the Tejas Light known as Staphylococcus epidermidis are Combat Aircraft (LCA). related to better-known and more deadly In News MRSA superbug.  Indian Air Force (IAF) has successfully  Both the statements are incorrect: This undertaken series of fl ight trials of Astra bacterium is found naturally on human Beyond Visual Range Air-to-Air Missile skin and most commonly infects elderly (BVR-AAM) from September 26 to or patients who have had prosthetic October 3, 2018. The tests were conducted materials implanted, such as catheters at Integrated Test Range (ITR), Balasore, and joint replacements. Odisha as part of fi nal development trials of the missile.  It can be deadly, but it is usually in patients who already are very sick in hospital and can be quite hard to eradicate and the 9. Correct Option: (d) infections can be severe. Explanation:  Some strains of Staphylococcus HOPE Probe epidermidis bug have made mall change  Both the statements are incorrect: The in its DNA that led to resistance to two of probe will be built by an Emirati team of most common antibiotics. engineers and experts and will be sent on a scientifi c voyage of discovery to 11. Correct Option: (b) the Red Planet. Explanation:  This will mark the Arab world’s entry into the era of space exploration and place the Cyclone-30 UAE among the major scientifi c countries  Statement 1 is incorrect: Cyclotron that have begun programmes to explore is used to produce radioisotopes for Mars. diagnostic and therapeutic use for cancer  The probe will be sent to explore the care. Radiations from these isotopes Red Planet by 2020. Following a journey are used to destroy cancer cells. It will of several months, the probe is expected produce radioisotopes vital for diagnosis to enter the RedGS Planet’s orbit in SCORE 2021, and treatment of cancer. coinciding with the 50th anniversary of  the formation of the UAE. Statement 2 is correct: It will be fi rst and only cyclotron facility in country to produce In News: Germanium 68 radioisotopes, which is used  UAE has started its preparations for the in diagnosis of breast cancer. upcoming Mars Mission named - HOPE.  It will also produce Palladium 103 isotopes, which is used for the treatment 10. Correct Option: (d) of prostate cancer. In its future stages, it will also produce Iodine 123 isotopes, Explanation: which can help detect thyroid cancer. Superbugs:  It started working for fi rst time when 30  Researchers from University of Melbourne, MeV beam reached Faraday Cup (a metal Australian have found that superbug cup designed to catch charged particles

5 in vacuum). The beam from this facility In News: was used to produce fl uorine-18 isotope  Kempegowda International Airport (KIA) for preparation of radio-pharmaceutical in Bengaluru, capital of Karnataka is set to fl uorodeoxyglucose (FDG), which is used become fi rst airport in Asia to introduce for diagnose various types of cancer. face recognition based passage system  It will start regular production by mid- technology from 2019. 2019 after supporting nuclear systems and  For this, Bengaluru International Airport regulatory clearances are commissioned. Ltd. (BIAL), which operates KIA has signed In News: agreement with Portuguese software fi rm Vision Box.  India’s biggest cyclotron facility named Cyclone-30 became operational at -based Variable Energy Cyclotron 13. Correct Option: (a) Centre (VECC), which comes under Explanation: Department of Atomic Energy (DAE). EyeROV TUNA  12. Correct Option: (c) EyeROV TUNA has been designed and developed indigenously by Kochi-based Explanation: start-up IROV Technologies Pvt Ltd (EyeROV). Facial Recognition technology  Option (a) is correct: It is smart micro-  Statement 1 is correct: The face ROV (Remotely Operated Vehicle) or recognition based passage system will underwater robotic drone. allow passengers to board fl ights and move across diff erent sections of the  It design is at par with global standard airport using automatic biometric and has operational capabilities to work boarding procedure instead of manual in harsh and mission-critical underwater boarding passes verifi cation. In this environment. case, face will serve as boarding pass. The  It is designed to perform visual inspection fi rst implementation milestone of this and surveys of submerged structures up to system at KIA will be completed in the fi rst depth of 100 metres. It can be-controlled quarter of 2019, with Jet Airways, Air Asia using laptop or joystick. and SpiceJet passengers as fi rst users.  It is fi tted with camera that helps to give  Face recognition based passage system live HD video feed of the submarine will simplify journey of air passengers by environment. making it paperless from registration to  It is also equipped to perform variety of boarding. functions, including inspection of ship  Biometric technology will identify hulls, undersea cables or bridge moorings, passengers by their face as they move fi sh farms, dams, port structure and across airport, avoiding stops and bridge foundations and also in various repeated presentation of boarding passes, underwater research operations. passports or GS other physical identitySCORE  Its commercial use will eliminate need for documents. costlier and riskier manual inspection by  It will enable seamless journey of divers passengers, without obstacles, waiting In News lines or hassles, from registration to boarding.  India’s fi rst underwater robotic drone EyeROV TUNA was handed over  Statement 2 is correct: On complete to Naval Physical and Oceanographic implementation, this will be fi rst end- Laboratory (NPOL) of Defence Research to-end face recognition-based walk and Development Organisation (DRDO). through experience in Asia and largest This underwater drone will be used by in the world. It is also one of the NPOL for research and development most signifi cant steps towards Digital activities which in turn would result India campaign endorsed by Central in commercial product for defence Government. purposes.

6 14. Correct Option: (b)  The programme aims at both prevention and treatment of hepatitis which is among Explanation: leading causes of liver cancer, cirrhosis of Supercomputer Pratyush liver and acute liver failure.  Statement 2 is incorrect: Supercomputer  Statement 2 is correct: It aims to Pratyush will be national facility for treat minimum of 3 lakh hepatitis C improving weather and climate forecasts cases over a period of three years for and services under umbrella of the eliminating deadly condition by 2030. Ministry of Earth Sciences (MoES).  The programme is part of National Health  Pratyush has 6.8 PF computational power Mission. installed at two MoES Institutes.  Statement 1 is incorrect: Under it,  4.0 Peta Flops HPC facility at IITM, Pune expensive antiviral for hepatitis B and and 2.8 Peta Flops facility at NCMRWF, C infections will be made available free Noida. (PF is a measure of a computer’s of cost at all government hospitals. processing speed).  It will set up and upgrade facilities for  It will help India with better forecasts diagnosis and treatment primarily of in terms of monsoon, extreme events, hepatitis B and C. cyclones, tsunamis, earthquakes, air  These designated treatment centres quality, lightning, fi shing, hot and cold will provide free anti-viral to hepatitis C waves, fl ood and drought among others. patients.  Pratyush is fourth fastest supercomputer  They will also provide hepatitis B vaccine in world dedicated for weather and climate to babies born to mothers carrying the research. It follows supercomputing virus within 24 hours of birth. machines in Japan, US and United Kingdom. Key strategies under the Program  Statement 1 is correct: It will improve  Preventive and promotive interventions rankings of Indian supercomputer from with focus on awareness generation. 300s to 30s in Top 500 list, a respected  Safe injection practices and socio-cultural international tracker of the world’s fastest practices, sanitation and hygiene, safe supercomputers. drinking water supply, infection control  The key function of the High Performance and immunization Computing (HPC) facility will be monsoon  Increasing access to testing and forecasting using dynamical model which management of viral hepatitis. requires simulating weather for given  month and letting custom-built model Promoting diagnosis and providing calculate impact of actual weather over treatment support for patients of hepatitis coming months. B and C through standardized testing and management protocols with focus on  It will enable to map regions in India at treatment of hepatitis B and C. resolution of 3 km and globe at 12 km. GS SCORE Co-ordination and collaboration with In News diff erent Ministries and departments.  India’s fastest and fi rst multi-petafl ops  Building capacities at national, state, (PF) supercomputer named Pratyush district levels and sub-district level up to was unveiled at Pune-based Indian Primary Health Centres (PHC) and health Institute of Tropical Meteorology (IITM). and wellness centres to scale program till The supercomputer has been named as lowest level of the healthcare facility in a ‘Pratyush’ meaning Sun. phased manner. In News: 15. Correct Option: (b)  Union Ministry of Health and Family Explanation: Welfare in collaboration with the National Viral Hepatitis Control World Health Organisation (WHO) has Programme launched National Viral Hepatitis Control

7 Programme to control viral hepatitis C substantiate and revolutionise his theory on the occasion of World Hepatitis Day of photoelectric eff ect. Einstein himself (July 28). The programme aims to combat translated Bose’s paper into German viral hepatitis and reduce mortality and and sent it to Zeitschrift für Physik with morbidity associated with it. his endorsement for publication. With his demigod status, Einstein’s words 16. Correct Option: (c) carried much weight. It was promptly published, and immediately Bose shot Explanation: into prominence. Satyendranath Bose (1894-1974)  After this, Einstein personally invited Bose to work with him, and their eff orts  He was an Indian Physicist best known for culminated in the Bose-Einstein his role in particle Physics. statistics, an important and seminal  Bose was a specialist in mathematical phenomenon in quantum physics. His physics. work was wholeheartedly supported  A fellow of the Royal Society, Bose was and appreciated by the leading lights awarded Padma Vibhushan in 1954, in quantum theory, such as Louise de incidentally, the Padma Vibhushan was Broglie, Erwin Schroedinger, Paul Dirac awarded for the fi rst time in 1954 only. and Heisenburg.   Statement 1 is correct: Bose never Statement 2 is correct: In honour of received a doctorate nor was he Bose, Paul Dirac coined the word ‘Boson’ awarded a Nobel Prize. He is largely for those particles which obey Bose’s forgotten and Nobel Committee has statistics. In atomic theory, only Fermions recognized all other scientists that did (named after Enrico Fermi) and Bosons work on the concept, whose historic were named after physicists. father was Satyendranath Bose. Contribution of Indian Scientists 17. Correct Option: (c)  Meghnad Saha was his classmate and J Explanation: C Bose as his teacher. In 1921, he joined Mass Embryo Transfer (ETT) the then newly created Dacca University  as Reader in Physics. While teaching, he ETT is one of the most important wrote a paper for deriving the Planck’s reproductive biotechnologies where male Law. His paper was titled ‘Planck’s Law and female genetic material can be utilized and Light Quantum Hypothesis.‘ for faster improvement of livestock.   In 1900, Max Planck had explained in the It has revolutionized breeding strategies theory of black body radiation that light in Bovines as tool to optimize genetic is emitted in discrete amounts (quanta) improvement in cattle. rather than as a continuous wave. But  Using it, embryos of higher genetic merit his derivation of this formula was not indigenous bovines such as Sahiwal, Gir, satisfactory to other scientists, in fact Red Sindhi, Ongole, Deoni and Vechur are even to himself.GS Later, Albert EinsteinSCORE being transferred in to surrogate cows. explained the photoelectric eff ect based  on Planck’s quanta as photons in 1905 in Government is taking this technology to a paper. Einstein was awarded the Nobel doorstep of farmers for rapid propagation Prize for this paper, not for his papers on of high genetic merit indigenous cattle. Relativity! Benefi ts of using ETT:  However, many of his colleagues were  Statement 1 is correct: Farmer can not fully convinced of his yet-to-be- get 5-6 fold increase in number of off - developed photon theory. Under these springs. circumstances, Bose re-sent the paper  to Albert Einstein in June 1924, with a So born calves will be of high genetic fervent appeal for his perusal. Einstein merit immediately recognised the signifi cance  Statement 2 is correct: Off -springs born of this paper. This paper was going to will be free from diseases.

8 18. Correct Option: (c) the leasing of a nuclear-powered attack submarine for the Indian Navy for a period Explanation: of 10 years. Scorpene Submarine  Statements 2 is correct: Russia will be  Statement 1 is correct: These are diesel- delivering the Akula class submarine, to electric attack submarines equipped be known as Chakra III, to the Indian Navy with anti-ship missile. by 2025.  The fi rst four submarines will be  India had earlier leased two nuclear conventional, while last two will be submarines from Russia. They are INS equipped with the Air Independent Chakra leased in 1988 under a three-year Propulsion (AIP) system, which will enable lease and second INS Chakra was taken them to stay underwater for longer on lease in 2012 for a period of 10 years. duration.  Russia will lease Akula class submarine for  The state-of-the-art Scorpene submarines the period of 10 years at the cost of USD have superior stealth and ability to launch 3 billion. crippling attacks with precision-guided  weapons. Chakra III will be equipped with Indian communication systems and sensors,  Statement 2 is correct: The attacks can including the indigenously-developed be carried out with torpedoes both USHUS integrated sonar system and while submerged or on surface — in all Panchendriya sonar. war theatres, including the tropics.  Chakra III get a unifi ed submarine sonar  INS Karanj has been named after older and tactical control system, which are submarine as per Indian naval tradition already in use on the INS Arihant. The that was in operation from 1969 to 2003. Chakra III will also get a refurbished hull. It served country for 34 years and was  also part of operations in 1971. Chakra III weighing around 8,140-ton will have a submerged speed of 30 knots and  INS Karanj has an overall length of 67.5 an operating depth of 530 meters and will metres and height of about 12.3 metres. be capable of carrying a crew of 73. Its hull form, fi n and hydroplanes are  specifi cally designed to produce minimum Chakra III will also be loaded with four underwater resistance. 650-millimetre and four 533-millimetre launch tubes for fi ring Type 65 and Type  In all, six scorpion submarines being 53 torpedoes. Both of these torpedoes indigenously built at MDL under Project are made in Russia. 75 with help of French naval defence and energy company DCNS. The fi rst of  Chakra III will be powered with 190 mW these, INS Kalvari was commissioned in nuclear reactor. December 2017 and second, INS Khanderi is undergoing sea trials. Remaining 4 20. Correct Option: (b) submarines will be inducted gradually by 2020. GS SCOREExplanation: In News: Down Syndrome  Indian Navy has launched Scorpene  Down Syndrome is a chromosomal submarine INS Karanj at the Mazagon condition associated with intellectual and Dock Shipbuilders Ltd (MDL), , learning disabilities. Maharashtra. It the third Scorpene class  Delayed development and behavioural submarine built by MDL under ambitious problems are often reported in children Project 75 of the Indian Navy. with Down Syndrome.  Statement 1 is incorrect: It is not a 19. Correct Option: (b) genetic disorder. Explanation:  Persons with Down Syndrome have 47  Statements 1 is incorrect: India and chromosomes as compared to 46 in Russia have signed an agreement for majority of people.

9  This one Extra chromosome causes its 22. Correct Option: (a) own impact and slows down learning process. Explanation:  Statement 2 is correct: It impacts Machine to Machine (M2M) neurological functioning causing Communication impairment of intellectual functions e.g.  Statement 1 is correct: Unlike person- analytical thinking, complex abstractions to-person (P2P) communication, M2M and judgement. technologies involve communication  People with Down Syndrome can and between two devices. do lead full and rewarding lives and  The technology is basis for automated contribute as valued and equal members information exchange between of their communities. machines.  Every year 20 to 22 thousand children take  Statement 2 is incorrect: It enables birth with Down Syndrome in our country wired and wireless devices to talk to and the need of the hour is to bring them each other using sensors. in the mainstream.  It is also a key component of the emerging In News: Internet of Things (IoT) revolution.  The National Conference was organized  TRAI acknowledged its potential in by The National Trust for the welfare of National Telecom Policy 2012. persons with Autism, Cerebral Palsy, Mental  Retardation and Multiple Disabilities Applications of self-communicating under M/o SJ&E in collaboration with machines using M2M communications Muskaan, a registered organisation of the can be deployed in new-age infrastructure National Trust. projects and various industry verticals such as smart cities, smart grids, smart homes, smart transportation and smart 21. Correct Option: (c) heath sectors. Explanation: Raman Eff ect 23. Correct Option: (c)  Statement 1 is correct: The Raman Explanation: Eff ect is a change in the wavelength of MILAN Exercise light that occurs when a light beam is defl ected by molecules.  Statement 1 is correct: MILAN is  congregation of littoral navies Some part of light beam after passing conducted biennially by Indian Navy through a transparent medium gets under aegis of the Andaman and scattered. Nicobar Command.  This phenomenon of scattering of light  Statement 2 is incorrect: It was fi rst is termed as Raman Scattering and the held in 1995. In the fi rst edition, only four cause of scattering is called the Raman littoral navies had participated in it. Eff ect. GS SCORE  Now it has now grown into prestigious  The wavelength of these scattered rays is international event and encompasses diff erent from that of the incident rays of participation by maritime forces from not light. just Bay of Bengal and South East Asia but  This phenomenon was explained/ larger Indian Ocean Region (lOR). discovered by Indian physicist  It provides eff ective forum to participating Chandrasekhara Venkata Raman (CV navies to come together to discuss Raman) on February 28, 1928. common concerns in the Indian Ocean  Statement 2 is correct: This discovery Region and forge deeper cooperation was awarded with the Nobel Prize in among friendly navies. Physics in 1930.  Milan 2018 will witness diverse mix of  Feb 28 is observed as National Science professional exercises and seminars, Day in India. social events and sporting fi xtures.

10  The interactions at event encompass  Statement 1 is correct: Tamil Nadu sharing of views and ideas on maritime Defence Industrial Corridor would be a good-order and enhancing regional specialised facility of aero component cooperation for combating unlawful manufacturing. activities at sea.  Statement 2 is incorrect: Tamil Nadu  Statement 3 is correct: 16 countries Defence Industrial Corridor is the participating in this edition of exercise country’s second defence industrial include Australia, Malaysia, Maldives, corridor after the fi rst one was launched Mauritius, Myanmar, New Zealand, Oman, in Aligarh, Uttar Pradesh August last Vietnam, Thailand, Tanzania, Sri Lanka, year. Singapore, Bangladesh, Indonesia, Kenya  Tamil Nadu currently ranks 4th in exports and Cambodia. with 9.8% of India’s total exports. The  The social interactions planned in Milan corridor will further expand the exporting 2018 include display by bands of the opportunities from the state. Indian Navy and Indian Army, ship visits In News: and colorful cultural evenings. Milan 2018 aims to showcase the rich heritage and  The Union Defence Minister Nirmala pristine natural beauty of A&N Islands to Sitharaman inaugurated the Tamil Nadu foreign visitors. Defence Industrial Corridor. During the inauguration, investments worth over In News: Rs 3,038 crore in this defence corridor  The Indian Navy is going to host 16 were announced. Lockheed Martin, a countries for multi-national 2018 MILAN global security giant, also announced its exercise at Port Blair in Andaman and intention to invest in the corridor. Nicobar (A&N) islands from March 6 to 13, 2018. The eight-day mega naval 25. Correct Option: (c) exercise will be held under aegis of Andaman and Nicobar Command with Explanation: theme of ‘Friendship Across the Seas’. Nerve Agents  24. Correct Option: (a) Statement 1 is correct: Nerve gases are among the most lethal form of chemical Explanation: weapons. Defence Industrial Corridor  They notably have no use other than in chemical warfare.  The important features about the Tamil Nadu Defence Industrial Corridor are:  The CWC lists chemicals under various degrees of manufacturing restriction.  It is also called as Tamil Nadu Defence Production Quad as the nodal cities of  Under this, the nerve gases are among Chennai, Hosur, Salem, Coimbatore and the most restricted. Tiruchirappalli form a quadrilateral with  Nerve gas weakens the mechanism within one of them at the centre. the body responsible for the conduction  The fi ve nodal citiesGS already have existingSCORE of nerve impulses. defence ecosystem in the form of ordnance  Acetyl cholineesterase is a compound factory boards, vendors working with that catalyses the breakdown of the defence PSUs, and other allied industries. neurotransmitter acetylcholine.  Together with helping in accelerated  Nerve gas prevents acetylcholinesterase growth and regional industry from performing its normal function of agglomeration, the defence corridor will breaking down acetylcholine. also facilitate a well-planned and effi cient industrial base which will lead to increased  It leads to the muscles going into a state defence production in the country and of uncontrolled contraction, a sign of the region. paralysis or a seizure-like state.  The corridor will help the industry to  Death usually happens because paralysis integrate with the global supply chain of extends to the cardiac and respiratory Defence manufacturing. muscles.

11  Other symptoms could include dilation of  As per State reports till 25.01.2019, 40% pupils, sweating and gastrointestinal pain notifi ed TB patients, 35% treatment etc. supporters and 8% private practitioners have been paid incentives through DBT.  Nerve agents can also be absorbed through the skin. Elimination of Tuberculosis What is Novichok?  The government has envisioned following strategic steps to eliminate TB by 2025:  As restrictions on weapons are based on  chemical formulae, newer molecules can A partnership with the private sector bypass restrictions. has been envisaged under Joint Eff ort for Elimination of Tuberculosis (JEET)  Countries thus started to develop newer for the elimination of TB by 2025, fi ve weapons to bypass the restrictions. years ahead of the global schedule.  Statement 2 is correct: This naturally  To increase the reporting from the led to the emergence of nerve agents private sector, public-private support and that is how Novichok evolved. agency approach has been initiated.  Novichok is said to be 5-8 times more  Indian Medial Association has been lethal than VX nerve agent. roped in for large scale sensitization and advocacy with private practitioners  Also, its eff ects are rapid, usually within 30 with standardised digital material. seconds to 2 minutes.  States have been supplied anti-TB 26. Correct Option: (a) drugs to provide for TB patients notifi ed from private sector based on Explanation: demand. Tuberculosis in India  States have been guided to keep  programme provided anti-TB drugs India accounts for about a quarter of the at private practitioner’s clinic or global TB burden. pharmacy  Statement 1 is incorrect: India has set  To ensure the TB patients are not the target to eliminate Tuberculosis in deprived of DBT benefi t fl exibility to India by 2025. provide the benefi t through the  To evolve a strategy in this regard, India has existing bank account of a blood put in place National Strategic Plan 2017 relative has been given. – 2025 which sets out the government  States have also been advised to plans of how the elimination of TB can be facilitate opening of zero balance achieved. accounts for TB patients, if necessary, under the Pradhan Mantri Jan Dhan What does the Elimination of a disease stand Yojana (PMJDY) and Indian Postal for? Bank.  Statement 2 is correct: Elimination of a In News: disease is defi GSned by the World HealthSCORE Organisation (WHO) as there should be  The Minister of State for Health and Family less than 1 case of TB for a population Welfare, Shri Ashwini Kumar Choubey has of a million people. stated in parliament that government is  committed to achieve the target of TB As a step towards eliminating TB, India elimination by 2025 and not considering has made following progress in 2018: any revision in targets.  In 2018, 5, 36,752 TB patients have been notifi ed from the private sector. There has 27. Correct Option: (c) been a 40% increase in TB notifi cation from the private sector as compared to Explanation: 2017. Sapochi  44,517 (8%) TB patients were given anti-  Statement 1 is correct: Cybersecurity TB drugs from the programme. agencies have detected a new malware

12 called Saposhi that can take over  Option (b) is correct: The fi rst phase electronic devices and use them for Shakti Sthala solar park having total Distributed Denial of Service (DDoS) capacity of 2,000 megawatts (MW) attacks. was inaugurated in drought-prone Pavagada region of Tumkur district,  Saposhi Malware is capable of taking Karanataka. It is claimed to be world’s over electronic devices and turning largest solar park. them into bots (device taken over by  malware) which can then be used for any The park ties in with the Central purpose, including DDoS attacks which, Government’s scheme to generate 100 gigawatts (GW) of solar power by 2020. It with enough fi repower, can cripple entire has been executed within record time of industries. two years, with zero land acquisition  Statement 2 is correct: Computer  The park’s development was initiated Emergency Response Team (CERT), a with creation of Karnataka Solar Power Central government body that deals Development Corp. Ltd (KSPDCL) in March with cyberattacks has so far has not 2015 as joint venture between Karnataka issued any alert regarding Saposhi Renewable Energy Development Ltd malware. (KREDL) and Solar Energy Corporation of  Saposhi is similar in its intensity to Reaper India (SECI). malware, which was taking over millions  KSPDCL had used “plug and play” model of devices at rate of 10,000 devices per for development of park, under which it day. In October 2017, CERT had issued acquires and develops land as blocks for alert about Reaper which is highly evolved solar power generation, embedded with malware capable of hacking devices like required government approvals and gives Wi-Fi routers and security cameras and it out to solar power developers (SPDs) also hiding its own presence in bot. through auctions.  The solar park is spread over 13,000 acres and fi ve villages. The land was taken on 25-year lease by government from around 2,300 farmers. In return, these farmers are paid annual rental of Rs.21,000 per acre, with scope for 5% increase every two years.  The fi rst phase of the Rs. 16,500 crore park will generate 600MW power, while the  In DDoS attacks, malware fi rst creates balance 1,400MW will be commissioned network of bots — called botnet and then in second phase by end year 2018. uses botnet to ping single server beyond The park will decrease dependence on its capacity at same time. traditional power sources and move to environmentally friendly ones to meet  As number of pings are far beyond server’s growing power needs of state. capacity, server crashes and denies service  to its consumers.GS Malwares like Saposhi,SCORE It will create employment and act as incentive for natives and farmers to Reaper and Mirai are primarily are used explore new opportunities of socio- for DDoS attacks. economic growth in the region. It will also  For example, if large botnet attacks server curb mass migration of people from the of fl eet cab provider, its server will crash, region which has been declared drought- and scores of consumers will be unable to hit in 54 of the last 60 years. avail of its services, causing chaos in daily commuting as well as massive losses to 29. Correct Option: (b) the company. Explanation: 28. Correct Option: (b) Rydberg polarons  Explanation: Statement 1 is incorrect: These atoms are held together by a weak bond and Shakti Sthala solar park is created at very cold temperatures.

13  It uses ideas from two diff erent fi elds: verify algorithms and also develop Bose Einstein Condensation and Rydberg complete 5G base station. It will help atoms. in manufacturing of 5G base stations  Statement 2 is correct: A ‘Rydberg atom’ in India with support and collaboration is an atom in which an electron has from industry. It will also be instrumental been kicked out to a very large orbit. in generating skilled manpower in this technology.  These have interesting properties and  have been studied for a long time. In this Massive MIMO lab, several antennas are deployed at prototype base station to allow network to communicate 30. Correct Option: (a) reliably with very large number of mobile Explanation: terminals simultaneously at same time and on same frequency channel. Lyrid meteor shower  The large antenna array also improves  The annual Lyrid meteor shower is set to system power effi ciency as mobile reach a peak. terminals will now be required to radiate 10  Statement 1 is correct: The Lyrid meteor times lesser power than in 3G/4G systems. shower is one of the oldest known In regular 3G/4G base station technology, meteor showers only few antennas are deployed.  They are caused by the Earth’s annual trip through a cloud of dust and debris 32. Correct Option: (a) left behind by the comet C/1861 G1 Explanation: Thatcher. Gravity RAT Malware  Statement 2 is incorrect: It is classifi ed as a medium-strength shower.  The RAT was fi rst detected by Indian Computer Emergency Response Team,  A meteor shower happens when Earth CERT-In, on various computers in 2017. passes through the path of a comet. It is designed to infi ltrate computers and  When this happens, the bits of comet steal the data of users, and relay the stolen debris create streaks of light in the data to Command and Control centres in night sky as they burn up in Earth’s other countries. atmosphere.  The ‘RAT’ in its name stands for Remote  Bits of debris which enter Earth’s Access Trojan, which is a program capable atmosphere are called meteors. of being controlled remotely and thus  A meteorite is a solid piece of debris from diffi cult to trace. an object, such as a comet, asteroid, or  The latest update to the program by its meteoroid which survives its passage developers is part of GravityRAT’s function through the atmosphere to reach the as an Advanced Persistent Threat (APT), surface of a planet or moon. which, once it infi ltrates a system, silently GS SCOREevolves and does long-term damage. 31. Correct Option: (d)  Option (a) is correct: GravityRAT is Explanation: unlike most malware, which are designed to infl ict short term damage. It lies hidden  Option (d) is correct: The Indian Institute in the system that it takes over and keeps of Technology (IIT)-Delhi has installed penetrating deeper. Massive Multiple-Input, Multiple-Output (MIMO) lab on campus for standardisation,  GravityRAT has now become self aware and R&D and manufacturing of 5G equipment. is capable of evading several commonly It will be the fi rst of its kind 5G lab in the used malware detection techniques. country and has been set-up at Bharti  GravityRAT infi ltrates a system in the form School of Telecommunication Technology of an innocuous looking email attachment, and Management. which can be in any format, including MS  The lab will serve as 5G base station Word, MS Excel, MS PowerPoint, Adobe prototype that will be used to test and Acrobat or even audio and video fi les.

14  The other concern is that the Command  Statement 3 is correct: It is based and Control servers are based in several on feedback collected from Start-up countries. The data is sent in an encrypted ecosystem stakeholders, which include format, making it diffi cult to detect exactly start-ups, mentors, investors, accelerators, what is leaked. incubators and government bodies.  CERT-In had issued an alert for it last  The parameters of this feedback focus on year, with advisory asking users to review all actions and initiatives undertaken by cybersecurity measures and update anti- states on or before March 2018. malware tools.  These parameters include having start- In News: up cell or helpline and mobile or web portal for queries, size of start-up mentor  Gravity RAT, a malware allegedly designed by Pakistani hackers, has recently been network created by state government and updated further and equipped with anti- number of key incubators for incubation malware evasion capabilities. support to start-ups.  It also gives greater thrust like seed funding 33. Correct Option: (d) support, women entrepreneurship. Explanation: In News:  Lassa Fever Statement 1 is correct: Department of Industrial Policy and Promotion  Lassa fever or Lassa hemorrhagic fever (DIPP) under Commerce Ministry (LHF) is caused by Lassa virus. has unveiled Start-up-India ranking  Statement 1 is incorrect: This disease framework to rank states/UTs based is mainly prevalent in West Africa on measures regions that have taken to countries such as Nigeria, Liberia, Sierra foster entrepreneurship. Leone, Guinea, and Ghana.  It was launched by Union Minister of  It results in 5000 deaths every year. Commerce and Industry at a function held in New Delhi.  Statement 2 is incorrect: It’s a zoonotic disease transmits to human from a species of mouse. 35. Correct Option: (c) Explanation: 34. Correct Option: (d) Nipah Virus (NiV) Infection Explanation:  Statement 1 is incorrect: Nipah virus Start-up States and UTs Ranking (NiV) infection is zoonotic disease that Framework causes severe disease in both animals and humans.  Statement 2 is correct: The framework aims to foster competitiveness and  Fruit bats of the Pteropodidae Family, propel states/UTS to act pro-actively in Pteropus genus are natural host of the start-up matters.GS SCOREvirus.  This ranking will help states to bring  Statement 2 is correct: It was fi rst progress made in their start-up ecosystem identifi ed during outbreak of disease and also make diff erent states learn and that took place in Kampung Sungai replicate good practices. Nipah, Malaysia in 1998.  Its objective is to encourage States/  The virus spread fast and is mostly fatal. UTs to take proactive steps towards Infected bats shed virus in their excretion strengthening Start-up ecosystems at and secretion. local level.  Statement 3 is incorrect: The virus  It will also measure impact of each cannot be transmitted through air. But step initiated at local level for building it is transmitted through direct contact strong Start-up ecosystem. It will also with infected bats, pigs. Human to Human enable continuous learning through the transmission from other NiV-infected dissemination of good practices. people is also reported

15  NiV infection can cause acute respiratory  These satellites have payload called Laser syndrome, breathing trouble, infl ammation Ranging Interferometer. It can precisely of the brain, fever, headache, drowsiness, measure gravity fi eld changes due to disorientation and delirium. Patient change in separation distance between can slip into coma within 48 hours. The two satellites, revealing information about mortality rate of patients infected with it what kinds of features they are fl ying is reportedly 70%. It is capable of causing over. Earth is not perfect sphere) diff erent diseases in domestic animals too. The features, like mountains and oceans, across  There is no vaccine for disease either for its surface, so gravitational pull exerted humans or animals. The main treatment on these satellites is not consistent. When for those infected is intensive supportive gravity fi eld changes, separation between care and supportive medicines. In disease two satellites changes slightly. prone areas, fruits strewn on the ground In News: should not be eaten, for safety.  SpaceX has successfully launched twin In News: NASA satellites GRACE-FO (Follow-On)  Kerala was on high alert as rare and deadly that will track Earth’s water Cycle i.e. water Nipah Virus (NiV) infection which was movement and ice-melt. It was launched detected for the fi rst time in the state. Its on board of SpaceX’s pre-fl own Falcon 9 traces were found in blood and body fl uid Rocket from Vandenberg Air Force Base samples of two persons who died of viral in California, US. fever in Kerala’s coastal Kozhikode region. It was confi rmed by laboratory results 37. Correct Option: (a) from National Institute of Virology, Pune. Explanation: 36. Correct Option: (a) Quadrivalent vaccine Explanation:  While a trivalent infl uenza vaccine contains both A and B subtype viruses, GRACE-FO (Gravity Recovery and Climate it has only one of the B subtype virus, Experiment Follow-On) the quadrivalent vaccine off ers a greater  Statement 2 is incorrect: GRACE- breadth of protection as it includes both FO is joint project between National B subtype viruses. Aeronautics and Space Administration  It is because of a greater breadth of (NASA) and German Research Center protection that a few other companies for Geosciences (GFZ). too have shifted from a trivalent to a  Statement 1 is correct: It is follow-on quadrivalent vaccine. mission to original GRACE mission,  which had mapped Earth’s water and Statement 1 is incorrect: Since the ice by measuring changes in Earth’s vast majority of infl uenza vaccines gravity fi eld from 2002 to 2017. manufactured were trivalent till recently, the World Health Organisation  GRACE-FO will pick up where GRACE left (WHO) used to recommend two A off to continue study of rising sea levels, subtypes and one B subtype, plus an melting of glaciersGS and polar ice caps SCORE and optional fourth strain (the other B virus other changes in distribution of water on strain). Earth.  The quadrivalent vaccine will contain four  The two GRACE-FO satellites will orbit infl uenza virus strains (two A subtypes together at 490km altitude near-polar and two B subtypes — H1N1 and H3N2, orbit, circling Earth every 90 minutes. and Victoria and Yamagata respectively).  To measure Earth’s gravity, two satellites  The viruses used in the vaccine are will orbit around Earth together, with killed and this eliminates the possibility one trailing behind other at distance of the virus in the vaccine itself causing of 220km. The instruments on board of infection. these satellites are so sensitive that they can detect changes with precision of  Statement 2 is correct: In India, the about 1 micrometer (i.e. about one-tenth vaccine will be available as a single of a human hair over long distance). dose pre-fi lled syringe

16  Eventually, it will be available in a vial for  Statement 1 is incorrect: It is jointly public health use. developed by Pune based Armament In News: Research & Development Establishment (ARDE), Defence Research and  Sanofi Pasteur’s injectable infl uenza Development Laboratory (DRDL), vaccine (FluQuadri) containing two A virus Hyderabad and RCI. strains — H1N1 and H3N2 — and two B virus strains — Victoria and Yamagata —  Statement 2 is correct: It is fi red using for active immunisation of adults of age multi-barrel rocket launcher (MBRL) 18 to 64 years was approved in May last which can fi re 12 Rockets loaded with year by the Drug Controller General of explosives within 44 seconds and destroy India (DCGI). target area of 4 sq km at a time.  Sanofi ’s quadrivalent infl uenza vaccine  It has capability to incorporate several was licensed for use by the U.S. Food and types of warheads makes it deadly Drug Administration (FDA) in 2013; it is for enemy as it can even destroy solid licensed in 26 countries. structures and bunkers.  38. Correct Option: (a) The quick reaction time and high rate of fi re of system gives an edge to Army Explanation: during low amount confl ict situation. It KATRIN experiment already has been inducted into Indian Army.  The KATRIN experiment is currently set up and commissioned on the Campus, North Upgraded Pinaka of the Karlsruhe Institute for Technology.  The guided version is Pinaka rocket  The experiment is a collaboration between (Pinaka mark-II) has evolved from earlier national and international partners version of unguided Pinaka mark-I. with currently more than 150 scientists,  engineers, technicians and students. The upgraded Pinaka mark-II has been fi tted with navigation, guidance and  Option (a) is correct: KATRIN measures control kit developed by Research Centre, the neutrino mass in a model- Imarat (RCI). independent way via ultrahigh precision measurements of the kinematics of  The conversion has helped to enhance electrons from beta-decay. range of Pinaka to more than 70 km (from earlier 40 km) and accuracy to 50m (from Neutrinos: earlier 500m).  Neutrinos are the most abundant massive elementary particles in nature. Due to In News: their minimalistic properties they are key  The Defence Research and Development particles for understanding physics on Organisation (DRDO) has successfully the smallest scale (elementary particle test-fi red upgraded guided version of physics) up to the largest scale – the Pinaka Rocket from Proof & Experiment universe (cosmology). Establishment (PXE) at Chandipur, Odisha.  Neutrinos areGS the only elementary SCORE During the tests, two rounds were particles of matter, which do not carry conducted and some more rounds have electrical or strong charge and thus are been planned. blind to the electromagnetic and the strong interaction and cannot be bound. 40. Correct Option: (c)  In the context of particle physics they participate only in the weak interaction. Explanation: This made neutrinos the most prominent candidate to explore with them the Coloured X-Ray on Human properties of the weak interaction.  Statement 1 is correct: The new device is based on the traditional black-and- 39. Correct Option: (a) white X-ray but incorporates particle- tracking technology called Medipix Explanation: developed by European Organization for Pinaka Missile Nuclear Research (CERN).

17  The Medipix technology developed by  It will provide platform to best minds CERN works like camera detecting and from industry, academia, start-ups counting individual sub-atomic particles and government to drive culture of as they collide with pixels while its shutter collaboration and co-creation to fi nd is open. solutions to accelerate India’s economic development.  Statement 2 is correct: This allows for high-resolution, high-contrast  It will also serve as platform for intelligence- pictures. sharing and technology collaboration between stakeholders to build collective  Its small pixels and accurate energy IoT capabilities. resolution makes this new imaging tool able to get images that no other imaging tool can achieve. 42. Correct Option: (c)  The technology is being commercialised Explanation: by New Zealand Company MARS Bio World Intellectual Property Organisation imaging, linked to the universities of (WIPO) Canterbury and Otago which helped  Statement 1 is correct: It is the global develop it. body for promotion and protection of  This colour X-ray imaging technique intellectual property rights (IPR). can produce clearer and more accurate  Statement 2 is correct: It is one of the 15 pictures and help doctors give their specialized agencies of United Nations patients more accurate diagnoses. The (UN). images very clearly show diff erence  between bone, muscle and cartilage and Statement 3 is correct: It was established also the position and size of cancerous in 1967 and is headquartered in Geneva, tumours. Switzerland.  In News: Its mandate is to encourage creative activity, to promote the protection of  Scientists from New Zealand have intellectual property throughout the performed world’s fi rst-ever 3-D, colour world. X-ray on human. It has potential to  Statement 4 is incorrect: It encourages improve the fi eld of medical diagnostics. and provides assistance to all its 188 member countries in formulating national 41. Correct Option: (d) IPR policy however it does not dictate or prescribe any mandatory measures. Explanation:   India is a member of WIPO and party to Statement 2 is correct: Indian IT several treaties administered by WIPO. industry’s apex body National Association of Software and Services Companies (NASSCOM) has launched its Centre of 43. Correct Option: (c) Excellence for the Internet of Things (CoE Explanation: – IoT) at Gurugram, Haryana. GS SCORE5G  Statement 3 is correct: This centre was  launched in collaboration with Haryana Statement 1 is correct: 5G is wireless government and is part of nationwide communication technology based on collaborative initiative involving the third-generation partnership project (3GPP). Ministry of Electronics and Information Technology (MeitY).  It is next generation mobile networks technology after 4G LTE networks.  Statement 1 is correct: It aims to act as perfect collaboration for innovation and  It is expected to off er enhanced mobile high-end technologies. broadband (eMBB) through R15 repeaters that can cater to coverage requirements.  It will provide one of the largest innovation platforms for enabling IOT  Statement 2 is correct: The 5G wireless revolution through connected devices technology will off er far greater upload using emerging technologies. and download speed i.e. 100 times

18 faster internet data speed than current  Option (b) is correct: It is a new mineral 4G networks. discovered in a meteorite in Eastern Russia.  It has potential to demonstrate spectral effi ciency and potential for diversifi ed  It is named “uakitite” after the Uakit services such as Internet of Things (IoT) region of Siberia where the meteorite was and augmented reality (AR)/virtual reality discovered. (VR).  98% of the Uakit meteorite is an iron alloy  High data speed off ered by 5G network called kamacite, which so far has only will help cloud systems to stream software been found in other meteorites. The other updates, music, and navigation data to two percent is comprised of minerals that driverless cars. Moreover, it holds the form in space. key to growth of artifi cial intelligence (AI)  Scientists don’t know a lot more about systems and enhances IoT. this mysterious space rock as they were unable to obtain all physical and optical 44. Correct Option: (b) properties of the mineral because of its small size. Explanation: Ballistic Missile Interceptor Advanced Area 46. Correct Option: (a) Defence (AAD) Explanation:  AAD is an anti-ballistic missile designed to intercept incoming ballistic missiles in Scutoid the endo-atmosphere at an altitude of  Option (a) is correct: Scientists have 30 km (19 mi) identifi ed new shape called scutoid while  Statement 2 is correct: AAD is a single- studying epithelial cells. Epithelial tissue stage, solid-fuelled missile. is one of four kinds of tissue that forms human body which acts as safety shield  Guidance is provided by an inertial of body that make up cell walls lining our navigation system. blood vessels and organs.  The endo-atmospheric missile, capable  Scutoid shape has fi ve sides on one end of intercepting incoming targets at an and six on the other and a triangular altitude of 15 to 25 kms was launched surface on one of its longer edges. against multiple simulated targets of 1500 km class ballistic missile.  It is completely new to geometry and resembles beetle’s scutellum (shield-like  One target among simultaneously structure) from top-down view. incoming multiple targets was selected on real-time, the weapon system radars  Scientists had used computer modelling tracked the target and the missile locked and imaging to identify this shape. on to it and intercepted the target with a  It had produced strange, prism-like shape, high degree of accuracy. one with six sides on one end, fi ve on  The complete event including the other, and strange triangular protrusion engagement and interception was tracked coming out of one of sides. by a number ofGS electro-optical trackingSCORE  Scientists had used micro-copy and systems, radars and telemetry stations. computer imaging to supplement fi ndings In News: of computer modelling by taking up-close look at epithelial tissues of fruit fl ies and  Statement 1 is incorrect: DRDO has zebrafi sh. As predicted, they discovered conducted the successful test of the scutoid shape. Ballistic Missile Interceptor Advanced Area Defence (AAD) from Abdul Kalam Island, Odisha. 47. Correct Option: (b) Explanation: 45. Correct Option: (b) National Wildlife Genetic Resource Bank Explanation:  Option (b) is correct: National Uakitite Wildlife Genetic Resource Bank was

19 inaugurated at Centre for Cellular and  Statement 3 is correct: It would also Molecular Biology’s (CCMB) Laboratory be an apex institution in the fi eld of of Conservation of Endangered Species food technology and management, (LaCONES) facility in Hyderabad, networking and co-ordinating with other Telangana. institutions in the same fi eld in India and Abroad.  It is India’s fi rst genetic resource bank where genetic material will be stored for posterity which will further the 49. Correct Option: (b) cause of conservation of endangered and Explanation: protected animals. Two-factor authentication (2FA)  It is equipped with sophisticated equipment to preserve the genetic  Two-factor authentication (2FA), resources that could be utilised to virtually sometimes referred to as two-step resurrect an animal species in case it goes verifi cation or dual factor authentication, extinct. is a security process in which the user provides two diff erent authentication  It will cryopreserve living cell lines, factors to verify themselves to better gametes and embryos of endangered protect both the user’s credentials and wild animal species in India. For cryogenic the resources the user can access. preservation, researchers at CCMB- LaCONES will use liquid Nitrogen that  Two-factor authentication provides is cooled down to as low as minus 195 a higher level of assurance than degrees Celsius. authentication methods that depend  on single-factor authentication (SFA), in It will aid wild life conservation eff orts which the user provides only one factor by taking up artifi cial reproduction, -- typically a password or passcode. conducting studies in evolution biology and wildlife medicine.  Statement 1 is incorrect: Two-factor authentication methods rely on users  Thus, it will also help in protecting India’s providing a password as well as a second biodiversity and environment. So far this factor, usually either a security token or bank has collected and preserved genetic a biometric factor like a fi ngerprint or resources of 23 species of Indian wild facial scan. animals.  Statement 2 is correct: Two-factor authentication adds an additional layer 48. Correct Option: (a) of security to the authentication process Explanation: by making it harder for attackers to gain access to a person’s devices or National Institute of Food Technology online accounts, because knowing the Entrepreneurship and Management victim’s password alone is not enough to (NIFTEM): pass the authentication check.  Statement 1 is incorrect: NIFTEM  Two-factor authentication has long been is the brainchildGS of the Ministry SCORE of used to control access to sensitive systems Food Processing Industries (MoFPI) and data, and online service providers Government of India. are increasingly using 2FA to protect  MoFPI in its Vision document-2015, their users’ credentials from being used envisaged creation of a world-class by hackers who have stolen a password institution to cater to the various database or used phishing campaigns to stakeholders such as entrepreneurs, obtain user passwords. food processing industry, exporters, policy makers, government and existing 50. Correct Option: (b) institution. Explanation:  Statement 2 is correct: NIFTEM will work actively for assisting in setting up food P null standards, businesses incubation and can  A team of doctors, led by Shamee Shastry also include knowledge sharing. from the Blood Bank of Kasturba Medical

20 College (KMC) here, has identifi ed a  However, there are more than 200 minor rare blood group called “pp” or “P null” blood group antigens known besides A, phenotype. B and Rh.  Option (b) is correct: A press release  A blood type is considered rare if fewer issued by the Manipal Academy of Higher than one in 1,000 people have it. A person Education (MAHE) here recently said that is said to have rare blood group when ABO and Rh D are the commonly typed he lacks the high frequency antigen or blood group systems. multiple common antigens.

**********

GS SCORE

21